Adult Health Exam 1 Practice Q's

Réussis tes devoirs et examens dès maintenant avec Quizwiz!

The nurse is caring for a patient suspected of having ARDS. What is the most likely diagnostic test ordered in the early stages of this disease to differentiate the patients symptoms from those of a cardiac etiology? A) Carboxyhemoglobin level B) Brain natriuretic peptide (BNP) level C) C-reactive protein (CRP) level D) Complete blood count

B Feedback: Common diagnostic tests performed for patients with potential ARDS include plasma brain natriuretic peptide (BNP) levels, echocardiography, and pulmonary artery catheterization. The BNP level is helpful in distinguishing ARDS from cardiogenic pulmonary edema. The carboxyhemoglobin level will be increased in a client with an inhalation injury, which commonly progresses into ARDS. CRP and CBC levels do not help differentiate from a cardiac problem.

The nurse is creating the plan of care for a patient who is status postsurgery for reduction of a femur fracture. What is the most important short-term goal for this patient? Test Bank - Brunner & Suddarth's Textbook of Medical-Surgical Nursing 14e (Hinkle 2017) 391 A) Relief of pain B) Adequate respiratory function C) Resumption of activities of daily living (ADLs) D) Unimpaired wound healing

B Feedback: Maintenance of the patients airway and breathing are imperative. Respiratory status is important because pulmonary complications are among the most frequent and serious problems encountered by the surgical patient. Wound healing and eventual resumption of ADLs would be later concerns. Pain management is a high priority, but respiratory function is a more acute physiological need.

While the surgical patient is anesthetized, the scrub nurse hears a member of the surgical team make an inappropriate remark about the patients weight. How should the nurse best respond? A) Ignore the comment because the patient is unconscious. B) Discourage the colleague from making such comments. C) Report the comment immediately to a supervisor. D) Realize that humor is needed in the workplace.

B Feedback: Patients, whether conscious or unconscious, should not be subjected to excess noise, inappropriate conversation, or, most of all, derogatory comments. The nurse must act as an advocate on behalf of the patient and discourage any such remarks. Reporting to a supervisor, however, is not likely necessary.

A nurses assessment reveals that a client with COPD may be experiencing bronchospasm. What Test Bank - Brunner & Suddarth's Textbook of Medical-Surgical Nursing 14e (Hinkle 2017) 483 assessment finding would suggest that the patient is experiencing bronchospasm? A) Fine or coarse crackles on auscultation B) Wheezes or diminished breath sounds on auscultation C) Reduced respiratory rate or lethargy D) Slow, deliberate respirations

B Feedback: Wheezing and diminished breath sounds are consistent with bronchospasm. Crackles are usually attributable to other respiratory or cardiac pathologies. Bronchospasm usually results in rapid, inefficient breathing and agitation.

The nurse is performing an assessment on a patient who has been diagnosed with cancer of the larynx. Part of the nurses assessment addresses the patients general state of nutrition. Which laboratory values would be assessed when determining the nutritional status of the patient? Select all that apply. a) while blood cell count b) protein level c) albumin level d) platelet count e) glucose level

B, C, E The nurse also assesses the patients general state of nutrition, including height and weight and body mass index, and reviews laboratory values that assist in determining the patients nutritional status (albumin, protein, glucose, and electrolyte levels). The white blood cell count and the platelet count would not normally assist in determining the patients nutritional status.

The nurse is reviewing the electronic health record of a patient with a history of incontinence. The nurse reads that the physician assessed the patients deep tendon reflexes. What condition of the urinary/renal system does this assessment address? A) Renal calculi B) Bladder dysfunction C) Benign prostatic hyperplasia (BPH) D) Recurrent urinary tract infections (UTIs)

B Feedback: The deep tendon reflexes of the knee are examined for quality and symmetry. This is an important part of testing for neurologic causes of bladder dysfunction, because the sacral area, which innervates the lower extremities, is in the same peripheral nerve area responsible for urinary continence. Neurologic function does not directly influence the course of renal calculi, BPH or UTIs.

A nurse is reviewing the pathophysiology of cystic fibrosis (CF) in anticipation of a new admission. The nurse should identify what characteristic aspects of CF? A) Alveolar mucus plugging, infection, and eventual bronchiectasis B) Bronchial mucus plugging, inflammation, and eventual bronchiectasis C) Atelectasis, infection, and eventual COPD D) Bronchial mucus plugging, infection, and eventual COPD

B Feedback: The hallmark pathology of CF is bronchial mucus plugging, inflammation, and eventual bronchiectasis. Commonly, the bronchiectasis begins in the upper lobes and progresses to involve all lobes. Infection, atelectasis, and COPD are not hallmark pathologies of CF. 22. An older adult patient has been diagnosed with COPD. Wha

A nurse is creating a health promotion intervention focused on chronic obstructive pulmonary disease (COPD). What should the nurse identify as a complication of COPD? A) Lung cancer B) Cystic fibrosis C) Respiratory failure D) Hemothorax

C Feedback: Complications of COPD include respiratory failure, pneumothorax, atelectasis, pneumonia, and pulmonary hypertension (corpulmonale). Lung cancer, cystic fibrosis, and hemothorax are not common complications.

A patient who has recently undergone ESWL for the treatment of renal calculi has phoned the urology Test Bank - Brunner & Suddarth's Textbook of Medical-Surgical Nursing 14e (Hinkle 2017) 1050 unit where he was treated, telling the nurse that he has a temperature of 101.1F (38.4C). How should the nurse best respond to the patient? A) Remind the patient that renal calculi have a noninfectious etiology and that a fever is unrelated to their recurrence. B) Remind the patient that occasional febrile episodes are expected following ESWL. C) Tell the patient to report to the ED for further assessment. D) Tell the patient to monitor his temperature for the next 24 hours and then contact his urologists office.

C Feedback: Following ESWL, the development of a fever is abnormal and is suggestive of a UTI; prompt medical assessment and treatment are warranted. It would be inappropriate to delay further treatment.

Diagnostic testing of an adult patient reveals renal glycosuria. The nurse should recognize the need for Test Bank - Brunner & Suddarth's Textbook of Medical-Surgical Nursing 14e (Hinkle 2017) 1007 the patient to be assessed for what health problem? A) Diabetes insipidus B) Syndrome of inappropriate antidiuretic hormone secretion (SIADH) C) Diabetes mellitus D) Renal carcinoma

C Feedback: Renal glycosuria can occur on its own as a benign condition. It also occurs in poorly controlled diabetes, the most common condition that causes the blood glucose level to exceed the kidneys reabsorption capacity. Glycosuria is not associated with SIADH, diabetes insipidus, or renal carcinoma.

A patient has a glomerular filtration rate (GFR) of 43 mL/min/1.73 m2. Based on this GFR, the nurse interprets that the patients chronic kidney disease is at what stage? A) Stage 1 B) Stage 2 C) Stage 3 D) Stage 4

C Feedback: Stages of chronic renal failure are based on the GFR. Stage 3 is defined by a GFR in the range of 30 to 59 mL/min/1.73 m2. This is considered a moderate decrease in GFR.

Maintaining an aseptic environment in the OR is essential to patient safety and infection control. When moving around surgical areas, what distance must the nurse maintain from the sterile field? A) 2 feet B) 18 inches C) 1 foot D) 6 inches

C Feedback: Sterile areas must be kept in view during movement around the area. At least a 1-foot distance from the sterile field must be maintained to prevent inadvertent contamination.

The nurse is caring for a 46-year-old patient recently diagnosed with the early stages of lung cancer. The nurse is aware that the preferred method of treating patients with nonsmall cell tumors is what? A) Chemotherapy B) Radiation C) Surgical resection D) Bronchoscopic opening of the airway

C Feedback: Surgical resection is the preferred method of treating patients with localized nonsmall cell tumors with no evidence of metastatic spread and adequate cardiopulmonary function. The other listed treatment options may be considered, but surgery is preferred.

The nurse is caring for a patient at risk for atelectasis. The nurse implements a first-line measure to prevent atelectasis development in the patient. What is an example of a first-line measure to minimize atelectasis? A) Incentive spirometry Test Bank - Brunner & Suddarth's Textbook of Medical-Surgical Nursing 14e (Hinkle 2017) 460 B) Intermittent positive-pressure breathing (IPPB) C) Positive end-expiratory pressure (PEEP) D) Bronchoscopy

A Feedback: Strategies to prevent atelectasis, which include frequent turning, early ambulation, lung-volume expansion maneuvers (deep breathing exercises, incentive spirometry), and coughing, serve as the firstline measures to minimize or treat atelectasis by improving ventilation. In patients who do not respond to first-line measures or who cannot perform deep-breathing exercises, other treatments such as positive end-expiratory pressure (PEEP), continuous or intermittent positive-pressure breathing (IPPB), or bronchoscopy may be used.

The nurse is caring for a patient postoperative day 4 following a kidney transplant. When assessing for potential signs and symptoms of rejection, what assessment should the nurse prioritize? A) Assessment of the quantity of the patients urine output B) Assessment of the patients incision C) Assessment of the patients abdominal girth D) Assessment for flank or abdominal pain

A Feedback: After kidney transplantation, the nurse should perform all of the listed assessments. However, oliguria is considered to be more suggestive of rejection than changes to the patients abdomen or incision.

The nurse has identified the nursing diagnosis of risk for infection in a patient who undergoes peritoneal dialysis. What nursing action best addresses this risk? A) Maintain aseptic technique when administering dialysate. B) Wash the skin surrounding the catheter site with soap and water prior to each exchange. C) Add antibiotics to the dialysate as ordered. D) Administer prophylactic antibiotics by mouth or IV as ordered.

A Feedback: Aseptic technique is used to prevent peritonitis and other infectious complications of peritoneal dialysis. It is not necessary to cleanse the skin with soap and water prior to each exchange. Antibiotics may be added to dialysate to treat infection, but they are not used to prevent infection

The perioperative nurse is providing care for a patient who is recovering on the postsurgical unit following a transurethral prostate resection (TUPR). The patient is reluctant to ambulate, citing the need to recover in bed. For what complication is the patient most at risk? Test Bank - Brunner & Suddarth's Textbook of Medical-Surgical Nursing 14e (Hinkle 2017) 379 A) Atelectasis B) Anemia C) Dehydration D) Peripheral edema

A Feedback: Atelectasis occurs when the postoperative patient fails to move, cough, and breathe deeply. With good nursing care, this is an avoidable complication, but reduced mobility greatly increases the risk. Anemia occurs rarely and usually in situations where the patient loses a significant amount of blood or continues bleeding postoperatively. Fluid shifts postoperatively may result in dehydration and peripheral edema, but the patient is most at risk for atelectasis.

Renal failure can have prerenal, renal, or postrenal causes. A patient with acute kidney injury is being assessed to determine where, physiologically, the cause is. If the cause is found to be prerenal, which condition most likely caused it? A) Heart failure B) Glomerulonephritis C) Ureterolithiasis D) Aminoglycoside toxicity

A Feedback: By causing inadequate renal perfusion, heart failure can lead to prerenal failure. Glomerulonephritis and aminoglycoside toxicity are renal causes, and ureterolithiasis is a postrenal cause.

A nurse is caring for a patient who has been admitted with an exacerbation of chronic bronchiectasis. The nurse should expect to assess the patient for which of the following clinical manifestations? A) Copious sputum production B) Pain on inspiration C) Pigeon chest D) Dry cough

A Feedback: Clinical manifestations of bronchiectasis include hemoptysis, chronic cough, copious purulent sputum, and clubbing of the fingers. Because of the copious production of sputum, the cough is rarely dry. A pigeon chest is not associated with the disease and patients do not normally experience pain on inspiration.

A nurse is working with a patient who will undergo invasive urologic testing. The nurse has informed the patient that slight hematuria may occur after the testing is complete. The nurse should recommend what action to help resolve hematuria? A) Increased fluid intake following the test B) Use of an OTC diuretic after the test C) Gentle massage of the lower abdomen D) Activity limitation for the first 12 hours after the test

A Feedback: Drinking fluids can help to clear hematuria. Diuretics are not used for this purpose. Activity limitation and massage are unlikely to resolve this expected consequence of testing.

A nurse is planning the care of a client with bronchiectasis. What goal of care should the nurse prioritize? A) The patient will successfully mobilize pulmonary secretions. B) The patient will maintain an oxygen saturation level of 98%. C) The patients pulmonary blood pressure will decrease to within reference ranges. Test Bank - Brunner & Suddarth's Textbook of Medical-Surgical Nursing 14e (Hinkle 2017) 485 D) The patient will resume prediagnosis level of function within 72 hours.

A Feedback: Nursing management focuses on alleviating symptoms and helping patients clear pulmonary secretions. Pulmonary pressures are not a central focus in the care of the patient with bronchiectasis. Rapid resumption of prediagnosis function and oxygen saturation above 98% are unrealistic goals.

A patient has had a nasogastric tube in place for 6 days due to the development of paralytic ileus after Test Bank - Brunner & Suddarth's Textbook of Medical-Surgical Nursing 14e (Hinkle 2017) 449 surgery. In light of the prolonged presence of the nasogastric tube, the nurse should prioritize assessments related to what complication? A) Sinus infections B) Esophageal strictures C) Pharyngitis D) Laryngitis

A Feedback: Patients with nasotracheal and nasogastric tubes in place are at risk for development of sinus infections. Thus, accurate assessment of patients with these tubes is critical. Use of a nasogastric tube is not associated with the development of the other listed pathologies.

A patient who underwent a bowel resection to correct diverticula suffered irreparable nerve damage. Test Bank - Brunner & Suddarth's Textbook of Medical-Surgical Nursing 14e (Hinkle 2017) 369 During the case review, the team is determining if incorrect positioning may have contributed to the patients nerve damage. What surgical position places the patient at highest risk for nerve damage? A) Trendelenburg B) Prone C) Dorsal recumbent D) Lithotomy

A Feedback: Shoulder braces must be well padded to prevent irreparable nerve injury, especially when the Trendelenburg position is necessary. The other listed positions are less likely to cause nerve injury.

A nurse has been asked to give a workshop on COPD for a local community group. The nurse emphasizes the importance of smoking cessation because smoking has what pathophysiologic effect? A) Increases the amount of mucus production B) Destabilizes hemoglobin C) Shrinks the alveoli in the lungs D) Collapses the alveoli in the lungs

A Feedback: Smoking irritates the goblet cells and mucous glands, causing an increased accumulation of mucus, which, in turn, produces more irritation, infection, and damage to the lung.

A nurse is working with a child who is undergoing a diagnostic workup for suspected asthma. What are the signs and symptoms that are consistent with a diagnosis of asthma? Select all that apply. A) Chest tightness B) Crackles C) Bradypnea D) Wheezing E) Cough

A, D, E Feedback: Asthma is a chronic inflammatory disease of the airways that causes airway hyperresponsiveness, mucosal edema, and mucus production. This inflammation ultimately leads to recurrent episodes of asthma symptoms: cough, chest tightness, wheezing, and dyspnea. Crackles and bradypnea are not typical symptoms of asthma.

The PACU nurse is caring for a 45-year-old male patient who had a left lobectomy. The nurse is assessing the patient frequently for airway patency and cardiovascular status. The nurse should know that the most common cardiovascular complications seen in the PACU include what? Select all that apply. A) Hypotension B) Hypervolemia C) Heart murmurs D) Dysrhythmias E) Hypertension

A, D, E Feedback: The primary cardiovascular complications seen in the PACU include hypotension and shock, hemorrhage, hypertension, and dysrhythmias. Heart murmurs are not adverse reactions to surgery. Hypervolemia is not a common cardiovascular complication seen in the PACU, though fluid balance must be vigilantly monitored.

The surgical nurse is preparing to send a patient from the presurgical area to the OR and is reviewing the patients informed consent form. What are the criteria for legally valid informed consent? Select all that apply. A) Consent must be freely given. B) Consent must be notarized. C) Consent must be signed on the day of surgery. Test Bank - Brunner & Suddarth's Textbook of Medical-Surgical Nursing 14e (Hinkle 2017) 353 D) Consent must be obtained by a physician. E) Signature must be witnessed by a professional staff member.

A, D, E Feedback: Valid consent must be freely given, without coercion. Consent must be obtained by a physician and the patients signature must be witnessed by a professional staff member. It does not need to be signed on the same day as the surgery and it does not need to be notarized.

A nurse on the renal unit is caring for a patient who will soon begin peritoneal dialysis. The family of the patient asks for education about the peritoneal dialysis catheter that has been placed in the patients peritoneum. The nurse explains the three sections of the catheter and talks about the two cuffs on the dialysis catheter. What would the nurse explain about the cuffs? Select all that apply. A) The cuffs are made of Dacron polyester. B) The cuffs stabilize the catheter. C) The cuffs prevent the dialysate from leaking. D) The cuffs provide a barrier against microorganisms. E) The cuffs absorb dialysate

A, B, C, D Feedback: Most of these catheters have two cuffs, which are made of Dacron polyester. The cuffs stabilize the catheter, limit movement, prevent leaks, and provide a barrier against microorganisms. They do not absorb dialysate.

The staff educator is giving a class for a group of nurses new to the renal unit. The educator is discussing renal biopsies. In what patient would the educator tell the new nurses that renal biopsies are contraindicated? A) A 64-year-old patient with chronic glomerulonephritis B) A 57-year-old patient with proteinuria C) A 42-year-old patient with morbid obesity D) A 16-year-old patient with signs of kidney transplant rejection

C Feedback: There are several contraindications to a kidney biopsy, including bleeding tendencies, uncontrolled hypertension, a solitary kidney, and morbid obesity. Indications for a renal biopsy include unexplained acute renal failure, persistent proteinuria or hematuria, transplant rejection, and glomerulopathies.

A gerontologic nurse is teaching a group of medical nurses about the high incidence and mortality of pneumonia in older adults. What is a contributing factor to this that the nurse should describe? A) Older adults have less compliant lung tissue than younger adults. B) Older adults are not normally candidates for pneumococcal vaccination. C) Older adults often lack the classic signs and symptoms of pneumonia. D) Older adults often cannot tolerate the most common antibiotics used to treat pneumonia.

C Test Bank - Brunner & Suddarth's Textbook of Medical-Surgical Nursing 14e (Hinkle 2017) 469 Feedback: The diagnosis of pneumonia may be missed because the classic symptoms of cough, chest pain, sputum production, and fever may be absent or masked in older adult patients. Mortality from pneumonia in the elderly is not a result of limited antibiotic options or lower lung compliance. The pneumococcal vaccine is appropriate for older adults.

As a perioperative nurse, you know that the National Patient Safety Goals have the potential to improve patient outcomes in a wide variety of health care settings. Which of these Goals has the most direct relevance to the OR? A) Improve safety related to medication use B) Reduce the risk of patient harm resulting from falls C) Reduce the incidence of health care-associated infections D) Reduce the risk of fires

D Feedback: The National Patient Safety Goals all pertain to the perioperative areas, but the one with the most direct relevance to the OR is the reduction of the risk of surgical fires.

A patients most recent laboratory findings indicate a glomerular filtration rate (GFR) of 58 mL/min. The nurse should recognize what implication of this diagnostic finding? A) The patient is likely to have a decreased level of blood urea nitrogen (BUN). B) The patient is at risk for hypokalemia. Test Bank - Brunner & Suddarth's Textbook of Medical-Surgical Nursing 14e (Hinkle 2017) 1008 C) The patient is likely to have irregular voiding patterns. D) The patient is likely to have increased serum creatinine levels.

D Feedback: The adult GFR can vary from a normal of approximately 125 mL/min (1.67 to 2.0 mL/sec) to a high of 200 mL/min. A low GFR is associated with increased levels of BUN, creatinine, and potassium.

An adult patient has been hospitalized with pyelonephritis. The nurses review of the patients intake and output records reveals that the patient has been consuming between 3 L and 3.5 L of oral fluid each day since admission. How should the nurse best respond to this finding? A) Supplement the patients fluid intake with a high-calorie diet. B) Emphasize the need to limit intake to 2 L of fluid daily. C) Obtain an order for a high-sodium diet to prevent dilutional hyponatremia. D) Encourage the patient to continue this pattern of fluid intake.

D Feedback: Unless contraindicated, 3 to 4 L of fluids per day is encouraged to dilute the urine, decrease burning on urination, and prevent dehydration. No need to supplement this fluid intake with additional calories or sodium.

23. The critical care nurse and the other members of the care team are assessing the patient to see if he is ready to be weaned from the ventilator. What are the most important predictors of successful weaning that the nurse should identify? A) Stable vital signs and ABGs B) Pulse oximetry above 80% and stable vital signs C) Stable nutritional status and ABGs D) Normal orientation and level of consciousness

Ans: A Feedback: Among many other predictors, stable vital signs and ABGs are important predictors of successful weaning. Pulse oximetry must greatly exceed 80%. Nutritional status is important, but vital signs and ABGs are even more significant. Patients who are weaned may or may not have full level of consciousness.

7. A patient is exhibiting signs of a pneumothorax following tracheostomy. The surgeon inserts a chest tube into the anterior chest wall. What should the nurse tell the family is the primary purpose of this chest tube? A) To remove air from the pleural space B) To drain copious sputum secretions C) To monitor bleeding around the lungs D) To assist with mechanical ventilation

Ans: A Feedback: Chest tubes and closed drainage systems are used to re-expand the lung involved and to remove excess air, fluid, and blood. The primary purpose of a chest tube is not to drain sputum secretions, monitor bleeding, or assist with mechanical ventilation.

10. The ED nurse is assessing a young gymnast who fell from a balance beam. The gymnast presents with a clear fluid leaking from her nose. What should the ED nurse suspect? A) Fracture of the cribriform plate B) Rupture of an ethmoid sinus C) Abrasion of the soft tissue D) Fracture of the nasal septum

Ans: A Feedback: Clear fluid from either nostril suggests a fracture of the cribriform plate with leakage of cerebrospinal fluid. The symptoms are not indicative of an abrasion of the soft tissue or rupture of a sinus. Clear fluid leakage from the nose would not be indicative of a fracture of the nasal septum.

8. The nurse is caring for a patient who needs education on his medication therapy for allergic rhinitis. The patient is to take cromolyn (Nasalcrom) daily. In providing education for this patient, how should the nurse describe the action of the medication? A) It inhibits the release of histamine and other chemicals. B) It inhibits the action of proton pumps. C) It inhibits the action of the sodium-potassium pump in the nasal epithelium. D) It causes bronchodilation and relaxes smooth muscle in the bronchi.

Ans: A Feedback: Cromolyn (Nasalcrom) inhibits the release of histamine and other chemicals. It is prescribed to treat allergic rhinitis. Beta-adrenergic agents lead to bronchodilation and stimulate beta-2adrenergic receptors in the smooth muscle of the bronchi and bronchioles. It does not affect proton pump action or the sodium-potassium pump in the nasal cells.

14. The medical nurse who works on a pulmonology unit is aware that several respiratory conditions can affect lung tissue compliance. The presence of what condition would lead to an increase in lung compliance? A) Emphysema B) Pulmonary fibrosis C) Pleural effusion D) Acute respiratory distress syndrome (ARDS)

Ans: A Feedback: High or increased compliance occurs if the lungs have lost their elasticity and the thorax is overdistended, in conditions such as emphysema. Conditions associated with decreased compliance include pneumothorax,hemothorax, pleural effusion, pulmonary edema, atelectasis, pulmonary fibrosis, and ARDS

21. A critical care nurse is caring for a client with an endotracheal tube who is on a ventilator. The nurse knows that meticulous airway management of this patient is necessary. What is the main rationale for this? A) Maintaining a patent airway B) Preventing the need for suctioning C) Maintaining the sterility of the patients airway D) Increasing the patients lung compliance

Ans: A Feedback: Maintaining a patent (open) airway is achieved through meticulous airway management, whether in an emergency situation such as airway obstruction or in long-term management, as in caring for a patient with an endotracheal or a tracheostomy tube. The other answers are incorrect.

23. The nurse is performing a respiratory assessment of an adult patient and is attempting to distinguish between vesicular, bronchovesicular, and bronchial (tubular) breath sounds. The nurse should distinguish between these normal breath sounds on what basis? A) Their location over a specific area of the lung B) The volume of the sounds C) Whether they are heard on inspiration or expiration D) Whether or not they are continuous breath sounds

Ans: A Feedback: Normal breath sounds are distinguished by their location over a specific area of the lung; they are identified as vesicular, bronchovesicular, and bronchial (tubular) breath sounds. Normal breath sounds are heard on both inspiration and expiration, and are continuous. They are not distinguished solely on the basis of volume.

26. The nurse is caring for a patient with a lower respiratory tract infection. When planning a focused respiratory assessment, the nurse should know that this type of infection most often causes what? A) Impaired gas exchange B) Collapsed bronchial structures C) Necrosis of the alveoli D) Closed bronchial tree

Ans: A Feedback: The lower respiratory tract consists of the lungs, which contain the bronchial and alveolar structures needed for gas exchange. A lower respiratory tract infection does not collapse bronchial structures or Test Bank - Brunner & Suddarth's Textbook of Medical-Surgical Nursing 14e (Hinkle 2017) 409 close the bronchial tree. An infection does not cause necrosis of lung tissues.

6. The home care nurse is assessing the home environment of a patient who will be discharged from the hospital shortly after his laryngectomy. The nurse should inform the patient that he may need to arrange for the installation of which system in his home? A) A humidification system B) An air conditioning system C) A water purification system D) A radiant heating system

Ans: A Feedback: The nurse stresses the importance of humidification at home and instructs the family to obtain and set up a humidification system before the patient returns home. Air-conditioning may be too cool and too drying for the patient. A water purification system or a radiant heating system is not necessary.

16. The nurse is caring for a patient who is ready to be weaned from the ventilator. In preparing to assist in the collaborative process of weaning the patient from a ventilator, the nurse is aware that the weaning of the patient will progress in what order? A) Removal from the ventilator, tube, and then oxygen B) Removal from oxygen, ventilator, and then tube C) Removal of the tube, oxygen, and then ventilator D) Removal from oxygen, tube, and then ventilator

Ans: A Feedback: The process of withdrawing the patient from dependence on the ventilator takes place in three stages: the patient is gradually removed from the ventilator, then from the tube, and, finally, oxygen.

32. A patient has been discharged home after thoracic surgery. The home care nurse performs the initial visit and finds the patient discouraged and saddened. The client states, I am recovering so slowly. I really thought I would be better by now. What nursing action should the nurse prioritize? A) Provide emotional support to the patient and family. B) Schedule a visit to the patients primary physician within 24 hours. C) Notify the physician that the patient needs a referral to a psychiatrist. D) Place a referral for a social worker to visit the patient.

Ans: A Feedback: The recovery process may take longer than the patient had expected, and providing support to the patient is an important task for the home care nurse. It is not necessary, based on this scenario, to schedule a visit with the physician within 24 hours, or to get a referral to a psychiatrist or a social worker.

The nurse is caring for a patient who is scheduled to have a needle biopsy of the pleura. The patient has had a consultation with the anesthesiologist and a conduction block will be used. Which local conduction block can be used to block the nerves leading to the chest? A) Transsacral block B) Brachial plexus block C) Peudental block D) Paravertebral block

D Feedback: Examples of common local conduction blocks include paravertebral anesthesia, which produces anesthesia of the nerves supplying the chest, abdominal wall, and extremities; brachial plexus block, which produces anesthesia of the arm; and transsacral (caudal) block, which produces anesthesia of the perineum and, occasionally, the lower abdomen. A peudental block was used in obstetrics before the almost-routine use of epidural anesthesia.

35. A team of community health nurses are planning to draft a proposal for a program that will increase the communitys alignment with the principles contained in the Healthy People 2020report. Which of the following activities would best demonstrate the priorities identified in this report? A) Addressing determinants of health such as clean environments and safety in the community B) Lobbying for increased funding to the county hospital where many residents receive primary care C) Collaborating with health professionals in neighboring communities to pool resources and increase efficiencies D) Creating clinical placements where nursing students and members of other health disciplines can gain experience in a community setting

Ans: A Feedback: Healthy People 2020 addresses social determinants of health such as safety and the state of the environment. This report does not specifically address matters such as hospital funding, nursing education, or resource allocation.

46. The nurse caring for a patient who is two days post hip replacement notifies the physician that the patients incision is red around the edges, warm to the touch, and seeping a white liquid with a foul odor. What type of problem is the nurse dealing with? A) Collaborative problem B) Nursing problem C) Medical problem Test Bank - Brunner & Suddarth's Textbook of Medical-Surgical Nursing 14e (Hinkle 2017) 62 D) Administrative problem

Ans: A Feedback: In addition to nursing diagnoses and their related nursing interventions, nursing practice involves certain situations and interventions that do not fall within the definition of nursing diagnoses. These activities pertain to potential problems or complications that are medical in origin and require collaborative interventions with the physician and other members of the health care team. The other answers are incorrect because the signs and symptoms of infection are a medical complication that requires interventions by the nurse.

22. A community health nurse has witnessed significant shifts in patterns of disease over the course of a four-decade career. Which of the following focuses most clearly demonstrates the changing pattern of disease in the United States? A) Type 1 diabetes management B) Treatment of community-acquired pneumonia C) Rehabilitation from traumatic brain injuries D) Management of acute Staphylococcus aureus infections

Ans: A Feedback: Test Bank - Brunner & Suddarth's Textbook of Medical-Surgical Nursing 14e (Hinkle 2017) 13 Management of chronic diseases such as diabetes is a priority focus of the current health care environment. This supersedes the treatment of acute infections and rehabilitation needs.

29. The nurse has explained to the patient that after his thoracotomy, it will be important to adhere to a coughing schedule. The patient is concerned about being in too much pain to be able to cough. What would be an appropriate nursing intervention for this client? A) Teach him postural drainage. B) Teach him how to perform huffing. C) Teach him to use a mini-nebulizer. D) Teach him how to use a metered dose inhaler.

Ans: B Feedback: The technique of huffing may be helpful for the patient with diminished expiratory flow rates or for the patient who refuses to cough because of severe pain. Huffing is the expulsion of air through an open glottis. Inhalers, nebulizers, and postural drainage are not substitutes for performing coughing exercises.

37. A medical patient rings her call bell and expresses alarm to the nurse, stating, Ive just coughed up this blood. That cant be good, can it? How can the nurse best determine whether the source of the blood was the patients lungs? A) Obtain a sample and test the pH of the blood, if possible. B) Try to see if the blood is frothy or mixed with mucus. C) Perform oral suctioning to see if blood is obtained. D) Swab the back of the patients throat to see if blood is present.

Ans: B Feedback: Though not definitive, blood from the lung is usually bright red, frothy, and mixed with sputum. Testing the pH of nonarterial blood samples is not common practice and would not provide important data. Similarly, oral suctioning and swabbing the patients mouth would not reveal the source.

24. The role of the certified nurse practitioner (CNP) has become a dominant role for nurses in all levels of health care. Which of the following activities are considered integral to the CNP role? Select all that apply. A) Educating patients and family members B) Coordinating care with other disciplines C) Using direct provision of interventions D) Educating registered nurses and practical nurses E) Coordinating payment plans for patients

Ans: A, B, C Feedback: This role is a dominant one for nurses in primary, secondary, and tertiary health care settings and in home care and community nursing. Nurses help patients meet their needs by using direct intervention, by teaching patients and family members to perform care, and by coordinating and collaborating with other disciplines to provide needed services. The other answers are incorrect because NPs do not commonly perform education of nurses and they do not focus on matters related to payment. Test Bank - Brunner & Suddarth's Textbook of Medical-Surgical Nursing 14e (Hinkle 2017) 14

36. A nurse educator is reviewing the implications of the oxyhemoglobin dissociation curve with regard to the case of a current patient. The patient currently has normal hemoglobin levels, but significantly decreased SaO2 and PaO2 levels. What is an implication of this physiological state? A) The patients tissue demands may be met, but she will be unable to respond to physiological stressors. Test Bank - Brunner & Suddarth's Textbook of Medical-Surgical Nursing 14e (Hinkle 2017) 413 B) The patients short-term oxygen needs will be met, but she will be unable to expel sufficient CO2 . C) The patient will experience tissue hypoxia with no sensation of shortness of breath or labored breathing. D) The patient will experience respiratory alkalosis with no ability to compensate.

Ans: A Feedback: With a normal hemoglobin level of 15 mg/dL and a PaO2 level of 40 mm Hg (SaO2 75%), there is adequate oxygen available for the tissues, but no reserve for physiological stresses that increase tissue oxygen demand. If a serious incident occurs (e.g., bronchospasm, aspiration, hypotension, or cardiac dysrhythmias) that reduces the intake of oxygen from the lungs, tissue hypoxia results.

29. The nurse is caring for a patient who has a pleural effusion and who underwent a thoracoscopic procedure earlier in the morning. The nurse should prioritize assessment for which of the following? A) Sputum production Test Bank - Brunner & Suddarth's Textbook of Medical-Surgical Nursing 14e (Hinkle 2017) 410 B) Shortness of breath C) Throat discomfort D) Epistaxis

Ans: B Feedback: Follow-up care in the health care facility and at home involves monitoring the patient for shortness of breath (which might indicate a pneumothorax). All of the listed options are relevant assessment findings, but shortness of breath is the most serious complication.

16. A nurse practitioner has provided care for three different patients with chronic pharyngitis over the past several months. Which patients are at greatest risk for developing chronic pharyngitis? A) Patients who are habitual users of alcohol and tobacco B) Patients who are habitual users of caffeine and other stimulants C) Patients who eat a diet high in spicy foods D) Patients who have gastrointestinal reflux disease (GERD)

Ans: A Test Bank - Brunner & Suddarth's Textbook of Medical-Surgical Nursing 14e (Hinkle 2017) 442 Feedback: Chronic pharyngitis is common in adults who live and work in dusty surroundings, use the voice to excess, suffer from chronic chough, and habitually use alcohol and tobacco. Caffeine and spicy foods have not been linked to chronic pharyngitis. GERD is not a noted risk factor.

1. The nurse is caring for a patient with chronic obstructive pulmonary disease (COPD). The patient has been receiving high-flow oxygen therapy for an extended time. What symptoms should the nurse anticipate if the patient were experiencing oxygen toxicity? A) Bradycardia and frontal headache B) Dyspnea and substernal pain C) Peripheral cyanosis and restlessness D) Hypotension and tachycardia

Ans: B Feedback: Oxygen toxicity can occur when patients receive too high a concentration of oxygen for an extended period. Symptoms of oxygen toxicity include dyspnea, substernal pain, restlessness, fatigue, and progressive respiratory difficulty. Bradycardia, frontal headache, cyanosis, hypotension, and tachycardia are not symptoms of oxygen toxicity.

47. While developing the plan of care for a new patient on the unit, the nurse must identify expected outcomes that are appropriate for the new patient. What resource should the nurse prioritize for identifying these appropriate outcomes? A) Community Specific Outcomes Classification (CSO) B) Nursing-Sensitive Outcomes Classification (NOC) C) State Specific Nursing Outcomes Classification (SSNOC) D) Department of Health and Human Services Outcomes Classification (DHHSOC)

Ans: B Feedback: Resources for identifying appropriate expected outcomes include the NOC and standard outcome criteria established by health care agencies for people with specific health problems. The other options are incorrect because they do not exist.

25. A patient is scheduled to have excess pleural fluid aspirated with a needle in order to relieve her dyspnea. The patient inquires about the normal function of pleural fluid. What should the nurse describe? A) It allows for full expansion of the lungs within the thoracic cavity. B) It prevents the lungs from collapsing within the thoracic cavity. C) It limits lung expansion within the thoracic cavity. D) It lubricates the movement of the thorax and lungs.

Ans: D Feedback: The visceral pleura cover the lungs; the parietal pleura line the thorax. The visceral and parietal pleura and the small amount of pleural fluid between these two membranes serve to lubricate the thorax and lungs and permit smooth motion of the lungs within the thoracic cavity with each breath. The pleura do not allow full expansion of the lungs, prevent the lungs from collapsing, or limit lung expansion within the thoracic cavity.

17. A patient has a diagnosis of multiple sclerosis. The nurse is aware that neuromuscular disorders such as multiple sclerosis may lead to a decreased vital capacity. What does vital capacity measure? A) The volume of air inhaled and exhaled with each breath B) The volume of air in the lungs after a maximal inspiration C) The maximal volume of air inhaled after normal expiration D) The maximal volume of air exhaled from the point of maximal inspiration

Ans: D Feedback: Vital capacity is measured by having the patient take in a maximal breath and exhale fully through a spirometer. Vital lung capacity is the maximal volume of air exhaled from the point of maximal inspiration, and neuromuscular disorders such as multiple sclerosis may lead to a decreased vital capacity. Tidal volume is defined as the volume of air inhaled and exhaled with each breath. The volume of air in the lungs after a maximal inspiration is the total lung capacity. Inspiratory capacity is the maximal volume of air inhaled after normal expiration.

37. A group of nursing students are learning about recent changes in the pattern of disease in the United States. Which of the following statements best describes these current changes? A) Infectious diseases continue to decrease in incidence and prevalence. B) Chronic illnesses are becoming increasingly resistant to treatment. C) Most acute, infectious diseases have been eradicated. D) Most, but not all, communicable diseases are declining.

Ans: D Feedback: Although some infectious diseases have been controlled or eradicated, others are on the rise. Antibiotic resistance is a more serious problem in acute, not chronic, illnesses.

30. A case manager has been hired at a rural hospital that has a combined medical-surgical unit. When defining this new role, which of the following outcomes should be prioritized by the hospitals leadership? A) Decreased need for physician services B) Improved patient and family education C) Increased adherence to the principles of EBP D) Increased coordination of health services

Ans: D Feedback: Case management is a system of coordinating health care services to ensure cost-effectiveness, accountability, and quality care. The case manager coordinates the care of a caseload of patients through facilitating communication between nurses, other health care personnel who provide care, and insurance companies. Reducing the need for physician services is not a central goal. Education and EBP are consistent with case management, but they are not central to this particular role.

31. The nursing instructor is explaining critical thinking to a class of first-semester nursing students. When promoting critical thinking skills in these students, the instructor should encourage them to do which of the following actions? Test Bank - Brunner & Suddarth's Textbook of Medical-Surgical Nursing 14e (Hinkle 2017) 55 A) Disregard input from people who do not have to make the particular decision. B) Set aside all prejudices and personal experiences when making decisions. C) Weigh each of the potential negative outcomes in a situation. D) Examine and analyze all available information.

Ans: D Feedback: Critical thinking involves reasoning and purposeful, systematic, reflective, rational, outcome-directed thinking based on a body of knowledge, as well as examination and analysis of all available information and ideas. A full disregard of ones own experiences is not possible. Critical thinking does not denote a focus on potential negative outcomes. Input from others is a valuable resource that should not be ignored.

36. The nurse is providing care for a patient with chronic obstructive pulmonary disease (COPD). The nurses most recent assessment reveals an SaO2 of 89%. The nurse is aware that part of critical thinking is determining the significance of data that have been gathered. What characteristic of critical thinking is used in determining the best response to this assessment finding? A) Extrapolation B) Inference C) Characterization D) Interpretation

Ans: D Feedback: Nurses use interpretation to determine the significance of data that are gathered. This specific process is not described as extrapolation, inference, or characterization.

20. The physician has recommended an amniocentesis for an 18-year-old primiparous woman. The patient is 34 weeks gestation and does not want this procedure. The physician is insistent the patient have the procedure. The physician arranges for the amniocentesis to be performed. The nurse should recognize that the physician is in violation of what ethical principle? A) Veracity B) Beneficence C) Nonmaleficence Test Bank - Brunner & Suddarth's Textbook of Medical-Surgical Nursing 14e (Hinkle 2017) 50 D) Autonomy

Ans: D Feedback: The principle of autonomy specifies that individuals have the ability to make a choice free from external constraints. The physicians actions in this case violate this principle. This action may or may not violate the principle of beneficence. Veracity centers on truth-telling and nonmaleficence is avoiding the infliction of harm.

48. The nurse has just taken report on a newly admitted patient who is a 15year-old girl who is a recent immigrant to the United States. When planning interventions for this patient, the nurse knows the interventions must be which of the following? Select all that apply. A) Appropriate to the nurses preferences B) Appropriate to the patients age C) Ethical D) Appropriate to the patients culture E) Applicable to others with the same diagnosis

Ans: B, C, D Feedback: Planned interventions should be ethical and appropriate to the patients culture, age, and gender. Planned interventions do not have to be in alignment with the nurses preferences nor do they have to be shared by everyone with the same diagnosis.

11. While assessing the patient, the nurse observes constant bubbling in the water-seal chamber of the patients closed chest-drainage system. What should the nurse conclude? A) The system is functioning normally. B) The patient has a pneumothorax. C) The system has an air leak. D) The chest tube is obstructed.

Ans: C Feedback: Constant bubbling in the chamber often indicates an air leak and requires immediate assessment and intervention. The patient with a pneumothorax will have intermittent bubbling in the water-seal chamber. If the tube is obstructed, the nurse should notice that the fluid has stopped fluctuating in the water-seal chamber.

26. A nursing student is discussing a patient with viral pharyngitis with the preceptor at the walk-in clinic. What should the preceptor tell the student about nursing care for patients with viral pharyngitis? A) Teaching focuses on safe and effective use of antibiotics. B) The patient should be preliminarily screened for surgery. C) Symptom management is the main focus of medical and nursing care. D) The focus of care is resting the voice to prevent chronic hoarseness.

Ans: C Feedback: Nursing care for patients with viral pharyngitis focuses on symptomatic management. Antibiotics are not prescribed for viral etiologies. Surgery is not indicated in the treatment of viral pharyngitis. Chronic hoarseness is not a common sequela of viral pharyngitis, so teaching ways to prevent it would be of no use in this instance.

25. The nurse is preparing to discharge a patient after thoracotomy. The patient is going home on oxygen therapy and requires wound care. As a result, he will receive home care nursing. What should the nurse include in discharge teaching for this patient? A) Safe technique for self-suctioning of secretions B) Technique for performing postural drainage C) Correct and safe use of oxygen therapy equipment D) How to provide safe and effective tracheostomy care

Ans: C Feedback: Respiratory care and other treatment modalities (oxygen, incentive spirometry, chest physiotherapy [CPT], and oral, inhaled, or IV medications) may be continued at home. Therefore, the nurse needs to instruct the patient and family in their correct and safe use. The scenario does not indicate the patient needs help with suctioning, postural drainage, or tracheostomy care.

34. The ED nurse is assessing the respiratory function of a teenage girl who presented with acute shortness of breath. Auscultation reveals continuous wheezes during inspiration and expiration. This finding is most suggestive what? A) Pleurisy B) Emphysema C) Asthma D) Pneumonia

Ans: C Feedback: Sibilant wheezes are commonly associated with asthma. They do not normally accompany pleurisy, emphysema, or pneumonia.

19. A patient with a severe exacerbation of COPD requires reliable and precise oxygen delivery. Which mask will the nurse expect the physician to order? A) Non-rebreather air mask B) Tracheostomy collar C) Venturi mask D) Face tent

Ans: C Feedback: The Venturi mask provides the most accurate method of oxygen delivery. Other methods of oxygen delivery include the aerosol mask, tracheostomy collar, and face tents, but these do not match the precision of a Venturi mask.

31. The nurse is assessing the respiratory status of a patient who is experiencing an exacerbation of her emphysema symptoms. When preparing to auscultate, what breath sounds should the nurse anticipate? A) Absence of breath sounds B) Wheezing with discontinuous breath sounds C) Faint breath sounds with prolonged expiration D) Faint breath sounds with fine crackles

Ans: C Feedback: The breath sounds of the patient with emphysema are faint or often completely inaudible. When they are heard, the expiratory phase is prolonged.

14. The nurse is explaining the safe and effective administration of nasal spray to a patient with seasonal allergies. What information is most important to include in this teaching? A) Finish the bottle of nasal spray to clear the infection effectively. B) Nasal spray can only be shared between immediate family members. C) Nasal spray should be administered in a prone position. Test Bank - Brunner & Suddarth's Textbook of Medical-Surgical Nursing 14e (Hinkle 2017) 441 D) Overuse of nasal spray may cause rebound congestion.

Ans: D Feedback: The use of topical decongestants is controversial because of the potential for a rebound effect. The patient should hold his or her head back for maximal distribution of the spray. Only the patient should use the bottle.

28. The decision has been made to discharge a ventilator-dependent patient home. The nurse is developing a teaching plan for this patient and his family. What would be most important to include in this teaching plan? A) Administration of inhaled corticosteroids B) Assessment of neurologic status C) Turning and coughing D) Signs of pulmonary infection

Ans: D Feedback: The nurse teaches the patient and family about the ventilator, suctioning, tracheostomy care, signs of pulmonary infection, cuff inflation and deflation, and assessment of vital signs. Neurologic assessment and turning and coughing are less important than signs and symptoms of infection. Inhaled corticosteroids may or may not be prescribed.

A nurse is assessing a patient who is suspected of having bronchiectasis. The nurse should consider which of the following potential causes? Select all that apply. A) Pulmonary hypertension B) Airway obstruction C) Pulmonary infections D) Genetic disorders E) Atelectasis

B, C, D Feedback: Bronchiectasis is a chronic, irreversible dilation of the bronchi and bronchioles. Under the new definition of COPD, it is considered a disease process separate from COPD. Bronchiectasis may be caused by a variety of conditions, including airway obstruction, diffuse airway injury, pulmonary infections and obstruction of the bronchus or complications of long-term pulmonary infections, or genetic disorders such as cystic fibrosis. Bronchiectasis is not caused by pulmonary hypertension or atelectasis.

Dipstick testing of an older adult patients urine indicates the presence of protein. Which of the following statements is true of this assessment finding? Select all that appl A) This finding needs to be considered in light of other forms of testing. B) This finding is a risk factor for urinary incontinence. C) This finding is likely the result of an age-related physiologic change. D) This result confirms that the patient has diabetes. y.

B, C, D Feedback: A dipstick examination, which can detect from 30 to 1000 mg/dL of protein, should be used as a screening test only, because urine concentration, pH, hematuria, and radiocontrast materials all affect the results. Proteinuria is not diagnostic of diabetes and it is neither an age-related change nor a risk factor for incontinence.

The nurse is caring for a postoperative patient who needs daily dressing changes. The patient is 3 days postoperative and is scheduled for discharge the next day. Until now, the patient has refused to learn how to change her dressing. What would indicate to the nurse the patients possible readiness to learn how to change her dressing? Select all that apply. A) The patient wants you to teach a family member to do dressing changes. B) The patient expresses interest in the dressing change. C) The patient is willing to look at the incision during a dressing change. Test Bank - Brunner & Suddarth's Textbook of Medical-Surgical Nursing 14e (Hinkle 2017) 394 D) The patient expresses dislike of the surgical wound. E) The patient assists in opening the packages of dressing material for the nurse.

B, C, E Feedback: While changing the dressing, the nurse has an opportunity to teach the patient how to care for the incision and change the dressings at home. The nurse observes for indicators of the patients readiness to learn, such as looking at the incision, expressing interest, or assisting in the dressing change. Expressing dislike and wanting to delegate to a family member do not suggest readiness to learn.

The nurse is caring for a patient with acute glomerular inflammation. When assessing for the characteristic signs and symptoms of this health problem, the nurse should include which assessments? Select all that apply. A) Percuss for pain in the right lower abdominal quadrant. B) Assess for the presence of peripheral edema. C) Auscultate the patients apical heart rate for dysrhythmias. D) Assess the patients BP. E) Assess the patients orientation and judgment.

B, D Feedback: Most patients with acute glomerular inflammation have some degree of edema and hypertension. Dysrhythmias, RLQ pain, and changes in mental status are not among the most common manifestations of acute glomerular inflammation.

An 84-year-old woman diagnosed with cancer is admitted to the oncology unit for surgical treatment. The patient has been on chemotherapeutic agents to decrease the tumor size prior to the planned surgery. The nurse caring for the patient is aware that what precipitating factors in this patient may contribute to AKI? Select all that apply. A) Anxiety B) Low BMI C) Age-related physiologic changes D) Chronic systemic disease E) NPO status

C, D Feedback: Changes in kidney function with normal aging increase the susceptibility of elderly patients to kidney dysfunction and renal failure. In addition, the presence of chronic, systemic diseases increases the risk of AKI. Low BMI and anxiety are not risk factors for acute renal disease. NPO status is not a risk, provided adequate parenteral hydration is administered.

28. The nurse is caring for a patient with lung metastases who just underwent a mediastinotomy. What should be the focus of the nurses postprocedure care? A) Assisting with pulmonary function testing (PFT) B) Maintaining the patients chest tube C) Administering oral suction as needed D) Performing chest physiotherapy

Feedback: Chest tube drainage is required after mediastinotomy. PFT, chest physiotherapy, and oral suctioning would all be contraindicated because of the patients unstable health status.

A patient presents to the ED stating she was in a boating accident about 3 hours ago. Now the patient has complaints of headache, fatigue, and the feeling that he just cant breathe enough. The nurse notes that the patient is restless and tachycardic with an elevated blood pressure. This patient may be in the early stages of what respiratory problem? A) Pneumoconiosis B) Pleural effusion C) Acute respiratory failure D) Pneumonia

Feedback: Early signs of acute respiratory failure are those associated with impaired oxygenation and may include restlessness, fatigue, headache, dyspnea, air hunger, tachycardia, and increased blood pressure. As the Test Bank - Brunner & Suddarth's Textbook of Medical-Surgical Nursing 14e (Hinkle 2017) 464 hypoxemia progresses, more obvious signs may be present, including confusion, lethargy, tachycardia, tachypnea, central cyanosis, diaphoresis, and, finally, respiratory arrest. Pneumonia is infectious and would not result from trauma. Pneumoconiosis results from exposure to occupational toxins. A pleural effusion does not cause this constellation of symptoms. 25. The nurse is caring for a 46-year-old patient recently diagnosed with the early stag

4. The ED nurse is assessing a patient complaining of dyspnea. The nurse auscultates the patients chest and hears wheezing throughout the lung fields. What might this indicate? A) The patient has a narrowed airway. B) The patient has pneumonia. C) The patient needs physiotherapy. D) The patient has a hemothorax.

Feedback: Wheezing is a high-pitched, musical sound that is often the major finding in a patient with bronchoconstriction or airway narrowing. Wheezing is not normally indicative of pneumonia or hemothorax. Wheezing does not indicate the need for physiotherapy.

A patient has been brought to the ED by the paramedics. The patient is suspected of having ARDS. What intervention should the nurse first anticipate? A) Preparing to assist with intubating the patient B) Setting up oxygen at 5 L/minute by nasal cannula C) Performing deep suctioning D) Setting up a nebulizer to administer corticosteroids

A Feedback: A patient who has ARDS usually requires intubation and mechanical ventilation. Oxygen by nasal cannula would likely be insufficient. Deep suctioning and nebulizers may be indicated, but the priority is to secure the airway.

A nurse is preparing a patient diagnosed with benign prostatic hypertrophy (BPH) for a lower urinary tract cystoscopic examination. The nurse informs the patient that the most common temporary complication experienced after this procedure is what? A) Urinary retention B) Bladder perforation C) Hemorrhage D) Nausea

A Feedback: After a cystoscopic examination, the patient with obstructive pathology may experience urine retention if the instruments used during the examination caused edema. The nurse will carefully monitor the patient with prostatic hyperplasia for urine retention. Post-procedure, the patient will experience some hematuria, but is not at great risk for hemorrhage. Unless the condition is associated with another disorder, nausea is not commonly associated with this diagnostic study. Bladder perforation is rare. 9. A patient with renal failure secondary to diabetic nephropathy has been admitted to the medical unit. What is the most life-threatening effect of renal failure for which the nurse should monitor the patient? A) Accumulation of wastes B) Retention of potassium C) Depletion of calcium D) Lack

The perioperative nurse is preparing to discharge a female patient home from day surgery performed Test Bank - Brunner & Suddarth's Textbook of Medical-Surgical Nursing 14e (Hinkle 2017) 378 under general anesthetic. What instruction should the nurse give the patient prior to the patient leaving the hospital? A) The patient should not drive herself home. B) The patient should take an OTC sleeping pill for 2 nights. C) The patient should attempt to eat a large meal at home to aid wound healing. D) The patient should remain in bed for the first 48 hours postoperative.

A Feedback: Although recovery time varies, depending on the type and extent of surgery and the patients overall condition, instructions usually advise limited activity for 24 to 48 hours. Complete bedrest is contraindicated in most cases, however. During this time, the patient should not drive a vehicle and should eat only as tolerated. The nurse does not normally make OTC recommendations for hypnotics.

A nurse is caring for a 73-year-old patient with a urethral obstruction related to prostatic enlargement. When planning this patients care, the nurse should be aware of the consequent risk of what complication? A) Urinary tract infection B) Enuresis C) Polyuria D) Proteinuria

A Feedback: An obstruction of the bladder outlet, such as in advanced benign prostatic hyperplasia, results in abnormally high voiding pressure with a slow, prolonged flow of urine. The urine may remain in the bladder, which increases the potential of a urinary tract infection. Older male patients are at risk for prostatic enlargement, which causes urethral obstruction and can result in hydronephrosis, renal failure, and urinary tract infections.

An x-ray of a trauma patient reveals rib fractures and the patient is diagnosed with a small flail chest injury. Which intervention should the nurse include in the patients plan of care? A) Suction the patients airway secretions. B) Immobilize the ribs with an abdominal binder. C) Prepare the patient for surgery. D) Immediately sedate and intubate the patient.

A Feedback: As with rib fracture, treatment of flail chest is usually supportive. Management includes clearing secretions from the lungs, and controlling pain. If only a small segment of the chest is involved, it is important to clear the airway through positioning, coughing, deep breathing, and suctioning. Intubation is required for severe flail chest injuries, and surgery is required only in rare circumstances to stabilize the flail segment.

You are providing preoperative teaching to a patient scheduled for hip replacement surgery in 1 month. During the preoperative teaching, the patient gives you a list of medications she takes, the dosage, and frequency. Which of the following interventions provides the patient with the most accurate information? A) Instruct the patient to stop taking St. Johns wort at least 2 weeks prior to surgery due to its interaction with anesthetic agents. B) Instruct the patient to continue taking ephedrine prior to surgery due to its beneficial effect on blood pressure. C) Instruct the patient to discontinue Synthroid due to its effect on blood coagulation and the potential for heart dysrhythmias. D) Instruct the patient to continue any herbal supplements unless otherwise instructed, and inform the patient that these supplements have minimal effect on the surgical procedure.

A Feedback: Because of the potential effects of herbal medications on coagulation and potential lethal interactions with other medications, the nurse must ask surgical patients specifically about the use of these agents, document their use, and inform the surgical team and anesthesiologist, anesthetist, or nurse anesthetist. Currently, it is recommended that the use of herbal products be discontinued at least 2 weeks before surgery. Patients with uncontrolled thyroid disorders are at risk for thyrotoxicosis and respiratory failure. The administration of Synthroid is imperative in the preoperative period. The use of ephedrine in the preoperative phase can cause hypertension and should be avoided.

The nurse has tested the pH of urine from a patients newly created ileal conduit and obtained a result of 6.8. What is the nurses best response to this assessment finding? A) Obtain an order to increase the patients dose of ascorbic acid. B) Administer IV sodium bicarbonate as ordered. C) Encourage the patient to drink at least 500 mL of water and retest in 3 hours. D) Irrigate the ileal conduit with a dilute citric acid solution as ordered.

A Feedback: Because severe alkaline encrustation can accumulate rapidly around the stoma, the urine pH is kept below 6.5 by administration of ascorbic acid by mouth. An increased pH may suggest a need to increase ascorbic acid dosing. This is not treated by administering bicarbonate or citric acid, nor by increasing fluid intake.

A patient with kidney stones is scheduled for extracorporeal shock wave lithotripsy (ESWL). What should the nurse include in the patients post-procedure care? A) Strain the patients urine following the procedure. B) Administer a bolus of 500 mL normal saline following the procedure. C) Monitor the patient for fluid overload following the procedure. D) Insert a urinary catheter for 24 to 48 hours after the procedure.

A Feedback: Following ESWL, the nurse should strain the patients urine for gravel or sand. There is no need to administer an IV bolus after the procedure and there is not a heightened risk of fluid overload. Catheter insertion is not normally indicated following ESWL.

The perioperative nurse is writing a care plan for a patient who has returned from surgery 2 hours prior. Which measure should the nurse implement to most decrease the patients risk of developing pulmonary emboli (PE)? A) Early ambulation B) Increased dietary intake of protein C) Maintaining the patient in a supine position Test Bank - Brunner & Suddarth's Textbook of Medical-Surgical Nursing 14e (Hinkle 2017) 461 D) Administering aspirin with warfarin

A Feedback: For patients at risk for PE, the most effective approach for prevention is to prevent deep vein thrombosis. Active leg exercises to avoid venous stasis, early ambulation, and use of elastic compression stocking are general preventive measures. The patient does not require increased dietary intake of protein directly related to prevention of PE, although it will assist in wound healing during the postoperative period. The patient should not be maintained in one position, but frequently repositioned, unless contraindicated by the surgical procedure. Aspirin should never be administered with warfarin because it will increase the patients risk for bleeding. 19. The school nurse is presenting a class on smoking cessation

A football player is thought to have sustained an injury to his kidneys from being tackled from behind. The ER nurse caring for the patient reviews the initial orders written by the physician and notes that an order to collect all voided urine and send it to the laboratory for analysis. The nurse understands that this nursing intervention is important for what reason? A) Hematuria is the most common manifestation of renal trauma and blood losses may be microscopic, so laboratory analysis is essential. B) Intake and output calculations are essential and the laboratory will calculate the precise urine output produced by this patient. Test Bank - Brunner & Suddarth's Textbook of Medical-Surgical Nursing 14e (Hinkle 2017) 1018 C) A creatinine clearance study may be ordered at a later time and the laboratory will hold all urine until it is determined if the test will be necessary. D) There is great concern about electrolyte imbalances and the laboratory will monitor the urine for changes in potassium and sodium concentrations.

A Feedback: Hematuria is the most common manifestation of renal trauma; its presence after trauma suggests renal injury. Hematuria may not occur, or it may be detectable only on microscopic examination. All urine should be saved and sent to the laboratory for analysis to detect RBCs and to evaluate the course of bleeding. Measuring intake and output is not a function of the laboratory. The laboratory does not save urine to test creatinine clearance at a later time. The laboratory does not monitor the urine for sodium or potassium concentrations.

A 45-year-old man with diabetic nephropathy has ESKD and is starting dialysis. What should the nurse teach the patient about hemodialysis? A) Hemodialysis is a treatment option that is usually required three times a week. B) Hemodialysis is a program that will require you to commit to daily treatment. C) This will require you to have surgery and a catheter will need to be inserted into your abdomen. D) Hemodialysis is a treatment that is used for a few months until your kidney heals and starts to produce urine again.

A Feedback: Hemodialysis is the most commonly used method of dialysis. Patients receiving hemodialysis must undergo treatment for the rest of their lives or until they undergo successful kidney transplantation. Treatments usually occur three times a week for at least 3 to 4 hours per treatment.

A patient is undergoing diagnostic testing for a suspected urinary obstruction. The nurse should know that incomplete emptying of the bladder due to bladder outlet obstruction can cause what? A) Hydronephrosis B) Nephritic syndrome C) Pylonephritis D) Nephrotoxicity

A Feedback: If voiding dysfunction goes undetected and untreated, the upper urinary system may become compromised. Chronic incomplete bladder emptying from poor detrusor pressure results in recurrent bladder infection. Incomplete bladder emptying due to bladder outlet obstruction, causing high-pressure detrusor contractions, can result in hydronephrosis from the high detrusor pressure that radiates up the ureters to the renal pelvis. This problem does not normally cause nephritic syndrome or pyelonephritis. Nephrotoxicity results from chemical causes.

The nurse is caring for a patient who is scheduled for a lobectomy for a diagnosis of lung cancer. While assisting with a subclavian vein central line insertion, the nurse notes the clients oxygen saturation rapidly dropping. The patient complains of shortness of breath and becomes tachypneic. The nurse suspects a pneumothorax has developed. Further assessment findings supporting the presence of a pneumothorax include what? A) Diminished or absent breath sounds on the affected side B) Paradoxical chest wall movement with respirations C) Sudden loss of consciousness D) Muffled heart sounds

A Feedback: In the case of a simple pneumothorax, auscultating the breath sounds will reveal absent or diminished breath sounds on the affected side. Paradoxical chest wall movements occur in flail chest conditions. Sudden loss of consciousness does not typically occur. Muffled or distant heart sounds occur in pericardial tamponade.

The nurse is preparing an elderly patient for a scheduled removal of orthopedic hardware, a procedure to be performed under general anesthetic. For which adverse effect should the nurse most closely monitor the patient? A) Hypothermia B) Pulmonary edema C) Cerebral ischemia D) Arthritis

A Feedback: Inadvertent hypothermia may occur as a result of a low temperature in the OR, infusion of cold fluids, inhalation of cold gases, open body wounds or cavities, decreased muscle activity, advanced age, or the pharmaceutical agents used (e.g., vasodilators, phenothiazines, general anesthetics). The anesthetist monitors for pulmonary edema and cerebral ischemia. Arthritis is not an adverse effect of surgical anesthesia. 2. The OR nurse act

The OR nurse acts in the circulating role during a patients scheduled cesarean section. For what task is this nurse solely responsible? A) Performing documentation B) Estimating the patients blood loss C) Setting up the sterile tables D) Keeping track of drains and sponges

A Feedback: Main responsibilities of the circulating nurse include verifying consent; coordinating the team; and ensuring cleanliness, proper temperature and humidity, lighting, safe function of equipment, and the availability of supplies and materials. The circulating nurse monitors aseptic practices to avoid breaks in technique while coordinating the movement of related personnel as well as implementing fire safety precautions. The circulating nurse also monitors the patient and documents specific activities throughout the operation to ensure the patients safety and well-being. Estimating the patients blood loss is the surgeons responsibility; setting up the sterile tables is the responsibility of the first scrub; and keeping track of the drains and sponges is the joint responsibility of the circulating nurse and the scrub nurse.

A nurse is caring for a 6-year-old patient with cystic fibrosis. In order to enhance the childs nutritional Test Bank - Brunner & Suddarth's Textbook of Medical-Surgical Nursing 14e (Hinkle 2017) 476 status, what intervention should most likely be included in the plan of care? A) Pancreatic enzyme supplementation with meals B) Provision of five to six small meals per day rather than three larger meals C) Total parenteral nutrition (TPN) D) Magnesium, thiamine, and iron supplementation

A Feedback: Nearly 90% of patients with CF have pancreatic exocrine insufficiency and require oral pancreatic enzyme supplementation with meals. Frequent, small meals or TPN are not normally indicated. Vitamin supplements are required, but specific replacement of magnesium, thiamine, and iron is not typical.

The nurse is caring for an 88-year-old patient who is recovering from an ileac-femoral bypass graft. The patient is day 2 postoperative and has been mentally intact, as per baseline. When the nurse assesses the patient, it is clear that he is confused and has been experiencing disturbed sleep patterns and impaired psychomotor skills. What should the nurse suspect is the problem with the patient? A) Postoperative delirium B) Postoperative dementia C) Senile dementia D) Senile confusion

A Feedback: Postoperative delirium, characterized by confusion, perceptual and cognitive deficits, altered attention levels, disturbed sleep patterns, and impaired psychomotor skills, is a significant problem for older adults. Dementia does not have a sudden onset. Senile confusion is not a recognized health problem.

The nurse on a urology unit is working with a patient who has been diagnosed with oxalate renal calculi. When planning this patients health education, what nutritional guidelines should the nurse provide? Test Bank - Brunner & Suddarth's Textbook of Medical-Surgical Nursing 14e (Hinkle 2017) 1036 A) Restrict protein intake as ordered. B) Increase intake of potassium-rich foods. C) Follow a low-calcium diet. D) Encourage intake of food containing oxalates.

A Feedback: Protein is restricted to 60 g/d, while sodium is restricted to 3 to 4 g/d. Low-calcium diets are generally not recommended except for true absorptive hypercalciuria. The patient should avoid intake of oxalatecontaining foods and there is no need to increase potassium intake.

A 42-year-old woman comes to the clinic complaining of occasional urinary incontinence when she sneezes. The clinic nurse should recognize what type of incontinence? A) Stress incontinence B) Reflex incontinence C) Overflow incontinence D) Functional incontinence

A Feedback: Stress incontinence is the involuntary loss of urine through an intact urethra as a result of sudden increase in intra-abdominal pressure. Reflex incontinence is loss of urine due to hyperreflexia or involuntary urethral relaxation in the absence of normal sensations usually associated with voiding. Overflow incontinence is an involuntary urine loss associated with overdistension of the bladder. Functional incontinence refers to those instances in which the function of the lower urinary tract is intact, but other factors (outside the urinary system) make it difficult or impossible for the patient to reach the toilet in time for voiding.

An adult patient has tested positive for tuberculosis (TB). While providing patient teaching, what information should the nurse prioritize? A) The importance of adhering closely to the prescribed medication regimen B) The fact that the disease is a lifelong, chronic condition that will affect ADLs C) The fact that TB is self-limiting, but can take up to 2 years to resolve D) The need to work closely with the occupational and physical therapists

A Feedback: Successful treatment of TB is highly dependent on careful adherence to the medication regimen. The disease is not self-limiting; occupational and physical therapy are not necessarily indicated. TB is curable.

The nurse is planning patient teaching for a patient with ESKD who is scheduled for the creation of a fistula. The nurse would include which of the following in teaching the patient about the fistula? A) A vein and an artery in your arm will be attached surgically. B) The arm should be immobilized for 4 to 6 days. C) One needle will be inserted into the fistula for each dialysis treatment. D) The fistula can be used 2 days after the surgery for dialysis treatment.

A Feedback: The fistula joins an artery and a vein, either side-to-side or end-to-end. This access will need time, usually 2 to 3 months, to mature before it can be used. The patient is encouraged to perform exercises to increase the size of the affected vessels (e.g., squeezing a rubber ball for forearm fistulas). Two needles will be inserted into the fistula for each dialysis treatment.

While planning a patients care, the nurse identifies nursing actions to minimize the patients pleuritic pain. Which intervention should the nurse include in the plan of care? A) Avoid actions that will cause the patient to breathe deeply. B) Ambulate the patient at least three times daily. C) Arrange for a soft-textured diet and increased fluid intake. D) Encourage the patient to speak as little as possible

A Feedback: The key characteristic of pleuritic pain is its relationship to respiratory movement. Taking a deep breath, coughing, or sneezing worsens the pain. A soft diet is not necessarily indicated and there is no need for the patient to avoid speaking. Ambulation has multiple benefits, but pain management is not among them. 18. The perioperative

One of the things a nurse has taught to a patient during preoperative teaching is to have nothing by mouth for the specified time before surgery. The patient asks the nurse why this is important. What is the most appropriate response for the patient? A) You will need to have food and fluid restricted before surgery so you are not at risk for choking. B) The restriction of food or fluid will prevent the development of pneumonia related to decreased lung capacity. C) The presence of food in the stomach interferes with the absorption of anesthetic agents. D) By withholding food for 8 hours before surgery, you will not develop constipation in the postoperative period.

A Feedback: The major purpose of withholding food and fluid before surgery is to prevent aspiration. There is no scientific basis for withholding food and the development of pneumonia or interference with absorption of anesthetic agents. Constipation in patients in the postoperative period is related to the anesthesia, not from withholding food or fluid in the hours before surgery.

The nurse is performing the shift assessment of a postsurgical patient. The nurse finds his mental status, level of consciousness, speech, and orientation are intact and at baseline, but the patient tells you he is very anxious. What should the nurse do next? A) Assess the patients oxygen levels. B) Administer antianxiety medications. C) Page the patients the physician. D) Initiate a social work referral.

A Feedback: The nurse assesses the patients mental status and level of consciousness, speech, and orientation and compares them with the preoperative baseline. Although a change in mental status or postoperative restlessness may be related to anxiety, pain, or medications, it may also be a symptom of oxygen deficit or hemorrhage. Antianxiety medications are not given until the cause of the anxiety is known. The physician is notified only if the reason for the anxiety is serious or if an order for medication is needed. A social work consult is inappropriate at this time.

The nurse is working on the renal transplant unit. To reduce the risk of infection in a patient with a transplanted kidney, it is imperative for the nurse to do what? A) Wash hands carefully and frequently. B) Ensure immediate function of the donated kidney. C) Instruct the patient to wear a face mask. D) Bar visitors from the patients room.

A Feedback: The nurse ensures that the patient is protected from exposure to infection by hospital staff, visitors, and other patients with active infections. Careful handwashing is imperative; face masks may be worn by hospital staff and visitors to reduce the risk for transmitting infectious agents while the patient is receiving high doses of immunosuppressants. Visitors may be limited, but are not normally barred outright. Ensuring kidney function is vital, but does not prevent infection.

The nurse is assessing a patient suspected of having developed acute glomerulonephritis. The nurse should expect to address what clinical manifestation that is characteristic of this health problem? A) Hematuria B) Precipitous decrease in serum creatinine levels C) Hypotension unresolved by fluid administration D) Glucosuria

A Feedback: The primary presenting feature of acute glomerulonephritis is hematuria (blood in the urine), which may be microscopic (identifiable through microscopic examination) or macroscopic or gross (visible to the eye). Proteinuria, primarily albumin, which is present, is due to increased permeability of the glomerular membrane. Blood urea nitrogen (BUN) and serum creatinine levels may rise as urine output drops. Some degree of edema and hypertension is noted in most patients.

A patient asks the nurse why kidney problems can cause gastrointestinal disturbances. What relationship should the nurse describe? A) The right kidneys proximity to the pancreas, liver, and gallbladder B) The indirect impact of digestive enzymes on renal function C) That the peritoneum encapsulates the GI system and the kidneys D) The left kidneys connection to the common bile duct

A Feedback: The proximity of the right kidney to the colon, duodenum, head of the pancreas, common bile duct, liver, and gallbladder may cause GI disturbances. The proximity of the left kidney to the colon (splenic flexure), stomach, pancreas, and spleen may also result in intestinal symptoms. Digestive enzymes do not affect renal function and the left kidney is not connected to the common bile duct.

A nurse knows that specific areas in the ureters have a propensity for obstruction. Prompt management of renal calculi is most important when the stone is located where? A) In the ureteropelvic junction B) In the ureteral segment near the sacroiliac junction C) In the ureterovesical junction D) In the urethra

A Feedback: The three narrowed areas of each ureter are the ureteropelvic junction, the ureteral segment near the sacroiliac junction, and the ureterovescial junction. These three areas of the ureters have a propensity for obstruction by renal calculi or stricture. Obstruction of the ureteropelvic junction is most serious because of its close proximity to the kidney and the risk of associated kidney dysfunction. The urethra is not part of the ureter.

A patient with elevated BUN and creatinine values has been referred by her primary physician for further evaluation. The nurse should anticipate the use of what initial diagnostic test? Test Bank - Brunner & Suddarth's Textbook of Medical-Surgical Nursing 14e (Hinkle 2017) 1004 A) Ultrasound B) X-ray C) Computed tomography (CT) D) Nuclear scan

A Feedback: Ultrasonography is a noninvasive procedure that passes sound waves into the body through a transducer to detect abnormalities of internal tissues and organs. Structures of the urinary system create characteristic ultrasonographic images. Because of its sensitivity, ultrasonography has replaced many other diagnostic tests as the initial diagnostic procedure.

The nurse is providing pre-procedure teaching about an ultrasound. The nurse informs the patient that in preparation for an ultrasound of the lower urinary tract the patient will require what? A) Increased fluid intake to produce a full bladder Test Bank - Brunner & Suddarth's Textbook of Medical-Surgical Nursing 14e (Hinkle 2017) 998 B) IV administration of radiopaque contrast agent C) Sedation and intubation D) Injection of a radioisotope

A Feedback: Ultrasonography requires a full bladder; therefore, fluid intake should be encouraged before the procedures. The administration of a radiopaque contrast agent is required to perform IV urography studies, such as an IV pyelogram. Ultrasonography is a quick and painless diagnostic test and does not require sedation or intubation. The injection of a radioisotope is required for nuclear scan and ultrasonography is not in this category of diagnostic studies.

The OR nurse is participating in the appendectomy of a 20 year-old female patient who has a dangerously low body mass index. The nurse recognizes the patients consequent risk for hypothermia. What action should the nurse implement to prevent the development of hypothermia? Test Bank - Brunner & Suddarth's Textbook of Medical-Surgical Nursing 14e (Hinkle 2017) 375 A) Ensure that IV fluids are warmed to the patients body temperature. B) Transfuse packed red blood cells to increase oxygen carrying capacity. C) Place warmed bags of normal saline at strategic points around the patients body. D) Monitor the patients blood pressure and heart rate vigilantly.

A Feedback: Warmed IV fluids can prevent the development of hypothermia. Applying warmed bags of saline around the patient is not common practice. The patient is not transfused to prevent hypothermia. Blood pressure and heart rate monitoring are important, but do not relate directly to the risk for hypothermia.

A female patient has been prescribed a course of antibiotics for the treatment of a UTI. When providing health education for the patient, the nurse should address what topic? A) The risk of developing a vaginal yeast infection as a consequent of antibiotic therapy Test Bank - Brunner & Suddarth's Textbook of Medical-Surgical Nursing 14e (Hinkle 2017) 1045 B) The need to expect a heavy menstrual period following the course of antibiotics C) The risk of developing antibiotic resistance after the course of antibiotics D) The need to undergo a series of three urine cultures after the antibiotics have been completed

A Feedback: Yeast vaginitis occurs in as many as 25% of patients treated with antimicrobial agents that affect vaginal flora. Yeast vaginitis can cause more symptoms and be more difficult and costly to treat than the original UTI. Antibiotics do not affect menstrual periods and serial urine cultures are not normally necessary. Resistance is normally a result of failing to complete a prescribed course of antibiotics.

The nurse is caring for a patient who has returned to the postsurgical suite after post-anesthetic recovery from a nephrectomy. The nurses most recent hourly assessment reveals a significant drop in level of consciousness and BP as well as scant urine output over the past hour. What is the nurses best response? A) Assess the patient for signs of bleeding and inform the physician. B) Monitor the patients vital signs every 15 minutes for the next hour. C) Reposition the patient and reassess vital signs. D) Palpate the patients flanks for pain and inform the physician.

A Test Bank - Brunner & Suddarth's Textbook of Medical-Surgical Nursing 14e (Hinkle 2017) 1030 Feedback: Bleeding may be suspected when the patient experiences fatigue and when urine output is less than 30 mL/h. The physician must be made aware of this finding promptly. Palpating the patients flanks would cause intense pain that is of no benefit to assessment.

The nurse and urologist have both been unsuccessful in catheterizing a patient with a prostatic obstruction and a full bladder. What approach does the nurse anticipate the physician using to drain the patients bladder? A) Insertion of a suprapubic catheter B) Scheduling the patient immediately for a prostatectomy C) Application of warm compresses to the perineum to assist with relaxation D) Medication administration to relax the bladder muscles and reattempting catheterization in 6 hours Ans:

A Test Bank - Brunner & Suddarth's Textbook of Medical-Surgical Nursing 14e (Hinkle 2017) 1038 Feedback: When the patient cannot void, catheterization is used to prevent overdistention of the bladder. In the case of prostatic obstruction, attempts at catheterization by the urologist may not be successful, requiring insertion of a suprapubic catheter. A prostatectomy may be necessary, but would not be undertaken for the sole purpose of relieving a urethral obstruction. Delaying by applying compresses or administering medications could result in harm.

The nurse is providing patient teaching to a patient diagnosed with acute rhinosinusitis. For what possible complication should the nurse teach the patient to seek immediate follow-up? A) Periorbital edema B) Headache unrelieved by OTC medications C) Clear drainage from nose D) Blood-tinged mucus when blowing the nose

A Test Bank - Brunner & Suddarth's Textbook of Medical-Surgical Nursing 14e (Hinkle 2017) 447 Feedback: Patient teaching is an important aspect of nursing care for the patient with acute rhinosinusitis. The nurse instructs the patient about symptoms of complications that require immediate follow-up. Referral to a physician is indicated if periorbital edema and severe pain on palpation occur. Clear drainage and bloodtinged mucus do not require follow-up if the patient has acute rhinosinusitis. A persistent headache does not necessarily warrant immediate follow-up.

A gerontologic nurse is assessing a patient who has numerous comorbid health problems. What assessment findings should prompt the nurse to suspect a UTI? Select all that apply. A) Food cravings B) Upper abdominal pain C) Insatiable thirst D) Uncharacteristic fatigue E) New onset of confusion

A gerontologic nurse is assessing a patient who has numerous comorbid health problems. What assessment findings should prompt the nurse to suspect a UTI? Select all that apply. A) Food cravings B) Upper abdominal pain C) Insatiable thirst D) Uncharacteristic fatigue E) New onset of confusion

The nurse is discharging a patient home from an outpatient surgery center. The nurse has reviewed all of the discharge instructions with the patient and her caregiver. What else should the nurse do before discharging the patient from the facility? Select all that apply. A) Provide all discharge instructions in writing. B) Provide the nurses or surgeons contact information. C) Give prescriptions to the patient. D) Irrigate the patients incision and perform a sterile dressing change. E) Administer a bolus dose of an opioid analgesic.

A, B, C Feedback: Before discharging the patient, the nurse provides written instructions, prescriptions and the nurses or surgeons telephone number. Administration of an opioid would necessitate further monitoring to ensure Test Bank - Brunner & Suddarth's Textbook of Medical-Surgical Nursing 14e (Hinkle 2017) 389 safety. A dressing change would not normally be ordered on the day of surgery.

The nurse is assessing a patient admitted with renal stones. During the admission assessment, what parameters would be priorities for the nurse to address? Select all that apply. A) Dietary history B) Family history of renal stones C) Medication history D) Surgical history E) Vaccination history

A, B, C Feedback: Dietary and medication histories and family history of renal stones are obtained to identify factors predisposing the patient to stone formation. When caring for a patient with renal stones it would not normally be a priority to assess the vaccination history or surgical history, since these factors are not usually related to the etiology of kidney stones.

A nurse in the preoperative holding area is admitting a woman prior to reduction mammoplasty. What should the nurse include in the care given to this patient? Select all that apply. A) Establishing an IV line B) Verifying the surgical site with the patient C) Taking measures to ensure the patients comfort D) Applying a grounding device to the patient E) Preparing the medications to be administered in the OR

A, B, C Feedback: In the holding area, the nurse reviews charts, identifies patients, verifies surgical site and marks site per Test Bank - Brunner & Suddarth's Textbook of Medical-Surgical Nursing 14e (Hinkle 2017) 349 institutional policy, establishes IV lines, administers medications, if prescribed, and takes measures to ensure each patients comfort. A nurse in the preoperative holding area does not prepare medications to be administered by anyone else. A grounding device is applied in the OR.

An intraoperative nurse is applying interventions that will address surgical patients risks for perioperative positioning injury. Which of the following factors contribute to this increased risk for injury in the intraoperative phase of the surgical experience? Select all that apply. A) Absence of reflexes B) Diminished ability to communicate C) Loss of pain sensation D) Nausea resulting from anesthetic E) Reduced blood pressure

A, B, C Feedback: Loss of pain sense, reflexes, and ability to communicate subjects the intraoperative patient to possible injury. Nausea and low blood pressure are not central factors that contribute to this risk, though they are adverse outcomes.

The nurse is caring for a patient who has just returned to the post-surgical unit following renal surgery. When assessing the patients output from surgical drains, the nurse should assess what parameters? Select all that apply. A) Quantity of output B) Color of the output C) Visible characteristics of the output D) Odor of the output E) pH of the output

A, B, C Feedback: Urine output and drainage from tubes inserted during surgery are monitored for amount, color, and type or characteristics. Odor and pH are not normally assessed.

The nurse is caring for a patient status after a motor vehicle accident. The patient has developed AKI. What is the nurses role in caring for this patient? Select all that apply. A) Providing emotional support for the family B) Monitoring for complications C) Participating in emergency treatment of fluid and electrolyte imbalances D) Providing nursing care for primary disorder (trauma) E) Directing nutritional interventions

A, B, C, D Feedback: The nurse has an important role in caring for the patient with AKI. The nurse monitors for complications, participates in emergency treatment of fluid and electrolyte imbalances, assesses the patients progress and response to treatment, and provides physical and emotional support. Additionally, the nurse keeps family members informed about the patients condition, helps them understand the treatments, and provides psychological support. Although the development of AKI may be the most serious problem, the nurse continues to provide nursing care indicated for the primary disorder (e.g., burns, shock, trauma, obstruction of the urinary tract). The nurse does not direct the patients nutritional status; the dietician and the physician normally collaborate on directing the patients nutritional status.

The nurse is preparing to collect an ordered urine sample for urinalysis. The nurse should be aware that this test will include what assessment parameters? Select all that apply. A) Specific gravity of the patients urine B) Testing for the presence of glucose in the patients urine C) Microscopic examination of urine sediment for RBCs D) Microscopic examination of urine sediment for casts E) Testing for BUN and creatinine in the patients urine

A, B, C, D Feedback: Urine testing includes testing for specific gravity, glucose, RBCs, and casts. BUN and creatinine are components of serum, not urine.

A patient with ESKD is scheduled to begin hemodialysis. The nurse is working with the patient to adapt the patients diet to maximize the therapeutic effect and minimize the risks of complications. The patients diet should include which of the following modifications? Select all that apply. A) Decreased protein intake B) Decreased sodium intake Test Bank - Brunner & Suddarth's Textbook of Medical-Surgical Nursing 14e (Hinkle 2017) 1022 C) Increased potassium intake D) Fluid restriction E) Vitamin D supplementation

A, B, D Feedback: Restricting dietary protein decreases the accumulation of nitrogenous wastes, reduces uremic symptoms, and may even postpone the initiation of dialysis for a few months. Restriction of fluid is also part of the dietary prescription because fluid accumulation may occur. As well, sodium is usually restricted to 2 to 3 g/day. Potassium intake is usually limited, not increased, and there is no particular need for vitamin D supplementation.

The occupational health nurse is assessing new employees at a company. What would be important to assess in employees with a potential occupational respiratory exposure to a toxin? Select all that apply. A) Time frame of exposure B) Type of respiratory protection used Test Bank - Brunner & Suddarth's Textbook of Medical-Surgical Nursing 14e (Hinkle 2017) 467 C) Immunization status D) Breath sounds E) Intensity of exposure

A, B, D, E Feedback: Key aspects of any assessment of patients with a potential occupational respiratory history include job and job activities, exposure levels, general hygiene, time frame of exposure, effectiveness of respiratory protection used, and direct versus indirect exposures. The patients current respiratory status would also be a priority. Occupational lung hazards are not normally influenced by immunizations.

When creating plans of nursing care for patients who are undergoing surgery using general anesthetic, what nursing diagnoses should the nurse identify? Select all that apply. A) Disturbed sensory perception related to anesthetic B) Risk for impaired nutrition: less than body requirements related to anesthesia C) Risk of latex allergy response related to surgical exposure D) Disturbed body image related to anesthesia Test Bank - Brunner & Suddarth's Textbook of Medical-Surgical Nursing 14e (Hinkle 2017) 371 E) Anxiety related to surgical concerns

A, C, E Feedback: Based on the assessment data, some major nursing diagnoses may include the following: anxiety related to surgical or environmental concerns, risk of latex allergy response due to possible exposure to latex in the OR environment, risk for perioperative positioning injury related to positioning in the OR, risk for injury related to anesthesia and surgical procedure, or disturbed sensory perception (global) related to general anesthesia or sedation. Malnutrition and disturbed body image are much less likely.

21. The clinic nurse is caring for a patient who has been diagnosed with emphysema and who has just had a pulmonary function test (PFT) ordered. The patient asks, What exactly is this test for? What would be the nurses best response? A) A PFT measures how much air moves in and out of your lungs when you breathe. B) A PFT measures how much energy you get from the oxygen you breathe. C) A PFT measures how elastic your lungs are. D) A PFT measures whether oxygen and carbon dioxide move between your lungs and your blood.

Ans: A Feedback: PFTs are routinely used in patients with chronic respiratory disorders. They are performed to assess respiratory function and to determine the extent of dysfunction. Such tests include measurements of lung volumes, ventilatory function, and the mechanics of breathing, diffusion, and gas exchange. Lung elasticity and diffusion can often be implied from PFTs, but they are not directly assessed. Energy obtained from respiration is not measured directly.

17. The nurse has admitted a patient who is scheduled for a thoracic resection. The nurse is providing preoperative teaching and is discussing several diagnostic studies that will be required prior to surgery. Which study will be performed to determine whether the planned resection will leave sufficient functioning lung tissue? A) Pulmonary function studies B) Exercise tolerance tests C) Arterial blood gas values D) Chest x-ray

Ans: A Feedback: Pulmonary function studies are performed to determine whether the planned resection will leave sufficient functioning lung tissue. ABG values are assessed to provide a more complete picture of the functional capacity of the lung. Exercise tolerance tests are useful to determine if the patient who is a candidate for pneumonectomy can tolerate removal of one of the lungs. Preoperative studies, such as a chest x-ray, are performed to provide a baseline for comparison during the postoperative period and to detect any unsuspected abnormalities.

40. The nurse has assessed a patients family history for three generations. The presence of which respiratory disease would justify this type of assessment? A) Asthma B) Obstructive sleep apnea C) Community-acquired pneumonia D) Pulmonary edema

Ans: A Feedback: Test Bank - Brunner & Suddarth's Textbook of Medical-Surgical Nursing 14e (Hinkle 2017) 415 Asthma is a respiratory illness that has genetic factors. Sleep apnea, pneumonia, and pulmonary edema lack genetic risk factors.

The perioperative nurse has completed the presurgical assessment of an 82-year-old female patient who is scheduled for a left total knee replacement. When planning this patients care, the nurse should address the consequences of the patients aging cardiovascular system. These include an increased risk of which of the following? A) Hypervolemia B) Hyponatremia C) Hyperkalemia D) Hyperphosphatemia

Ans: A Feedback: The aging heart and blood vessels have decreased ability to respond to stress. Reduced cardiac output Test Bank - Brunner & Suddarth's Textbook of Medical-Surgical Nursing 14e (Hinkle 2017) 373 and limited cardiac reserve make the elderly patient vulnerable to changes in circulating volume and blood oxygen levels. There is not an increased risk for hypopnea, hyperkalemia, or hyperphosphatemia because of an aging cardiovascular system.

24. The OR nurse is setting up a water-seal chest drainage system for a patient who has just had a thoracotomy. The nurse knows that the amount of suction in the system is determined by the water level. At what suction level should the nurse set the system? A) 20 cm H2O B) 15 cm H2O C) 10 cm H2O D) 5 cm H2O

Ans: A Feedback: The amount of suction is determined by the water level. It is usually set at 20 cm H2O; adding more fluid results in more suction.

38. Postural drainage has been ordered for a patient who is having difficulty mobilizing her bronchial secretions. Before repositioning the patient and beginning treatment, the nurse should perform what health assessment? A) Chest auscultation B) Pulmonary function testing C) Chest percussion D) Thoracic palpation

Ans: A Test Bank - Brunner & Suddarth's Textbook of Medical-Surgical Nursing 14e (Hinkle 2017) 433 Feedback: Chest auscultation should be performed before and after postural drainage in order to evaluate the effectiveness of the therapy. Percussion and palpation are less likely to provide clinically meaningful data for the nurse. PFTs are normally beyond the scope of the nurse and are not necessary immediately before postural drainage.

7. The nurse is caring for a patient whose recent unexplained weight loss and history of smoking have prompted diagnostic testing for cancer. What symptom is most closely associated with the early stages of laryngeal cancer? A) Hoarseness B) Dyspnea C) Dysphagia D) Frequent nosebleeds

Ans: A Test Bank - Brunner & Suddarth's Textbook of Medical-Surgical Nursing 14e (Hinkle 2017) 438 Feedback: Hoarseness is an early symptom of laryngeal cancer. Dyspnea, dysphagia, and lumps are later signs of laryngeal cancer. Alopecia is not associated with a diagnosis of laryngeal cancer.

9. You are admitting a patient to your medical unit after the patient has been transferred from the emergency department. What is your priority nursing action at this time? A) Identifying the immediate needs of the patient B) Checking the admitting physicians orders C) Obtaining a baseline set of vital signs D) Allowing the family to be with the patient

Ans: A Feedback: Among the nurses important functions in health care delivery, identifying the patients immediate needs and working in concert with the patient to address them is most important. The other nursing functions are important, but they are not the most important functions.

18. Nursing is, by necessity, a flexible profession. It has adapted to meet both the expectations and the changing health needs of our aging population. What is one factor that has impacted the need for certified nurse practitioners (CNPs)? A) The increased need for primary care providers B) The need to improve patient diagnostic services C) The push to drive institutional excellence Test Bank - Brunner & Suddarth's Textbook of Medical-Surgical Nursing 14e (Hinkle 2017) 11 D) The need to decrease the number of medical errors

Ans: A Feedback: CNPs who are educationally prepared with a population focus in adult-gerontology or pediatrics receive additional focused training in primary care or acute care. CNPs help meet the need for primary care providers. Diagnostic services, institutional excellence, and reduction of medical errors are congruent with the CNP role, but these considerations are the not primary impetus for the increased role for CNPs.

21. A public health nurse has been commissioned to draft a health promotion program that meets the health care needs and expectations of the community. Which of the following focuses is most likely to influence the nurses choice of interventions? A) Management of chronic conditions and disability B) Increasing need for self-care among a younger population C) A shifting focus to disease management D) An increasing focus on acute conditions and rehabilitation

Ans: A Feedback: In response to current priorities, health care must focus more on management of chronic conditions and disability than in previous times. The other answers are incorrect because the change in focus of health care is not an increasing need for self-care among our aging population; our focus is shifting away from disease management, not toward it; and we are moving away from the management of acute conditions to managing chronic conditions.

23. You are writing a care plan for an 85-year-old patient who has community-acquired pneumonia and you note decreased breath sounds to bilateral lung bases on auscultation. What is the most appropriate nursing diagnosis for this patient? A) Ineffective airway clearance related to tracheobronchial secretions B) Pneumonia related to progression of disease process C) Poor ventilation related to acute lung infection D) Immobility related to fatigue

Ans: A Feedback: Nursing diagnoses are not medical diagnoses or treatments. The most appropriate nursing diagnosis for this patient is ineffective airway clearance related to copious tracheobronchial secretions. Pneumonia and poor ventilation are not nursing diagnoses. Immobility is likely, but is less directly related to the patients admitting medical diagnosis and the nurses assessment finding.

31. A hospitals current quality improvement program has integrated the principles of the Institute for Healthcare Improvement (IHI) 5 Million Lives Campaign. How can the hospital best achieve the campaign goals of reducing preventable harm and death? A) By adhering to EBP guidelines B) By reducing nurse-to-patient ratios and increasing accountability C) By having researchers from outside the facility evaluate care D) By involving patients and families in their care planning Test Bank - Brunner & Suddarth's Textbook of Medical-Surgical Nursing 14e (Hinkle 2017) 1

Ans: A Feedback: The 5 Million Lives Campaign posits that if evidence-based guidelines it advocated were voluntarily implemented by U.S. hospitals, 5 million lives would be saved from either harm or death over a two- year period. Nurse-to-patient ratios, family participation, and independent evaluation are not stated components of the campaign.

24. You are providing care for a patient who has a diagnosis of pneumonia attributed toStreptococcus pneumonia infection. Which of the following aspects of nursing care would constitute part of the planning phase of the nursing process? A) Achieve SaO2 92% at all times. B) Auscultate chest q4h. C) Administer oral fluids q1h and PRN. D) Avoid overexertion at all times.

Ans: A Feedback: The planning phase entails specifying the immediate, intermediate, and long-term goals of nursing action, such as maintaining a certain level of oxygen saturation in a patient with pneumonia. Providing Test Bank - Brunner & Suddarth's Textbook of Medical-Surgical Nursing 14e (Hinkle 2017) 52 fluids and avoiding overexertion are parts of the implementation phase of the nursing process. Chest auscultation is an assessment.

32. Over the past several decades, nursing roles have changed and expanded in many ways. Which of the following factors has provided the strongest impetus for this change? A) The need to decrease the cost of health care B) The need to improve the quality of nursing education C) The need to increase the number of nursing jobs available D) The need to increase the public perception of nursing

Ans: A Feedback: The role of the nurse has expanded to improve the distribution of health care services and to decrease the cost of health care. The other answers are incorrect because the expansion of roles in nursing did not occur to improve education, increase the number of nursing jobs, or increase public perception.

27. Our world is connected by a sophisticated communication system that makes much health information instantly accessible, no matter where the patient is being treated. This instant access to health information has impacted health care delivery strategies, including the delivery of nursing care. What is one way the delivery of health care has been impacted by this phenomenon? A) Brisk changes as well as swift obsolescence Test Bank - Brunner & Suddarth's Textbook of Medical-Surgical Nursing 14e (Hinkle 2017) 15 B) Rapid change that is nearly permanent C) Limitations on the settings where care can be provided D) Increased need for social acceptance

Ans: A Feedback: The sophisticated communication systems that connect most parts of the world, with the capability of rapid storage, retrieval, and dissemination of information, have stimulated brisk change as well as swift obsolescence in health care delivery strategies. The other answers are incorrect because, although we have rapid change in the delivery of nursing care, it does not last a long time; it is evolving as health care itself evolves. Giving nursing care has not become easier, it becomes more complex with every change; and it does not need to be more socially acceptable; it needs to be more culturally sensitive.

2. A nurse is speaking to a group of prospective nursing students about what it is like to be a nurse. What is one characteristic the nurse would cite as necessary to possess to be an effective nurse? A) Sensitivity to cultural differences B) Team-focused approach to problem-solving C) Strict adherence to routine D) Ability to face criticism

Ans: A Feedback: To promote an effective nurse-patient relationship and positive outcomes of care, nursing care must be culturally competent, appropriate, and sensitive to cultural differences. Team-focused nursing and strict adherence to routine are not characteristics needed to be an effective nurse. The ability to handle criticism is important, but to a lesser degree than cultural competence.

22. Your older adult patient has a diagnosis of rheumatoid arthritis (RA) and has been achieving only modest relief of her symptoms with the use of nonsteroidal anti-inflammatory drugs (NSAIDs). When creating this patients plan of care, which nursing diagnosis would most likely be appropriate? A) Self-care deficit related to fatigue and joint stiffness B) Ineffective airway clearance related to chronic pain C) Risk for hopelessness related to body image disturbance D) Anxiety related to chronic joint pain

Ans: A Test Bank - Brunner & Suddarth's Textbook of Medical-Surgical Nursing 14e (Hinkle 2017) 51 Feedback: Nursing diagnoses are actual or potential problems that can be managed by independent nursing actions. Self-care deficit would be the most likely consequence of rheumatoid arthritis. Anxiety and hopelessness are plausible consequences of a chronic illness such as RA, but challenges with self-care are more likely. Ineffective airway clearance is unlikely.

30. A nurse educator is reviewing the indications for chest drainage systems with a group of medical nurses. What indications should the nurses identify? Select all that apply. A) Post thoracotomy B) Spontaneous pneumothorax C) Need for postural drainage D) Chest trauma resulting in pneumothorax E) Pleurisy

Ans: A, B, D Feedback: Chest drainage systems are used in treatment of spontaneous pneumothorax and trauma resulting in pneumothorax. Postural drainage and pleurisy are not criteria for use of a chest drainage system.

The nurse is planning the care of a patient who has type 1 diabetes and who will be undergoing knee replacement surgery. This patients care plan should reflect an increased risk of what postsurgical complications? Select all that apply. A) Hypoglycemia B) Delirium C) Acidosis D) Glucosuria E) Fluid overload Ans: A, C, D

Ans: A, C, D Feedback: Hypoglycemia may develop during anesthesia or postoperatively from inadequate carbohydrates or excessive administration of insulin. Hyperglycemia, which can increase the risk for surgical wound infection, may result from the stress of surgery, which can trigger increased levels of catecholamine. Other risks are acidosis and glucosuria. Risks of fluid overload and delirium are not normally increased.

33. Achieving adequate pain management for a postoperative patient will require sophisticated critical thinking skills by the nurse. What are the potential benefits of critical thinking in nursing? Select all that apply. A) Enhancing the nurses clinical decision making B) Identifying the patients individual preferences Test Bank - Brunner & Suddarth's Textbook of Medical-Surgical Nursing 14e (Hinkle 2017) 56 C) Planning the best nursing actions to assist the patient D) Increasing the accuracy of the nurses judgments E) Helping identify the patients priority needs

Ans: A, C, D, E Feedback: Independent judgments and decisions evolve from a sound knowledge base and the ability to synthesize information within the context in which it is presented. Critical thinking enhances clinical decision making, helping to identify patient needs and the best nursing actions that will assist patients in meeting those needs. Critical thinking does not normally focus on identify patient desires; these would be identified by asking the patient.

13. The nurse is assessing a newly admitted medical patient and notes there is a depression in the lower portion of the patients sternum. This patients health record should note the presence of what chest deformity? A) A barrel chest B) A funnel chest C) A pigeon chest D) Kyphoscoliosis

Ans: B Feedback: A funnel chest occurs when there is a depression in the lower portion of the sternum, and this may lead to compression of the heart and great vessels, resulting in murmurs. A barrel chest is characterized by an increase in the anteroposterior diameter of the thorax and is a result of overinflation of the lungs. A pigeon chest occurs as a result of displacement of the sternum and includes an increase in the Test Bank - Brunner & Suddarth's Textbook of Medical-Surgical Nursing 14e (Hinkle 2017) 403 anteroposterior diameter. Kyphoscoliosis, which is characterized by elevation of the scapula and a corresponding S-shaped spine, limits lung expansion within the thorax.

20. The nurse is caring for a patient who is experiencing mild shortness of breath during the immediate postoperative period, with oxygen saturation readings between 89% and 91%. What method of oxygen delivery is most appropriate for the patients needs? A) Non-rebreathing mask B) Nasal cannula C) Simple mask D) Partial-rebreathing mask

Ans: B Feedback: A nasal cannula is used when the patient requires a low to medium concentration of oxygen for which precise accuracy is not essential. The Venturi mask is used primarily for patients with COPD because it can accurately provide an appropriate level of supplemental oxygen, thus avoiding the risk of suppressing the hypoxic drive. The patients respiratory status does not require a partial- or non- rebreathing mask.

3. What would the critical care nurse recognize as a condition that may indicate a patients need to have a Test Bank - Brunner & Suddarth's Textbook of Medical-Surgical Nursing 14e (Hinkle 2017) 417 tracheostomy? A) A patient has a respiratory rate of 10 breaths per minute. B) A patient requires permanent ventilation. C) A patient exhibits symptoms of dyspnea. D) A patient has respiratory acidosis.

Ans: B Feedback: A tracheostomy permits long-term use of mechanical ventilation to prevent aspiration of oral and gastric secretions in the unconscious or paralyzed patient. Indications for a tracheostomy do not include a respiratory rate of 10 breaths per minute, symptoms of dyspnea, or respiratory acidosis.

The OR will be caring for a patient who will receive a transsacral block. For what patient would the use of a transsacral block be appropriate for pain control? A) A middle-aged man who is scheduled for a thoracotomy B) An older adult man who will undergo an inguinal hernia repair C) A 50-year-old woman who will be having a reduction mammoplasty D) A child who requires closed reduction of a right humerus fracture

Ans: B Feedback: A transsacral block produces anesthesia for the perineum and lower abdomen. Both a thoracotomy and breast reduction are in the chest region, and a transsacral block would not provide pain control for these procedures. A closed reduction of a right humerus is a procedure on the right arm, and a transsacral block would not provide pain control.

27. The nurse is performing a respiratory assessment of a patient who has been experiencing episodes of hypoxia. The nurse is aware that this is ultimately attributable to impaired gas exchange. On what factor does adequate gas exchange primarily depend? A) An appropriate perfusiondiffusion ratio B) An adequate ventilationperfusion ratio C) Adequate diffusion of gas in shunted blood D) Appropriate blood nitrogen concentration

Ans: B Feedback: Adequate gas exchange depends on an adequate ventilationperfusion ratio. There is no perfusiondiffusion ratio. Adequate gas exchange does not depend on the diffusion of gas in shunted blood or a particular concentration of nitrogen.

The nurse is performing a preoperative assessment on a patient going to surgery. The patient informs the nurse that he drinks approximately two bottles of wine each day and has for the last several years. What postoperative difficulties can the nurse anticipate for this patient? A) Alcohol withdrawal syndrome immediately following surgery B) Alcohol withdrawal syndrome 2 to 4 days after his last alcohol drink C) Alcohol withdrawal syndrome upon administration of general anesthesia D) Alcohol withdrawal syndrome 1 week after his last alcohol drink

Ans: B Feedback: Alcohol withdrawal syndrome may be anticipated between 48 and 96 hours after alcohol withdrawal and is associated with a significant mortality rate when it occurs postoperatively.

10. A patient with a decreased level of consciousness is in a recumbent position. How should the nurse best assess the lung fields for a patient in this position? A) Inform that physician that the patient is in a recumbent position and anticipate an order for a portable chest x-ray. B) Turn the patient to enable assessment of all the patients lung fields. C) Avoid turning the patient, and assess the accessible breath sounds from the anterior chest wall. D) Obtain a pulse oximetry reading, and, if the reading is low, reposition the patient and auscultate breath sounds.

Ans: B Feedback: Assessment of the anterior and posterior lung fields is part of the nurses routine evaluation. If the patient is recumbent, it is essential to turn the patient to assess all lung fields so that dependent areas can be assessed for breath sounds, including the presence of normal breath sounds and adventitious sounds. Failure to examine the dependent areas of the lungs can result in missing significant findings. This makes the other given options unacceptable.

21. The nurse is performing the health interview of a patient with chronic rhinosinusitis who experiences frequent nose bleeds. The nurse asks the patient about her current medication regimen. Which medication would put the patient at a higher risk for recurrent epistaxis? A) Afrin B) Beconase C) Sinustop Pro D) Singulair

Ans: B Feedback: Beconase should be avoided in patients with recurrent epistaxis, glaucoma, and cataracts. Sinustop Pro and Afrin are pseudoephedrine and do not have a side effect of epistaxis. Singulair is a bronchodilator and does not have epistaxis as a side effect.

27. The nurse is caring for a client with an endotracheal tube who is on a ventilator. When assessing the client, the nurse knows to maintain what cuff pressure to maintain appropriate pressure on the tracheal wall? A) Between 10 and 15 mm Hg B) Between 15 and 20 mm Hg Test Bank - Brunner & Suddarth's Textbook of Medical-Surgical Nursing 14e (Hinkle 2017) 428 C) Between 20 and 25 mm Hg D) Between 25 and 30 mm Hg

Ans: B Feedback: Complications can occur from pressure exerted by the cuff on the tracheal wall. Cuff pressures should be maintained between 15 and 20 mm Hg.

13. The nurse is conducting a presurgical interview for a patient with laryngeal cancer. The patient states that he drinks approximately six to eight shots of vodka per day. It is imperative that the nurse inform the surgical team so the patient can be assessed for what? A) Increased risk for infection B) Delirium tremens C) Depression D) Nonadherence to postoperative care

Ans: B Feedback: Considering the known risk factors for cancer of the larynx, it is essential to assess the patients history of alcohol intake. Infection is a risk in the postoperative period, but not an appropriate answer based on the patients history. Depression and nonadherence are risks in the postoperative phase, but would not be critical short-term assessments.

The nurse recognizes that aspiration is a potential complication of a laryngectomy. How should the nurse best manage this risk? A) Facilitate total parenteral nutrition (TPN). B) Keep a complete suction setup at the bedside. C) Feed the patient several small meals daily. D) Refer the patient for occupational therapy.

Ans: B Feedback: Due to the risk for aspiration, the nurse keeps a suction setup available in the hospital and instructs the family to do so at home for use if needed. TPN is not indicated and small meals do not necessarily reduce the risk of aspiration. Physical therapists do not address swallowing ability.

17. The perioperative nurse has admitted a patient who has just underwent a tonsillectomy. The nurses postoperative assessment should prioritize which of the following potential complications of this surgery? A) Difficulty ambulating B) Hemorrhage C) Infrequent swallowing D) Bradycardia

Ans: B Feedback: Hemorrhage is a potential complication of a tonsillectomy. Increased pulse, fever, and restlessness may indicate a postoperative hemorrhage. Difficulty ambulating and bradycardia are not common complications in a patient after a tonsillectomy. Infrequent swallowing does not indicate hemorrhage; frequent swallowing does.

33. A patient is being admitted to the preoperative holding area for a thoracotomy. Preoperative teaching includes what? A) Correct use of a ventilator B) Correct use of incentive spirometry C) Correct use of a mini-nebulizer D) Correct technique for rhythmic breathing

Ans: B Feedback: Instruction in the use of incentive spirometry begins before surgery to familiarize the patient with its correct use. You do not teach a patient the use of a ventilator; you explain that he may be on a ventilator to help him breathe. Rhythmic breathing and mini-nebulizers are unnecessary.

15. While caring for a patient with an endotracheal tube, the nurses recognizes that suctioning is required how often? A) Every 2 hours when the patient is awake B) When adventitious breath sounds are auscultated C) When there is a need to prevent the patient from coughing D) When the nurse needs to stimulate the cough reflex

Ans: B Feedback: It is usually necessary to suction the patients secretions because of the decreased effectiveness of the cough mechanism. Tracheal suctioning is performed when adventitious breath sounds are detected or whenever secretions are present. Unnecessary suctioning, such as scheduling every 2 hours, can initiate bronchospasm and cause trauma to the tracheal mucosa.

13. The physician has ordered continuous positive airway pressure (CPAP) with the delivery of a patients high-flow oxygen therapy. The patient asks the nurse what the benefit of CPAP is. What would be the nurses best response? A) CPAP allows a higher percentage of oxygen to be safely used. B) CPAP allows a lower percentage of oxygen to be used with a similar effect. C) CPAP allows for greater humidification of the oxygen that is administered. D) CPAP allows for the elimination of bacterial growth in oxygen delivery systems.

Ans: B Feedback: Prevention of oxygen toxicity is achieved by using oxygen only as prescribed. Often, positive end- expiratory pressure (PEEP) or CPAP is used with oxygen therapy to reverse or prevent microatelectasis, thus allowing a lower percentage of oxygen to be used. Oxygen is moistened by passing through a humidification system. Changing the tubing on the oxygen therapy equipment is the best technique for controlling bacterial growth.

30. A gerontologic nurse is analyzing the data from a patients focused respiratory assessment. The nurse is aware that the amount of respiratory dead space increases with age. What is the effect of this physiological change? A) Increased diffusion of gases B) Decreased diffusion capacity for oxygen C) Decreased shunting of blood D) Increased ventilation

Ans: B Feedback: The amount of respiratory dead space increases with age. Combined with other changes, this results in a decreased diffusion capacity for oxygen with increasing age, producing lower oxygen levels in the arterial circulation. Decreased shunting and increased ventilation do not occur with age.

The nurse is doing discharge teaching in the ED with a patient who had a nosebleed. What should the nurse include in the discharge teaching of this patient? A) Avoid blowing the nose for the next 45 minutes. B) In case of recurrence, apply direct pressure for 15 minutes. C) Do not take aspirin for the next 2 weeks. D) Seek immediate medical attention if the nosebleed recurs.

Ans: B Feedback: The nurse explains how to apply direct pressure to the nose with the thumb and the index finger for 15 minutes in case of a recurrent nosebleed. If recurrent bleeding cannot be stopped, the patient is instructed to seek additional medical attention. ASA is not contraindicated in most cases and the patient should avoiding blowing the nose for an extended period of time, not just 45 minutes. 31. The nurse recognizes that aspiration is a potential complication of a laryngectomy. How sh

20. While assessing a patient who has pneumonia, the nurse has the patient repeat the letter E while the nurses auscultates. The nurse notes that the patients voice sounds are distorted and that the letter A is audible instead of the letter E. How should this finding be documented? A) Bronchophony B) Egophony Test Bank - Brunner & Suddarth's Textbook of Medical-Surgical Nursing 14e (Hinkle 2017) 406 C) Whispered pectoriloquy D) Sonorous wheezes

Ans: B Feedback: This finding would be documented as egophony, which can be best assessed by instructing the patient to repeat the letter E. The distortion produced by consolidation transforms the sound into a clearly heard A rather than E. Bronchophony describes vocal resonance that is more intense and clearer than normal. Whispered pectoriloquy is a very subtle finding that is heard only in the presence of rather dense consolidation of the lungs. Sound is so enhanced by the consolidated tissue that even whispered words are heard. Sonorous wheezes are not defined as a voice sound, but rather as a breath sound.

24. A patient has been diagnosed with pulmonary hypertension, in which the capillaries in the alveoli are squeezed excessively. The nurse should recognize a disturbance in what aspect of normal respiratory function? A) Acidbase balance B) Perfusion C) Diffusion D) Ventilation

Ans: B Test Bank - Brunner & Suddarth's Textbook of Medical-Surgical Nursing 14e (Hinkle 2017) 408 Feedback: Perfusion is influenced by alveolar pressure. The pulmonary capillaries are sandwiched between adjacent alveoli and, if the alveolar pressure is sufficiently high, the capillaries are squeezed. This does not constitute a disturbance in ventilation (air movement), diffusion (gas exchange), or acidbase balance.

4. An elderly patient is admitted to your unit with a diagnosis of community-acquired pneumonia. During admission the patient states, I have a living will. What implication of this should the nurse recognize? A) This document is always honored, regardless of circumstances. B) This document specifies the patients wishes before hospitalization. C) This document that is binding for the duration of the patients life. D) This document has been drawn up by the patients family to determine DNR status.

Ans: B Feedback: A living will is one type of advance directive. In most situations, living wills are limited to situations in which the patients medical condition is deemed terminal. The other answers are incorrect because living wills are not always honored, they are not binding for the duration of the patients life, and they are not drawn up by the patients family. Test Bank - Brunner & Suddarth's Textbook of Medical-Surgical Nursing 14e (Hinkle 2017) 43

15. A medical nurse is caring for a patient who is palliative following metastasis. The nurse is aware of the need to uphold the ethical principle of beneficence. How can the nurse best exemplify this principle in the care of this patient? A) The nurse tactfully regulates the number and timing of visitors as per the patients wishes. B) The nurse stays with the patient during his or her death. C) The nurse ensures that all members of the care team are aware of the patients DNR order. D) The nurse liaises with members of the care team to ensure continuity of care.

Ans: B Feedback: Beneficence is the duty to do good and the active promotion of benevolent acts. Enacting the patients wishes around visitors is an example of this. Each of the other nursing actions is consistent with ethical practice, but none directly exemplifies the principle of beneficence. Test Bank - Brunner & Suddarth's Textbook of Medical-Surgical Nursing 14e (Hinkle 2017) 48

13. Nurses have different educational backgrounds and function under many titles in their practice setting. If a nurse practicing in an oncology clinic had the goal of improving patient outcomes and nursing care by influencing the patient, the nurse, and the health care system, what would most accurately describe this nurses title? A) Nursing care expert B) Clinical nurse specialist C) Nurse manager D) Staff nurse

Ans: B Feedback: Clinical nurse specialists are prepared as specialists who practice within a circumscribed area of care (e.g., cardiovascular, oncology). They define their roles as having five major components: clinical practice, education, management, consultation, and research. The other answers are incorrect because they are not the most accurate titles for this nurse.

34. A nurse is unsure how best to respond to a patients vague complaint of feeling off. The nurse is attempting to apply the principles of critical thinking, including metacognition. How can the nurse best foster metacognition? A) By eliciting input from a variety of trusted colleagues B) By examining the way that she thinks and applies reason C) By evaluating her responses to similar situations in the past D) By thinking about the way that an ideal nurse would respond in this situation

Ans: B Feedback: Critical thinking includes metacognition, the examination of ones own reasoning or thought processes, to help refine thinking skills. Metacognition is not characterized by eliciting input from others or evaluating previous responses.

19. A nurse is providing care for a patient who is postoperative day one following a bowel resection for the treatment of colorectal cancer. How can the nurse best exemplify the QSEN competency of quality improvement? A) By liaising with the members of the interdisciplinary care team B) By critically appraising the outcomes of care that is provided C) By integrating the patients preferences into the plan of care D) By documenting care in the electronic health record in a timely fashion

Ans: B Feedback: Evaluation of outcomes is central to the QSEN competency of quality improvements. Each of the other listed activities is a component of quality nursing care, but none clearly exemplifies quality improvement activities.

10. The nurse is caring for a patient who is withdrawing from heavy alcohol use and who is consequently combative and confused, despite the administration of benzodiazepines. The patient has a fractured hip that he suffered in a traumatic accident and is trying to get out of bed. What is the most appropriate action for the nurse to take? A) Leave the patient and get help. B) Obtain a physicians order to restrain the patient. C) Read the facilitys policy on restraints. D) Order soft restraints from the storeroom.

Ans: B Feedback: It is mandatory in most settings to have a physicians order before restraining a patient. Before restraints are used, other strategies, such as asking family members to sit with the patient, or utilizing a specially trained sitter, should be tried. A patient should never be left alone while the nurse summons assistance.

39. Leadership of a medical unit have been instructed to integrate the principles of the Quality and Safety Education for Nurses (QSEN) competency of quality improvement. What action should the units leaders take? A) Provide access to online journals and Web-based clinical resources for nursing staff. B) Use flow charts to document the processes of care that are used on the unit. C) Enforce continuing education requirements for all care providers. D) Reduce the use of chemical and physical restraints on the unit.

Ans: B Feedback: One of the quality improvement skills is to use tools, such as flow charts and cause-effect diagrams, to make processes of care explicit. Each of the other listed actions has the potential to benefit patients and care givers, but none is an explicit knowledge, skill, or attitude associated with this QSEN competency.

28. With the changing population of health care consumers, it has become necessary for nurses to work more closely with other nurses, as when acute care nurses collaborate with public health and home health nurses. What nursing function has increased in importance because of this phenomenon? A) Prescribing medication B) Performing discharge planning C) Promoting family involvement D) Forming collegial relationships

Ans: B Feedback: The importance of effective discharge planning and quality improvement cannot be overstated. The other answers are incorrect because giving medication and family involvement in the patients care have not grown in importance. Making and maintaining collegial relationships has become a necessity in working in the health care delivery system. Effective discharge planning aids in getting patients out of the inpatient setting sooner, cutting costs, and making rehabilitation in the community and home setting possible.

3. An adult patient has requested a do not resuscitate (DNR) order in light of his recent diagnosis with late stage pancreatic cancer. The patients son and daughter-in-law are strongly opposed to the patients request. What is the primary responsibility of the nurse in this situation? A) Perform a slow code until a decision is made. B) Honor the request of the patient. C) Contact a social worker or mediator to intervene. D) Temporarily withhold nursing care until the physician talks to the family.

Ans: B Feedback: The nurse must honor the patients wishes and continue to provide required nursing care. Discussing the matter with the physician may lead to further communication with the family, during which the family may reconsider their decision. It is not normally appropriate for the nurse to seek the assistance of a social worker or mediator. A slow code is considered unethical.

1. A nurse has been offered a position on an obstetric unit and has learned that the unit offers therapeutic abortions, a procedure which contradicts the nurses personal beliefs. What is the nurses ethical obligation to these patients? A) The nurse should adhere to professional standards of practice and offer service to these patients. B) The nurse should make the choice to decline this position and pursue a different nursing role. C) The nurse should decline to care for the patients considering abortion. D) The nurse should express alternatives to women considering terminating their pregnancy.

Ans: B Feedback: To avoid facing ethical dilemmas, nurses can follow certain strategies. For example, when applying for a job, a nurse should ask questions regarding the patient population. If a nurse is uncomfortable with a particular situation, then not accepting the position would be the best option. The nurse is only required by law (and practice standards) to provide care to the patients the clinic accepts; the nurse may not discriminate between patients and the nurse expressing his or her own opinion and providing another option is inappropriate.

A patient is scheduled for diagnostic testing to address prolonged signs and symptoms of genitourinary dysfunction. What signs and symptoms are particularly suggestive of urinary tract disease? Select all that apply. A) Petechiae B) Pain C) Gastrointestinal symptoms D) Changes in voiding E) Jaundice

Ans: B, C, D Feedback: Dysfunction of the kidney can produce a complex array of symptoms throughout the body. Pain, changes in voiding, and gastrointestinal symptoms are particularly suggestive of urinary tract disease. Jaundice and petechiae are not associated with genitourinary health problems.

35. The nursing instructor cites a list of skills that support critical thinking in clinical situations. The nurse should describe skills in which of the following domains? Select all that apply. A) Self-esteem B) Self-regulation C) Inference D) Autonomy E) Interpretation

Ans: B, C, E Feedback: Skills needed in critical thinking include interpretation, analysis, evaluation, inference, explanation, and self-regulation. Self-esteem and autonomy would not be on the list because they are not skills.

9. The critical care nurse is precepting a new nurse on the unit. Together they are caring for a patient who has a tracheostomy tube and is receiving mechanical ventilation. What action should the critical care nurse recommend when caring for the cuff? A) Deflate the cuff overnight to prevent tracheal tissue trauma. B) Inflate the cuff to the highest possible pressure in order to prevent aspiration. C) Monitor the pressure in the cuff at least every 8 hours D) Keep the tracheostomy tube plugged at all times.

Ans: C Feedback: Cuff pressure must be monitored by the respiratory therapist or nurse at least every 8 hours by attaching a handheld pressure gauge to the pilot balloon of the tube or by using the minimal leak volume or minimal occlusion volume technique. Plugging is only used when weaning the patient from tracheal support. Deflating the cuff overnight would be unsafe and inappropriate. High cuff pressure can cause tissue trauma.

It is cold season and the school nurse been asked to provide an educational event for the parent teacher organization of the local elementary school. What should the nurse include in teaching about the treatment of pharyngitis? A) Pharyngitis is more common in children whose immunizations are not up to date. B) There are no effective, evidence-based treatments for pharyngitis. C) Use of warm saline gargles or throat irrigations can relieve symptoms. D) Heat may increase the spasms in pharyngeal muscles.

Ans: C Feedback: Depending on the severity of the pharyngitis and the degree of pain, warm saline gargles or throat irrigations are used. The benefits of this treatment depend on the degree of heat that is applied. The nurse teaches about these procedures and about the recommended temperature of the solution: high enough to be effective and as warm as the patient can tolerate, usually 105F to 110F (40.6C to 43.3C). Irrigating the throat may reduce spasm in the pharyngeal muscles and relieve soreness of the

12. The occupational health nurse is obtaining a patient history during a pre-employment physical. During the history, the patient states that he has hereditary angioedema. The nurse should identify what implication of this health condition? Test Bank - Brunner & Suddarth's Textbook of Medical-Surgical Nursing 14e (Hinkle 2017) 440 A) It will result in increased loss of work days. B) It may cause episodes of weakness due to reduced cardiac output. C) It can cause life-threatening airway obstruction. D) It is unlikely to interfere with the individuals health.

Ans: C Feedback: Hereditary angioedema is an inherited condition that is characterized by episodes of life-threatening laryngeal edema. No information supports lost days of work or reduced cardiac function.

36. The nurse is performing nasotracheal suctioning on a medical patient and obtains copious amounts of secretions from the patients airway, even after inserting and withdrawing the catheter several times. How should the nurse proceed? A) Continue suctioning the patient until no more secretions are obtained. B) Perform chest physiotherapy rather than nasotracheal suctioning. Test Bank - Brunner & Suddarth's Textbook of Medical-Surgical Nursing 14e (Hinkle 2017) 432 C) Wait several minutes and then repeat suctioning. D) Perform postural drainage and then repeat suctioning.

Ans: C Feedback: If additional suctioning is needed, the nurse should withdraw the catheter to the back of the pharynx, reassure the patient, and oxygenate for several minutes before resuming suctioning. Chest physiotherapy and postural drainage are not necessarily indicated.

3. The nurse is creating a plan of car for a patient diagnosed with acute laryngitis. What intervention should Test Bank - Brunner & Suddarth's Textbook of Medical-Surgical Nursing 14e (Hinkle 2017) 436 be included in the patients plan of care? A) Place warm cloths on the patients throat, as needed. B) Have the patient inhale warm steam three times daily. C) Encourage the patient to limit speech whenever possible. D) Limit the patients fluid intake to 1.5 L/day.

Ans: C Feedback: Management of acute laryngitis includes resting the voice, avoiding irritants (including smoking), resting, and inhaling cool steam or an aerosol. Fluid intake should be increased. Warm cloths on the throat will not help relieve the symptoms of acute laryngitis.

The perioperative nurse is constantly assessing the surgical patient for signs and symptoms of complications of surgery. Which symptom should first signal to the nurse the possibility that the patient is developing malignant hyperthermia? A) Increased temperature B) Oliguria C) Tachycardia D) Hypotension Test Bank - Brunner & Suddarth's Textbook of Medical-Surgical Nursing 14e (Hinkle 2017) 372

Ans: C Feedback: The initial symptoms of malignant hyperthermia are related to cardiovascular and musculoskeletal activity. Tachycardia (heart rate greater than 150 beats per minute) is often the earliest sign. Oliguria, hypotension, and increased temperature are later signs of malignant hyperthermia.

18. While assessing an acutely ill patients respiratory rate, the nurse assesses four normal breaths followed by an episode of apnea lasting 20 seconds. How should the nurse document this finding? A) Eupnea Test Bank - Brunner & Suddarth's Textbook of Medical-Surgical Nursing 14e (Hinkle 2017) 405 B) Apnea C) Biots respiration D) Cheyne-Stokes

Ans: C Feedback: The nurse will document that the patient is demonstrating a Biots respiration pattern. Biots respiration is characterized by periods of normal breathing (three to four breaths) followed by varying periods of apnea (usually 10 seconds to 1 minute). Cheyne-Stokes is a similar respiratory pattern, but it involves a regular cycle where the rate and depth of breathing increase and then decrease until apnea occurs. Biots respiration is not characterized by the increase and decrease in the rate and depth, as characterized by Cheyne-Stokes. Eupnea is a normal breathing pattern of 12 to 18 breaths per minute. Bradypnea is a slower-than-normal rate (<10 breaths per minute), with normal depth and regular rhythm, and no apnea.

14. The home care nurse is assessing a patient who requires home oxygen therapy. What criterion indicates that an oxygen concentrator will best meet the needs of the patient in the home environment? A) The patient desires a low-maintenance oxygen delivery system that delivers oxygen flow rates up to 6 L/min. B) The patient requires a high-flow system for use with a tracheostomy collar. C) The patient desires a portable oxygen delivery system that can deliver 2 L/min. D) The patients respiratory status requires a system that provides an FiO2 of 65%.

Ans: C Feedback: The use of oxygen concentrators is another means of providing varying amounts of oxygen, especially in the home setting. They can deliver oxygen flows from 1 to 10 L/min and provide an FiO2 of about 40%. They require regular maintenance and are not used for high-flow applications. The patient desiring a portable oxygen delivery system of 2L/min will benefit from the use of an oxygen concentrator.

7. A patient with chronic lung disease is undergoing lung function testing. What test result denotes the volume of air inspired and expired with a normal breath? A) Total lung capacity B) Forced vital capacity C) Tidal volume Test Bank - Brunner & Suddarth's Textbook of Medical-Surgical Nursing 14e (Hinkle 2017) 400 D) Residual volume

Ans: C Feedback: Tidal volume refers to the volume of air inspired and expired with a normal breath. Total lung capacity is the maximal amount of air the lungs and respiratory passages can hold after a forced inspiration. Forced vital capacity is vital capacity performed with a maximally forced expiration. Residual volume is the maximal amount of air left in the lung after a maximal expiration.

22. The nurse is preparing to suction a patient with an endotracheal tube. What should be the nurses first step in the suctioning process? A) Explain the suctioning procedure to the patient and reposition the patient. B) Turn on suction source at a pressure not exceeding 120 mm Hg. C) Assess the patients lung sounds and SAO2 via pulse oximeter. D) Perform hand hygiene and don nonsterile gloves, goggles, gown, and mask.

Ans: C Feedback:Assessment data indicate the need for suctioning and allow the nurse to monitor the effect of suction on the patients level of oxygenation. Explaining the procedure would be the second step; performing hand hygiene is the third step, and turning on the suction source is the fourth step.

The nurse is caring for a patient who is admitted to the ER with the diagnosis of acute appendicitis. The nurse notes during the assessment that the patients ribs and xiphoid process are prominent. The patient states she exercises two to three times daily and her mother indicates that she is being treated for anorexia nervosa. How should the nurse best follow up these assessment data? A) Inform the postoperative team about the patients risk for wound dehiscence. B) Evaluate the patients ability to manage her pain level. C) Facilitate a detailed analysis of the patients electrolyte levels. D) Instruct the patient on the need for a high-sodium diet to promote healing.

Ans: C Feedback:The surgical team should be informed about the patients medical history regarding anorexia nervosa. Any nutritional deficiency, such as malnutrition, should be corrected before surgery to provide adequate protein for tissue repair. The electrolyte levels should be evaluated and corrected to prevent metabolic abnormalities in the operative and postoperative phase. The risk of wound dehiscence is more likely associated with obesity. Instruction on proper nutrition should take place in the postoperative period, and a consultation should be made with her psychiatric specialist. Evaluation of pain management is always important, but not particularly significant in this scenario.

9. A patient asks the nurse why an infection in his upper respiratory system is affecting the clarity of his speech. Which structure serves as the patients resonating chamber in speech? A) Trachea B) Pharynx C) Paranasal sinuses D) Larynx

Ans: C Test Bank - Brunner & Suddarth's Textbook of Medical-Surgical Nursing 14e (Hinkle 2017) 401 Feedback: A prominent function of the sinuses is to serve as a resonating chamber in speech. The trachea, also known as the windpipe, serves as the passage between the larynx and the bronchi. The pharynx is a tubelike structure that connects the nasal and oral cavities to the larynx. The pharynx also functions as a passage for the respiratory and digestive tracts. The major function of the larynx is vocalization through the function of the vocal cords. The vocal cords are ligaments controlled by muscular movements that produce sound.

11. A patient is undergoing testing to see if he has a pleural effusion. Which of the nurses respiratory assessment findings would be most consistent with this diagnosis? A) Increased tactile fremitus, egophony, and a dull sound upon percussion of the chest wall B) Decreased tactile fremitus, wheezing, and a hyperresonant sound upon percussion of the chest wall C) Lung fields dull to percussion, absent breath sounds, and a pleural friction rub D) Normal tactile fremitus, decreased breath sounds, and a resonant sound upon percussion of the chest wall

Ans: C Test Bank - Brunner & Suddarth's Textbook of Medical-Surgical Nursing 14e (Hinkle 2017) 402 Feedback: Assessment findings consistent with a pleural effusion include affected lung fields being dull to percussion and absence of breath sounds. A pleural friction rub may also be present. The other listed signs are not typically associated with a pleural effusion.

2. A terminally ill patient you are caring for is complaining of pain. The physician has ordered a large dose of intravenous opioids by continuous infusion. You know that one of the adverse effects of this medicine is respiratory depression. When you assess your patients respiratory status, you find that the rate has decreased from 16 breaths per minute to 10 breaths per minute. What action should you take? A) Decrease the rate of IV infusion. B) Stimulate the patient in order to increase respiratory rate. C) Report the decreased respiratory rate to the physician. D) Allow the patient to rest comfortably.

Ans: C Feedback: End-of life issues that often involve ethical dilemmas include pain control, do not resuscitate orders, life- Test Bank - Brunner & Suddarth's Textbook of Medical-Surgical Nursing 14e (Hinkle 2017) 42 support measures, and administration of food and fluids. The risk of respiratory depression is not the intent of the action of pain control. Respiratory depression should not be used as an excuse to withhold pain medication for a terminally ill patient. The patients respiratory status should be carefully monitored and any changes should be reported to the physician.

8. Nurses in acute care settings must work with other health care team members to maintain quality care while facing pressures to care for patients who are hospitalized for shorter periods of time than in the past. To ensure positive health outcomes when patients return to their homes, what action should the nurse prioritize? A) Promotion of health literacy during hospitalization B) Close communication with insurers C) Thorough and evidence-based discharge planning D) Participation in continuing education initiatives

Ans: C Feedback: Following discharges that occur after increasingly short hospital stays, nurses in the community care for patients who need high-technology acute care services as well as long-term care in the home. This is dependent on effective discharge planning to a greater degree than continuing education, communication with insurers, or promotion of health literacy.

16. A nurse is planning a medical patients care with consideration of Maslows hierarchy of needs. Within this framework of understanding, what would be the nurses first priority? A) Allowing the family to see a newly admitted patient B) Ambulating the patient in the hallway C) Administering pain medication Test Bank - Brunner & Suddarth's Textbook of Medical-Surgical Nursing 14e (Hinkle 2017) 10 D) Teaching the patient to self-administer insulin safely

Ans: C Feedback: In Maslows hierarchy of needs, pain relief addresses the patients basic physiologic need. Activity, such as ambulation, is a higher level need above the physiologic need. Allowing the patient to see family addresses a higher level need related to love and belonging. Teaching the patient is also a higher level need related to the desire to know and understand and is not appropriate at this time, as the basic physiologic need of pain control must be addressed before the patient can address these higher level needs.

15. A hospice nurse is caring for a patient who is dying of lymphoma. According to Maslows hierarchy of needs, what dimension of care should the nurse consider primary in importance when caring for a dying patient? A) Spiritual B) Social C) Physiologic D) Emotional

Ans: C Feedback: Maslow ranked human needs as follows: physiologic needs; safety and security; sense of belonging and affection; esteem and self-respect; and self-actualization, which includes self-fulfillment, desire to know and understand, and aesthetic needs. Such a hierarchy of needs is a useful framework that can be applied to the various nursing models for assessment of a patients strengths, limitations, and need for nursing interventions. The other answers are incorrect because they are not of primary importance when caring for a dying patient, though each should certainly be addressed.

17. A medical-surgical nurse is aware of the scope of practice as defined in the state where the nurse provides care. This nurses compliance with the nurse practice act demonstrates adherence to which of the following? A) National Council of Nursings guidelines for care B) National League for Nursings Code of Conduct C) American Nurses Associations Social Policy Statement D) Department of Health and Human Services White Paper on Nursing

Ans: C Feedback: Nurses have a responsibility to carry out their role as described in the Social Policy Statement to comply with the nurse practice act of the state in which they practice and to comply with the Code of Ethics for Nurses as spelled out by the ANA (2001) and the International Council of Nurses (International Council of Nurses [ICN], 2006). The other answers are incorrect; the Code of Ethics for nursing is not included in the ANAs white paper. The DHHS has not published a white paper on nursing nor has the NLN published a specific code of conduct.

44. Your patient has been admitted for a liver biopsy because the physician believes the patient may have liver cancer. The family has told both you and the physician that if the patient is terminal, the family does not want the patient to know. The biopsy results are positive for an aggressive form of liver cancer and the patient asks you repeatedly what the results of the biopsy show. What strategy can you use to give ethical care to this patient? A) Obtain the results of the biopsy and provide them to the patient. Test Bank - Brunner & Suddarth's Textbook of Medical-Surgical Nursing 14e (Hinkle 2017) 61 B) Tell the patient that only the physician knows the results of the biopsy. C) Promptly communicate the patients request for information to the family and the physician. D) Tell the patient that the biopsy results are not back yet in order temporarily to appease him.

Ans: C Feedback: Strategies nurses could consider include the following: not lying to the patient, providing all information related to nursing procedures and diagnoses, and communicating the patients requests for information to the family and physician. Ethically, you cannot tell the patient the results of the biopsy and you cannot lie to the patient.

30. A patient with migraines does not know whether she is receiving a placebo for pain management or the new drug that is undergoing clinical trials. Upon discussing the patients distress, it becomes evident to the nurse that the patient did not fully understand the informed consent document that she signed. Which ethical principle is most likely involved in this situation? A) Sanctity of life B) Confidentiality C) Veracity D) Fidelity

Ans: C Feedback: Telling the truth (veracity) is one of the basic principles of our culture. Three ethical dilemmas in clinical practice that can directly conflict with this principle are the use of placebos (nonactive substances used for treatment), not revealing a diagnosis to a patient, and revealing a diagnosis to persons other than the patient with the diagnosis. All involve the issue of trust, which is an essential element in the nursepatient relationship. Sanctity of life is the perspective that life is the highest good. Confidentiality deals with privacy of the patient. Fidelity is promise-keeping and the duty to be faithful to ones commitments.

20. Professional nursing expands and grows because of factors driven by the changing needs of health care consumers. Which of the following is a factor that nurses should reflect in the planning and provision of health care? A) Decreased access to health care information by individuals B) Gradual increases in the cultural unity of the American population C) Increasing mean and median age of the American population D) Decreasing consumer expectations related to health care outcomes

Ans: C Feedback: Test Bank - Brunner & Suddarth's Textbook of Medical-Surgical Nursing 14e (Hinkle 2017) 12 The decline in birth rate and the increase in lifespan due to improved health care have resulted in fewer school-age children and more senior citizens, many of whom are women. The population has become more culturally diverse as increasing numbers of people from different national backgrounds enter the country. Access to information and consumer expectations continue to increase.

40. The IOM Report Health Professions Education: A Bridge to Quality issued a number of challenges to the educational programs that teach nurses and members of other health professions. According to this report, what activity should educational institutions prioritize? A) More clearly delineate each professions scope of practice during education B) Move toward developing a single health curriculum that can be adapted for any health profession C) Include interdisciplinary core competencies into curricula D) Elicit input from patients and families into health care curricula

Ans: C Feedback: Test Bank - Brunner & Suddarth's Textbook of Medical-Surgical Nursing 14e (Hinkle 2017) 21 Health Professions Education: A Bridge to Quality challenged health professions education programs to integrate interdisciplinary core competencies into their respective curricula to include patient-centered care, interdisciplinary teamwork and collaboration, evidence-based practice, quality improvement, safety, and informatics. This report did not specify clearer definitions of scope of practice, patient input, or a single curriculum. Test Bank - Brunner & Suddarth's Textbook of Medical-Surgical Nursing 14e (Hinkle 2017) 22

40. A class of nursing students is in their first semester of nursing school. The instructor explains that one of the changes they will undergo while in nursing school is learning to think like a nurse. What is the most current model of this thinking process? A) Critical-thinking Model B) Nursing Process Model C) Clinical Judgment Model D) Active Practice Model

Ans: C Feedback: To depict the process of thinking like a nurse, Tanner (2006) developed a model known as the clinical judgment model.

41. Critical thinking and decision-making skills are essential parts of nursing in all venues. What are examples of the use of critical thinking in the venue of genetics-related nursing? Select all that apply. A) Notifying individuals and family members of the results of genetic testing B) Providing a written report on genetic testing to an insurance company C) Assessing and analyzing family history data for genetic risk factors D) Identifying individuals and families in need of referral for genetic testing E) Ensuring privacy and confidentiality of genetic information

Ans: C, D, E Feedback: Nurses use critical thinking and decision-making skills in providing genetics-related nursing care when they assess and analyze family history data for genetic risk factors, identify those individuals and families in need of referral for genetic testing or counseling, and ensure the privacy and confidentiality of genetic information. Nurses who work in the venue of genetics-related nursing do not notify family members of the results of an individuals genetic testing, and they do not provide written reports to insurance companies concerning the results of genetic testing.

20. The nurse is caring for a patient who is postoperative day 2 following a total laryngectomy for supraglottic cancer. The nurse should prioritize what assessment? A) Assessment of body image B) Assessment of jugular venous pressure C) Assessment of carotid pulse D) Assessment of swallowing ability

Ans: D Feedback: A common postoperative complication from this type of surgery is difficulty in swallowing, which creates a potential for aspiration. Cardiovascular complications are less likely at this stage of recovery. The patients body image should be assessed, but dysphagia has the potential to affect the patients airway, and is a consequent priority.

9. The campus nurse at a university is assessing a 21-year-old student who presents with a severe nosebleed. The site of bleeding appears to be the anterior portion of the nasal septum. The nurse instructs the student to tilt her head forward and the nurse applies pressure to the nose, but the students nose continues to bleed. Which intervention should the nurse next implement? A) Apply ice to the bridge of her nose B) Lay the patient down on a cot C) Arrange for transfer to the local ED D) Insert a tampon in the affected nare

Ans: D Feedback: A cotton tampon may be used to try to stop the bleeding. The use of ice on the bridge of the nose has no Test Bank - Brunner & Suddarth's Textbook of Medical-Surgical Nursing 14e (Hinkle 2017) 439 scientific rationale for care. Laying the client down on the cot could block the clients airway. Hospital admission is necessary only if the bleeding becomes serious.

6. A nurse is educating a patient in anticipation of a procedure that will require a water-sealed chest drainage system. What should the nurse tell the patient and the family that this drainage system is used for? A) Maintaining positive chest-wall pressure B) Monitoring pleural fluid osmolarity C) Providing positive intrathoracic pressure D) Removing excess air and fluid

Ans: D Feedback: Chest tubes and closed drainage systems are used to re-expand the lung involved and to remove excess air, fluid, and blood. They are not used to maintain positive chest-wall pressure, monitor pleural fluid, or provide positive intrathoracic pressure.

35. The nurse is caring for a patient who has been scheduled for a bronchoscopy. How should the nurse prepare the patient for this procedure? A) Administer a bolus of IV fluids. B) Arrange for the insertion of a peripherally inserted central catheter. C) Administer nebulized bronchodilators every 2 hours until the test. D) Withhold food and fluids for several hours before the test.

Ans: D Feedback: Food and fluids are withheld for 4 to 8 hours before the test to reduce the risk of aspiration when the cough reflex is blocked by anesthesia. IV fluids, bronchodilators, and a central line are unnecessary.

4. A patient comes to the ED and is admitted with epistaxis. Pressure has been applied to the patients midline septum for 10 minutes, but the bleeding continues. The nurse should anticipate using what treatment to control the bleeding? A) Irrigation with a hypertonic solution B) Nasopharyngeal suction C) Normal saline application D) Silver nitrate application

Ans: D Feedback: If pressure to the midline septum does not stop the bleeding for epistaxis, additional treatment of silver nitrate application, Gelfoam, electrocautery, or vasoconstrictors may be used. Suction may be used to visualize the nasal septum, but it does not alleviate the bleeding. Irrigation with a hypertonic solution is not used to treat epistaxis.

12. A patient recovering from thoracic surgery is on long-term mechanical ventilation and becomes very frustrated when he tries to communicate. What intervention should the nurse perform to assist the patient? A) Assure the patient that everything will be all right and that remaining calm is the best strategy. B) Ask a family member to interpret what the patient is trying to communicate. C) Ask the physician to wean the patient off the mechanical ventilator to allow the patient to speak freely. D) Express empathy and then encourage the patient to write, use a picture board, or spell words with an alphabet board.

Ans: D Feedback: If the patient uses an alternative method of communication, he will feel in better control and likely be less frustrated. Assuring the patient that everything will be all right offers false reassurance, and telling him not to be upset minimizes his feelings. Neither of these methods helps the patient to communicate. In a patient with an endotracheal or tracheostomy tube, the family members are also likely to encounter difficulty interpreting the patients wishes. Making them responsible for interpreting the patients gestures may frustrate the family. The patient may be weaned off a mechanical ventilator only when the physiologic parameters for weaning have been met.

A patients total laryngectomy has created a need for alaryngeal speech which will be achieved through the use of tracheoesophageal puncture. What action should the nurse describe to the patient when teaching him about this process? A) Training on how to perform controlled belching B) Use of an electronically enhanced artificial pharynx C) Insertion of a specialized nasogastric tube D) Fitting for a voice prosthesis

Ans: D Feedback: In patients receiving transesophageal puncture, a valve is placed in the tracheal stoma to divert air into the esophagus and out the mouth. Once the puncture is surgically created and has healed, a voice prosthesis (Blom-Singer) is fitted over the puncture site. A nasogastric tube and belching are not required. An artificial pharynx is not used.

32. The patient has just had an MRI ordered because a routine chest x-ray showed suspicious areas in the right lung. The physician suspects bronchogenic carcinoma. An MRI would most likely be order to assess for what in this patient? A) Alveolar dysfunction B) Forced vital capacity C) Tidal volume D) Chest wall invasion

Ans: D Feedback: MRI is used to characterize pulmonary nodules; to help stage bronchogenic carcinoma (assessment of chest wall invasion); and to evaluate inflammatory activity in interstitial lung disease, acute pulmonary embolism, and chronic thrombolytic pulmonary hypertension. Imaging would not focus on the alveoli since the problem in the bronchi. A static image such as MRI cannot inform PFT.

8. A patients plan of care specifies postural drainage. What action should the nurse perform when providing this noninvasive therapy? A) Administer the treatment with the patient in a high Fowlers or semi-Fowlers position. B) Perform the procedure immediately following the patients meals. C) Apply percussion firmly to bare skin to facilitate drainage. D) Assist the patient into a position that will allow gravity to move secretions.

Ans: D Feedback: Postural drainage is usually performed two to four times per day. The patient uses gravity to facilitate postural draining. The skin should be covered with a cloth or a towel during percussion to protect the skin. Postural drainage is not administered in an upright position or directly following a meal.

22. A patient is being treated for a pulmonary embolism and the medical nurse is aware that the patient suffered an acute disturbance in pulmonary perfusion. This involved an alteration in what aspect of normal physiology? A) Maintenance of constant osmotic pressure in the alveoli B) Maintenance of muscle tone in the diaphragm Test Bank - Brunner & Suddarth's Textbook of Medical-Surgical Nursing 14e (Hinkle 2017) 407 C) pH balance in the pulmonary veins and arteries D) Adequate flow of blood through the pulmonary circulation.

Ans: D Feedback: Pulmonary perfusion is the actual blood flow through the pulmonary circulation. Perfusion is not defined in terms of pH balance, muscle tone, or osmotic pressure.

8. In addition to heart rate, blood pressure, respiratory rate, and temperature, the nurse needs to assess a patients arterial oxygen saturation (SaO2 ). What procedure will best accomplish this? A) Incentive spirometry B) Arterial blood gas (ABG) measurement C) Peak flow measurement D) Pulse oximetry

Ans: D Feedback: Pulse oximetry is a noninvasive procedure in which a small sensor is positioned over a pulsating vascular bed. It can be used during transport and causes the patient no discomfort. An incentive spirometer is used to assist the patient with deep breathing after surgery. ABG measurement can measure SaO2, but this is an invasive procedure that can be painful. Some patients with asthma use peak flow meters to measure levels of expired air.

2. The nurse is caring for a patient who has just returned to the unit after a colon resection. The patient is showing signs of hypoxia. The nurse knows that this is probably caused by what? A) Nitrogen narcosis B) Infection C) Impaired diffusion D) Shunting

Ans: D Feedback: Shunting appears to be the main cause of hypoxia after thoracic or abdominal surgery and most types of respiratory failure. Impairment of normal diffusion is a less common cause. Infection would not likely be present at this early stage of recovery and nitrogen narcosis only occurs from breathing compressed air.

4. The medical nurse is creating the care plan of an adult patient requiring mechanical ventilation. What nursing action is most appropriate? A) Keep the patient in a low Fowlers position. B) Perform tracheostomy care at least once per day. C) Maintain continuous bedrest. D) Monitor cuff pressure every 8 hours.

Ans: D Feedback: The cuff pressure should be monitored every 8 hours. It is important to perform tracheostomy care at least every 8 hours because of the risk of infection. The patient should be encouraged to ambulate, if possible, and a low Fowlers position is not indicated.

18. The nurse is discussing activity management with a patient who is postoperative following thoracotomy. What instructions should the nurse give to the patient regarding activity immediately following discharge? A) Walk 1 mile 3 to 4 times a week. B) Use weights daily to increase arm strength. C) Walk on a treadmill 30 minutes daily. D) Perform shoulder exercises five times daily.

Ans: D Feedback: The nurse emphasizes the importance of progressively increased activity. The nurse also instructs the patient on the importance of performing shoulder exercises five times daily. The patient should ambulate with limits and realize that the return of strength will likely be gradual and likely will not include weight lifting or lengthy walks.

39. A nurse is teaching a patient how to perform flow type incentive spirometry prior to his scheduled thoracic surgery. What instruction should the nurse provide to the patient? A) Hold the spirometer at your lips and breathe in and out like you normally would. B) When youre ready, blow hard into the spirometer for as long as you can. C) Take a deep breath and then blow short, forceful breaths into the spirometer. D) Breathe in deeply through the spirometer, hold your breath briefly, and then exhale.

Ans: D Feedback: The patient should be taught to lace the mouthpiece of the spirometer firmly in the mouth, breathe air in through the mouth, and hold the breath at the end of inspiration for about 3 seconds. The patient should then exhale slowly through the mouthpiece.

38. The Joint Commission and the Centers for Medicare and Medicaid Services (CMS) are evaluating a large, university medical center according to core measures. Evaluators should perform this evaluation in what way? A) By auditing the medical centers electronic health records B) By performing focus groups and interviews with care providers from numerous disciplines C) By performing statistical analysis of patient satisfaction surveys D) By comparing the centers patient outcomes to best practice indicators

Ans: D Feedback: Core measures are used to gauge how well a hospital gives care to its patients who are admitted to seek treatment for a specific disease or who need a specific treatment as compared to evidence-based guidelines and standards of care. Benchmark standards of quality are used to compare the care or treatment patients receive with the best practice standards. Patient satisfaction is considered, but this is not the only criterion.

45. The nurse admits a patient to an oncology unit that is a site for a study on the efficacy of a new chemotherapeutic drug. The patient knows that placebos are going to be used for some participants in the study but does not know that he is receiving a placebo. When is it ethically acceptable to use placebos? A) Whenever the potential benefits of a study are applicable to the larger population B) When the patient is unaware of it and it is deemed unlikely that it would cause harm C) Whenever the placebo replaces an active drug D) When the patient knows placebos are being used and is involved in the decision-making process

Ans: D Feedback: Placebos may be used in experimental research in which a patient is involved in the decision-making process and is aware that placebos are being used in the treatment regimen. Placebos may not ethically be used solely when there is a potential benefit, when the patient is unaware, or when a placebo replaces an active drug.

26. A nurse has accepted a position as a clinical nurse leader (CNL), a new role that has been launched within the past decade. In this role, the nurse should prioritize which of the following activities? A) Acting as a spokesperson for the nursing profession B) Generating and disseminating new nursing knowledge C) Diagnosing and treating health problems that have a predictable course D) Helping patients to navigate the health care system

Ans: D Feedback: The CNL is a nurse generalist with a masters degree in nursing and a special background in clinical leadership, educated to help patients navigate through the complex health care system. The other answers are incorrect because they are not what nursing has identified as the CNL role.

12. During report, a nurse finds that she has been assigned to care for a patient admitted with an opportunistic infection secondary to AIDS. The nurse informs the clinical nurse leader that she is refusing to care for him because he has AIDS. The nurse has an obligation to this patient under which legal premise? A) Good Samaritan Act B) Nursing Interventions Classification (NIC) C) Patient Self-Determination Act D) ANA Code of Ethics

Ans: D Feedback: The ethical obligation to care for all patients is clearly identified in the first statement of the ANA Code of Ethics for Nurses. The Good Samaritan Act relates to lay people helping others in need. The NIC is a standardized classification of nursing treatment that includes independent and collaborative interventions. The Patient Self-Determination Act encourages people to prepare advance directives in which they indicate their wishes concerning the degree of supportive care to be provided if they become incapacitated.

6. A group of nursing students are participating in a community health clinic. When providing care in this context, what should the students teach participants about disease prevention? A) It is best achieved through attending self-help groups. B) It is best achieved by reducing psychological stress. C) It is best achieved by being an active participant in the community. D) It is best achieved by exhibiting behaviors that promote health.

Ans: D Feedback: Today, increasing emphasis is placed on health, health promotion, wellness, and self-care. Health is seen as resulting from a lifestyle oriented toward wellness. Nurses in community health clinics do not teach that disease prevention is best achieved through attending self-help groups, by reducing stress, or by being an active participant in the community, though each of these activities is consistent with a healthy lifestyle.

12. Nurses now have the option to practice in a variety of settings and one of the fastest growing venues of practice for the nurse in todays health care environment is home health care. What is the main basis for Test Bank - Brunner & Suddarth's Textbook of Medical-Surgical Nursing 14e (Hinkle 2017) 8 the growth in this health care setting? A) Chronic nursing shortage B) Western focus on treatment of disease C) Nurses preferences for day shifts instead of evening or night shifts D) Discharge of patients who are more critically ill

Ans: D Feedback: With shorter hospital stays and increased use of outpatient health care services, more nursing care is provided in the home and community setting. The other answers are incorrect because they are not the basis for the growth in nursing care delivered in the home setting.

37. A nurse is admitting a new patient to the medical unit. During the initial nursing assessment, the nurse has asked many supplementary open-ended questions while gathering information about the new patient. What is the nurse achieving through this approach? A) Interpreting what the patient has said B) Evaluating what the patient has said C) Assessing what the patient has said D) Validating what the patient has said

Ans: D Test Bank - Brunner & Suddarth's Textbook of Medical-Surgical Nursing 14e (Hinkle 2017) 58 Feedback: Critical thinkers validate the information presented to make sure that it is accurate (not just supposition or opinion), that it makes sense, and that it is based on fact and evidence. The nurse is not interpreting, evaluating, or assessing the information the patient has given.

The nurse is caring for a patient who has just been diagnosed with chronic rhinosinusitis. While being admitted to the clinic, the patient asks, Will this chronic infection hurt y new kidney? What should the nurse know about chronic rhinosinusitis in patients who have had a transplant? a) the patient will have exaggerated symptoms of rhinosinusitis due to immunosuppression. b) taking immunosuppressive drugs can conribute to chronic rhinosinusitis c) chronic rhinosinusitis can damage the transplanted organ d) immunosuppressive drugs can cause organ rejection

B URIs, specifically chronic rhinosinusitis and recurrent acute rhinosinusitis, may be linked to primary or secondary immune deficiency or treatment with immunosuppressive therapy (i.e., for cancer or organ transplantaion). Typical symptoms may be blunted or absent due to immunosuppression. No evidence indicates damage to the transplanted organ due to chronic rhinosinusitis. Immunosuppressive drugs do not cause organ rejection.

A nurse is caring for a young adult patient whose medical history includes an alpha1-antitrypsin deficiency. This deficiency predisposes the patient to what health problem? Test Bank - Brunner & Suddarth's Textbook of Medical-Surgical Nursing 14e (Hinkle 2017) 473 A) Pulmonary edema B) Lobular emphysema C) Cystic fibrosis (CF) D) Empyema

B Feedback: A host risk factor for COPD is a deficiency of alpha1-antitrypsin, an enzyme inhibitor that protects the lung parenchyma from injury. This deficiency predisposes young patients to rapid development of lobular emphysema even in the absence of smoking. This deficiency does not influence the patients risk of pulmonary edema, CF, or empyema.

A patient on the critical care unit is postoperative day 1 following kidney transplantation from a living donor. The nurses most recent assessments indicate that the patient is producing copious quantities of dilute urine. What is the nurses most appropriate response? A) Assess the patient for further signs or symptoms of rejection. B) Recognize this as an expected finding. C) Inform the primary care provider of this finding. D) Administer exogenous antidiuretic hormone as ordered.

B Feedback: A kidney from a living donor related to the patient usually begins to function immediately after surgery and may produce large quantities of dilute urine. This is not suggestive of rejection and treatment is not warranted. There is no obvious need to report this finding.

perioperative nurse is caring for a postoperative patient. The patient has a shallow respiratory pattern and is reluctant to cough or to begin mobilizing. The nurse should address the patients increased risk for what complication? A) Acute respiratory distress syndrome (ARDS) B) Atelectasis C) Aspiration D) Pulmonary embolism

B Feedback: A shallow, monotonous respiratory pattern coupled with immobility places the patient at an increased risk of developing atelectasis. These specific factors are less likely to result in pulmonary embolism or aspiration. ARDS involves an exaggerated inflammatory response and does not normally result from factors such as immobility and shallow breathing.

A firefighter was trapped in a fire and is admitted to the ICU for smoke inhalation. After 12 hours, the firefighter is exhibiting signs of ARDS and is intubated. What other supportive measures are initiated in a patient with ARDS? A) Psychological counseling B) Nutritional support C) High-protein oral diet D) Occupational therapy

B Feedback: Aggressive, supportive care must be provided to compensate for the severe respiratory dysfunction. This supportive therapy almost always includes intubation and mechanical ventilation. In addition, circulatory support, adequate fluid volume, and nutritional support are important. Oral intake is contraindicated by intubation. Counseling and occupational therapy would not be priorities during the acute stage of ARDS.

The nurse caring for a patient with suspected renal dysfunction calculates that the patients weight has increased by 5 pounds in the past 24 hours. The nurse estimates that the patient has retained approximately how much fluid? A) 1,300 mL of fluid in 24 hours B) 2,300 mL of fluid in 24 hours C) 3,100 mL of fluid in 24 hours D) 5,000 mL of fluid in 24 hours

B Feedback: An increase in body weight commonly accompanies edema. To calculate the approximate weight gain from fluid retention, remember that 1 kg of weight gain equals approximately 1,000 mL of fluid. Five lbs = 2.27 kg = 2,270 mL.

An asthma nurse educator is working with a group of adolescent asthma patients. What intervention is most likely to prevent asthma exacerbations among these patients? A) Encouraging patients to carry a corticosteroid rescue inhaler at all times B) Educating patients about recognizing and avoiding asthma triggers C) Teaching patients to utilize alternative therapies in asthma management D) Ensuring that patients keep their immunizations up to date

B Feedback: Asthma exacerbations are best managed by early treatment and education, including the use of written action plans as part of any overall effort to educate patients about self-management techniques, especially those with moderate or severe persistent asthma or with a history of severe exacerbations. Corticosteroids are not used as rescue inhalers. Alternative therapies are not normally a high priority, though their use may be appropriate in some cases. Immunizations should be kept up to date, but this does not necessarily prevent asthma exacerbations.

A nurse is caring for a patient who has been hospitalized with an acute asthma exacerbation. What drugs should the nurse expect to be ordered for this patient to gain underlying control of persistent asthma? A) Rescue inhalers B) Anti-inflammatory drugs C) Antibiotics D) Antitussives

B Feedback: Because the underlying pathology of asthma is inflammation, control of persistent asthma is accomplished primarily with regular use of anti-inflammatory medications. Rescue inhalers, antibiotics, and antitussives do not aid in the first-line control of persistent asthma.

The nurse is caring for a patient scheduled for renal angiography following a motor vehicle accident. What patient preparation should the nurse most likely provide before this test? A) Administration of IV potassium chloride B) Administration of a laxative C) Administration of Gastrografin D) Administration of a 24-hour urine test

B Feedback: Before the procedure, a laxative may be prescribed to evacuate the colon so that unobstructed x-rays can be obtained. A 24-hour urine test is not necessary prior to the procedure. Gastrografin and potassium chloride are not administered prior to renal angiography.

A patient states that her family has had several colds during this winter and spring despite their commitment to handwashing. The high communicability of the common cold is attributable to what factor? A) Cold viruses are increasingly resistant to common antibiotics. B) The virus is shed for 2 days prior to the emergence of symptoms. C) A genetic predisposition to viral rhinitis has recently been identified. D) Overuse of OTC cold remedies creates a rebound susceptibility to future colds.

B Feedback: Colds are highly contagious because virus is shed for about 2 days before the symptoms appear and during the first part of the symptomatic phase. Antibiotic resistance is not relevant to viral illnesses and OTC medications do not have a rebound effect. Genetic factors do not exist.

The nurse knows that elderly patients are at higher risk for complications and adverse outcomes during the intraoperative period. What is the best rationale for this phenomenon? A) The elderly patient has a more angular bone structure than a younger person. B) The elderly patient has reduced ability to adjust rapidly to emotional and physical stress. C) The elderly patient has impaired thermoregulatory mechanisms, which increase susceptibility to hyperthermia. D) The elderly patient has an impaired ability to decrease his or her metabolic rate.

B Feedback: Factors that affect the elderly surgical patient in the intraoperative period include the following: impaired ability to increase metabolic rate and impaired thermoregulatory mechanisms increase susceptibility to hypothermia. Bone loss (25% in women, 12% in men) necessitates careful manipulation and positioning during surgery. Reduced ability to adjust rapidly to emotional and physical stress influences surgical outcomes and requires meticulous observation of vital functions. Older adults do not have more angular bones than younger people.

The patients surgery is nearly finished and the surgeon has opted to use tissue adhesives to close the surgical wound. This requires the nurse to prioritize assessments related to what complication? A) Hypothermia B) Anaphylaxis C) Infection D) Malignant hyperthermia

B Feedback: Fibrin sealants are used in a variety of surgical procedures, and cyanoacrylate tissue adhesives are used to close wounds without the use of sutures. These sealants have been implicated in allergic reactions and anaphylaxis. There is not an increased risk of malignant hyperthermia, hypothermia, or infection because of the use of tissue adhesives.

During the care of a preoperative patient, the nurse has given the patient a preoperative benzodiazepine. The patient is now requesting to void. What action should the nurse take? A) Assist the patient to the bathroom. B) Offer the patient a bedpan or urinal. C) Wait until the patient gets to the operating room and is catheterized. D) Have the patient go to the bathroom.

B Feedback: If a preanesthetic medication is administered, the patient is kept in bed with the side rails raised because the medication can cause lightheadedness or drowsiness. If a patient needs to void following administration of a sedative, the nurse should offer the patient a urinal. The patient should not get out of bed because of the potential for lightheadedness.

The intraoperative nurse is implementing a care plan that addresses the surgical patients risk for vomiting. Interventions that address the potential for vomiting reduce the risk of what subsequent surgical complication? A) Impaired skin integrity B) Hypoxia C) Malignant hyperthermia D) Hypothermia

B Feedback: If the patient aspirates vomitus, an asthma-like attack with severe bronchial spasms and wheezing is triggered. Pneumonitis and pulmonary edema can subsequently develop, leading to extreme hypoxia. Vomiting can cause choking, but the question asks about aspirated vomitus. Malignant hyperthermia is an adverse reaction to anesthesia. Aspirated vomitus does not cause hypothermia. Vomiting does not result in impaired skin integrity.

A patient is 2 hours postoperative with a Foley catheter in situ. The last hourly urine output recorded for this patient was 10 mL. The tubing of the Foley is patent. What should the nurse do? A) Irrigate the Foley with 30 mL normal saline. B) Notify the physician and continue to monitor the hourly urine output closely. C) Decrease the IV fluid rate and massage the patients abdomen. D) Have the patient sit in high-Fowlers position.

B Feedback: If the patient has an indwelling urinary catheter, hourly outputs are monitored and rates of less than 30 mL/hr are reported. The urine output should continue to be monitored hourly by the nurse. Irrigation would not be warranted.

A patient with chronic kidney disease is completing an exchange during peritoneal dialysis. The nurse observes that the peritoneal fluid is draining slowly and that the patients abdomen is increasing in girth. What is the nurses most appropriate action? Test Bank - Brunner & Suddarth's Textbook of Medical-Surgical Nursing 14e (Hinkle 2017) 1033 A) Advance the catheter 2 to 4 cm further into the peritoneal cavity. B) Reposition the patient to facilitate drainage. C) Aspirate from the catheter using a 60-mL syringe. D) Infuse 50 mL of additional dialysate.

B Feedback: If the peritoneal fluid does not drain properly, the nurse can facilitate drainage by turning the patient from side to side or raising the head of the bed. The catheter should never be pushed further into the peritoneal cavity. It would be unsafe to aspirate or to infuse more dialysate.

The nurse has implemented a bladder retraining program for an older adult patient. The nurse places the patient on a timed voiding schedule and performs an ultrasonic bladder scan after each void. The nurse notes that the patient typically has approximately 50 mL of urine remaining in her bladder after voiding. What would be the nurses best response to this finding? A) Perform a straight catheterization on this patient. B) Avoid further interventions at this time, as this is an acceptable finding. C) Place an indwelling urinary catheter. D) Press on the patients bladder in an attempt to encourage complete emptying.

B Feedback: In adults older than 60 years of age, 50 to 100 mL of residual urine may remain after each voiding because of the decreased contractility of the detrusor muscle. Consequently, further interventions are not likely warranted.

patient with thoracic trauma is admitted to the ICU. The nurse notes the patients chest and neck are swollen and there is a crackling sensation when palpated. The nurse consequently identifies the presence of subcutaneous emphysema. If this condition becomes severe and threatens airway patency, what intervention is indicated? A) A chest tube B) A tracheostomy C) An endotracheal tube D) A feeding tube

B Feedback: In severe cases in which there is widespread subcutaneous emphysema, a tracheostomy is indicated if airway patency is threatened by pressure of the trapped air on the trachea. The other listed tubes would neither resolve the subcutaneous emphysema nor the consequent airway constriction.

recognizes that the patient is in stage II (the excitement stage) of anesthesia. Which intervention would be most appropriate for the nurse to implement during this stage? A) Rub the patients back. B) Restrain the patient. C) Encourage the patient to express feelings. D) Stroke the patients hand.

B Feedback: In stage II, the patient may struggle, shout, or laugh. The movements of the patient may be uncontrolled, so it is essential the nurse help to restrain the patient for safety. None of the other listed actions protects the patients safety.

Verification that all required documentation is completed is an important function of the intraoperative nurse. The intraoperative nurse should confirm that the patients accompanying documentation includes which of the following? A) Discharge planning B) Informed consent C) Analgesia prescription D) Educational resources

B Feedback: It is important to review the patients record for the following: correct informed surgical consent, with patients signature; completed records for health history and physical examination; results of diagnostic studies; and allergies (including latex). Discharge planning records and prescriptions are not normally Test Bank - Brunner & Suddarth's Textbook of Medical-Surgical Nursing 14e (Hinkle 2017) 368 necessary. Educational resources would not be included at this stage of the surgical process.

The OR nurse is taking the patient into the OR when the patient informs the operating nurse that his grandmother spiked a 104F temperature in the OR and nearly died 15 years ago. What relevance does this information have regarding the patient? A) The patient may be experiencing presurgical anxiety. B) The patient may be at risk for malignant hyperthermia. C) The grandmothers surgery has minimal relevance to the patients surgery. D) The patient may be at risk for a sudden onset of postsurgical infection.

B Feedback: Malignant hyperthermia is an inherited muscle disorder chemically induced by anesthetic agents. Identifying patients at risk is imperative because the mortality rate is 50%. The patients anxiety is not relevant, the grandmothers surgery is very relevant, and all patients are at risk for hypothermia.

The nurse is providing preoperative teaching to a patient scheduled for surgery. The nurse is instructing the patient on the use of deep breathing, coughing, and the use of incentive spirometry when the patient states, I dont know why youre focusing on my breathing. My surgery is on my hip, not my chest. What rationale for these instructions should the nurse provide? A) To prevent chronic obstructive pulmonary disease (COPD) Test Bank - Brunner & Suddarth's Textbook of Medical-Surgical Nursing 14e (Hinkle 2017) 346 B) To promote optimal lung expansion C) To enhance peripheral circulation D) To prevent pneumothorax

B Feedback: One goal of preoperative nursing care is to teach the patient how to promote optimal lung expansion and consequent blood oxygenation after anesthesia. COPD is not a realistic risk and pneumothorax is also unlikely. Breathing exercises do not primarily affect peripheral circulation.

A patient on the medical unit has a documented history of polycystic kidney disease (PKD). What principle should guide the nurses care of this patient? A) The disease is self-limiting and cysts usually resolve spontaneously in the fifth or sixth decade of life. B) The patients disease is incurable and the nurses interventions will be supportive. C) The patient will eventually require surgical removal of his or her renal cysts. D) The patient is likely to respond favorably to lithotripsy treatment of the cysts.

B Feedback: PKD is incurable and care focuses on support and symptom control. It is not self-limiting and is not treated surgically or with lithotripsy.

A patient is scheduled for a diagnostic MRI of the lower urinary system. What pre-procedure education should the nurse include? A) The need to be NPO for 12 hours prior to the test B) Relaxation techniques to apply during the test C) The need for conscious sedation prior to the test D) The need to limit fluid intake to 1 liter in the 24 hours before the test

B Feedback: Patient preparation should include teaching relaxation techniques because the patient needs to remain still during an MRI. The patient does not normally need to be NPO or fluid-restricted before the test and conscious sedation is not usually implemented.

The nursing instructor is discussing the difference between ambulatory surgical centers and hospitalbased surgical units. A student asks why some patients have surgery in the hospital and others are sent to ambulatory surgery centers. What is the instructors best response? A) Patients who go to ambulatory surgery centers are more independent than patients admitted to the hospital. B) Patients admitted to the hospital for surgery usually have multiple health needs. C) In most cases, only emergency and trauma patients are admitted to the hospital. D) Patients who have surgery in the hospital are those who need to have anesthesia administered.

B Feedback: Patients admitted to the clinical unit for postoperative care have multiple needs and stay for a short period of time. Patients who have surgery in ambulatory centers do not necessarily have greater independence. It is not true that only trauma and emergency surgeries are done in the hospital. Ambulatory centers can administer anesthesia.

A nurse on a busy medical unit provides care for many patients who require indwelling urinary catheters at some point during their hospital care. The nurse should recognize a heightened risk of injury associated with indwelling catheter use in which patient? A) A patient whose diagnosis of chronic kidney disease requires a fluid restriction B) A patient who has Alzheimers disease and who is acutely agitated C) A patient who is on bed rest following a recent episode of venous thromboembolism D) A patient who has decreased mobility following a transmetatarsal amputation

B Feedback: Patients who are confused and agitated risk trauma through the removal of an indwelling catheter which has the balloon still inflated. Recent VTE, amputation, and fluid restriction do not directly create a risk for injury or trauma associated with indwelling catheter use.

A patient is scheduled for a CT scan of the abdomen with contrast. The patient has a baseline creatinine level of 2.3 mg/dL. In preparing this patient for the procedure, the nurse anticipates what orders? A) Monitor the patients electrolyte values every hour before the procedure. B) Preprocedure hydration and administration of acetylcysteine C) Hemodialysis immediately prior to the CT scan Test Bank - Brunner & Suddarth's Textbook of Medical-Surgical Nursing 14e (Hinkle 2017) 1023 D) Obtain a creatinine clearance by collecting a 24-hour urine specimen.

B Feedback: Radiocontrast-induced nephropathy is a major cause of hospital-acquired acute kidney injury. Baseline levels of creatinine greater than 2 mg/dL identify the patient as being high risk. Preprocedure hydration and prescription of acetylcysteine (Mucomyst) the day prior to the test is effective in prevention. The nurse would not monitor the patients electrolytes every hour preprocedure. Nothing in the scenario indicates the need for hemodialysis. A creatinine clearance is not necessary prior to a CT scan with contrast.

A patient with renal failure secondary to diabetic nephropathy has been admitted to the medical unit. What is the most life-threatening effect of renal failure for which the nurse should monitor the patient? A) Accumulation of wastes B) Retention of potassium C) Depletion of calcium D) Lack of BP control

B Feedback: Retention of potassium is the most life-threatening effect of renal failure. Aldosterone causes the kidney to excrete potassium, in contrast to aldosterones effects on sodium described previously. Acidbase balance, the amount of dietary potassium intake, and the flow rate of the filtrate in the distal tubule also influence the amount of potassium secreted into the urine. Hypocalcemia, the accumulation of wastes, and lack of BP control are complications associated with renal failure, but do not have same level of threat to the patients well-being as hyperkalemia.

An older adult patient has been diagnosed with COPD. What characteristic of the patients current health status would preclude the safe and effective use of a metered-dose inhaler (MDI)? A) The patient has not yet quit smoking. B) The patient has severe arthritis in her hands. C) The patient requires both corticosteroids and beta2-agonists. D) The patient has cataracts.

B Feedback: Safe and effective MDI use requires the patient to be able to manipulate the device independently, which may be difficult if the patient has arthritis. Smoking does not preclude MDI use. A modest loss of vision does not preclude the use of an MDI and a patient can safely use more than one MDI.

A patient with recurrent urinary tract infections has just undergone a cystoscopy and complains of slight hematuria during the first void after the procedure. What is the nurses most appropriate action? A) Administer a STAT dose of vitamin K, as ordered. B) Reassure the patient that this is not unexpected and then monitor the patient for further bleeding. C) Promptly inform the physician of this assessment finding. D) Position the patient supine and insert a Foley catheter, as ordered.

B Feedback: Some burning on voiding, blood-tinged urine, and urinary frequency from trauma to the mucous membranes can be expected after cystoscopy. The nurse should explain this to the patient and ensure that the bleeding resolves. No clear need exists to report this finding and it does not warrant insertion of a Foley catheter or vitamin K administration.

The nurse coming on shift on the medical unit is taking a report on four patients. What patient does the nurse know is at the greatest risk of developing ESKD? A) A patient with a history of polycystic kidney disease B) A patient with diabetes mellitus and poorly controlled hypertension C) A patient who is morbidly obese with a history of vascular disorders Test Bank - Brunner & Suddarth's Textbook of Medical-Surgical Nursing 14e (Hinkle 2017) 1019 D) A patient with severe chronic obstructive pulmonary disease

B Feedback: Systemic diseases, such as diabetes mellitus (leading cause); hypertension; chronic glomerulonephritis; pyelonephritis; obstruction of the urinary tract; hereditary lesions, such as in polycystic kidney disease; vascular disorders; infections; medications; or toxic agents may cause ESKD. A patient with more than one of these risk factors is at the greatest risk for developing ESKD. Therefore, the patient with diabetes and hypertension is likely at highest risk for ESKD.

A patient has undergone the creation of an Indiana pouch for the treatment of bladder cancer. The nurse identified the nursing diagnosis of disturbed body image. How can the nurse best address the effects of this urinary diversion on the patients body image? A) Emphasize that the diversion is an integral part of successful cancer treatment. B) Encourage the patient to speak openly and frankly about the diversion. C) Allow the patient to initiate the process of providing care for the diversion. D) Provide the patient with detailed written materials about the diversion at the time of discharge.

B Feedback: Test Bank - Brunner & Suddarth's Textbook of Medical-Surgical Nursing 14e (Hinkle 2017) 1053 Allowing the patient to express concerns and anxious feelings can help with body image, especially in adjusting to the changes in toileting habits. The nurse may have to initiate dialogue about the management of the diversion, especially if the patient is hesitant. Provision of educational materials is rarely sufficient to address a sudden change and profound change in body image. Emphasizing the role of the diversion in cancer treatment does not directly address the patients body image.

A patient has just been diagnosed with squamous cell carcinoma of the neck. While the nurse is doing health education, the patient asks, Does this kind of cancer tend to spread to other parts of the body? What is the nurses best response? A) In many cases, this type of cancer spreads to other parts of the body. B) This cancer usually does not spread to distant sites in the body. C) You will have to speak to your oncologist about that. D) Squamous cell carcinoma is nothing to be concerned about, so try to focus on your health.

B Feedback: Test Bank - Brunner & Suddarth's Textbook of Medical-Surgical Nursing 14e (Hinkle 2017) 452 The incidence of distant metastasis with squamous cell carcinoma of the head and neck (including larynx cancer) is relatively low. The patients prognosis is determined by the oncologist, but the patient has asked a general question and it would be inappropriate to refuse a response. The nurse must not downplay the patients concerns.

The nurse is teaching a health class about UTIs to a group of older adults. What characteristic of UTIs should the nurse cite? Test Bank - Brunner & Suddarth's Textbook of Medical-Surgical Nursing 14e (Hinkle 2017) 1040 A) Men over age 65 are equally prone to UTIs as women, but are more often asymptomatic. B) The prevalence of UTIs in men older than 50 years of age approaches that of women in the same age group. C) Men of all ages are less prone to UTIs, but typically experience more severe symptoms. D) The prevalence of UTIs in men cannot be reliably measured, as men generally do not report UTIs.

B Feedback: The antibacterial activity of the prostatic secretions that protect men from bacterial colonization of the urethra and bladder decreases with aging. The prevalence of infection in men older than 50 years of age approaches that of women in the same age group. Men are not more likely to be asymptomatic and are not known to be reluctant to report UTIs.

A patient with a diagnosis of respiratory acidosis is experiencing renal compensation. What function does the kidney perform to assist in restoring acidbase balance? A) Sequestering free hydrogen ions in the nephrons B) Returning bicarbonate to the bodys circulation C) Returning acid to the bodys circulation D) Excreting bicarbonate in the urine

B Feedback: The kidney performs two major functions to assist in acidbase balance. The first is to reabsorb and return to the bodys circulation any bicarbonate from the urinary filtrate; the second is to excrete acid in the urine. Retaining bicarbonate will counteract an acidotic state. The nephrons do not sequester free hydrogen ions.

The nurse is caring for a patient who is postoperative day 2 following a colon resection. While turning him, wound dehiscence with evisceration occurs. What should be the nurses first response? A) Return the patient to his previous position and call the physician. B) Place saline-soaked sterile dressings on the wound. C) Assess the patients blood pressure and pulse. D) Pull the dehiscence closed using gloved hands.

B Feedback: The nurse should first place saline-soaked sterile dressings on the open wound to prevent tissue drying and possible infection. Then the nurse should call the physician and take the patients vital signs. The dehiscence needs to be surgically closed, so the nurse should never try to close it.

A patient who involved in a workplace accident suffered a penetrating wound of the chest that led to acute respiratory failure. What goal of treatment should the care team prioritize when planning this patients care? A) Facilitation of long-term intubation B) Restoration of adequate gas exchange C) Attainment of effective coping Test Bank - Brunner & Suddarth's Textbook of Medical-Surgical Nursing 14e (Hinkle 2017) 468 D) Self-management of oxygen therapy

B Feedback: The objectives of treatment are to correct the underlying cause of respiratory failure and to restore adequate gas exchange in the lung. This is priority over coping and self-care. Long-term ventilation may or may not be indicated.

Resection of a patients bladder tumor has been incomplete and the patient is preparing for the administration of the first ordered instillation of topical chemotherapy. When preparing the patient, the nurse should emphasize the need to do which of the following? A) Remain NPO for 12 hours prior to the treatment. B) Hold the solution in the bladder for 2 hours before voiding. Test Bank - Brunner & Suddarth's Textbook of Medical-Surgical Nursing 14e (Hinkle 2017) 1051 C) Drink the intravesical solution quickly and on an empty stomach. D) Avoid acidic foods and beverages until the full cycle of treatment is complete.

B Feedback: The patient is allowed to eat and drink before the instillation procedure. Once the bladder is full, the patient must retain the intravesical solution for 2 hours before voiding. The solution is instilled through the meatus; it is not consumed orally. There is no need to avoid acidic foods and beverages during treatment.

The nurse admits a patient to the PACU with a blood pressure of 132/90 mm Hg and a pulse of 68 beats per minute. After 30 minutes, the patients blood pressure is 94/47 mm Hg, and the pulse is 110. The nurse documents that the patients skin is cold, moist, and pale. Of what is the patient showing signs? A) Hypothermia B) Hypovolemic shock C) Neurogenic shock D) Malignant hyperthermia

B Feedback: The patient is exhibiting symptoms of hypovolemic shock; therefore, the nurse should notify the patients physician and anticipate orders for fluid and/or blood product replacement. Neurogenic shock does not normally result in tachycardia and malignant hyperthermia would not present at this stage in the operative experience. Hypothermia does not cause hypotension and tachycardia.

A nurse is caring for a female patient whose urinary retention has not responded to conservative treatment. When educating this patient about self-catheterization, the nurse should encourage what practice? A) Assuming a supine position for self-catheterization B) Using clean technique at home to catheterize C) Inserting the catheter 1 to 2 inches into the urethra D) Self-catheterizing every 2 hours at home

B Feedback: The patient may use a clean (nonsterile) technique at home, where the risk of cross-contamination is reduced. The average daytime clean intermittent catheterization schedule is every 4 to 6 hours and just before bedtime. The female patient assumes a Fowlers position and uses a mirror to help locate the urinary meatus. The nurse teaches her to catheterize herself by inserting a catheter 7.5 cm (3 inches) into the urethra, in a downward and backward direction.

You are the nurse caring for an unconscious trauma victim who needs emergency surgery. The patient is a 55-year-old man with an adult son. He is legally divorced and is planning to be remarried in a few weeks. His parents are at the hospital with the other family members. The physician has explained the need for surgery, the procedure to be done, and the risks to the children, the parents, and the fianc. Who should be asked to sign the surgery consent form? A) The fianc B) The son C) The physician, acting as a surrogate D) The patients father

B Feedback: The patient personally signs the consent if of legal age and mentally capable. Permission is otherwise obtained from a surrogate, who most often is a responsible family member (preferably next of kin) or legal guardian. In this instance, the child would be the appropriate person to ask to sign the consent form as he is the closest relative at the hospital. The fianc is not legally related to him as the marriage has not yet taken place. The father would only be asked to sign the consent if no children were present to sign. The physician would not sign if family members were available.

A 52-year-old patient is scheduled to undergo ileal conduit surgery. When planning this patients discharge education, what is the most plausible nursing diagnosis that the nurse should address? A) Impaired mobility related to limitations posed by the ileal conduit B) Deficient knowledge related to care of the ileal conduit C) Risk for deficient fluid volume related to urinary diversion D) Risk for autonomic dysreflexia related to disruption of the sacral plexus

B Feedback: The patient will most likely require extensive teaching about the care and maintenance of a new urinary diversion. A diversion does not create a serious risk of fluid volume deficit. Mobility is unlikely to be impaired after the immediate postsurgical recovery. The sacral plexus is not threatened by the creation of a urinary diversion.

The nurse is providing care for a patient who has just been admitted to the postsurgical unit following a laryngectomy. What assessment should the nurse prioritize? A) The patients swallowing ability Test Bank - Brunner & Suddarth's Textbook of Medical-Surgical Nursing 14e (Hinkle 2017) 450 B) The patients airway patency C) The patients carotid pulses D) Signs and symptoms of infection

B Feedback: The patient with a laryngectomy is a risk for airway occlusion and respiratory distress. As in all nursing situations, assessment of the airway is a priority over other potential complications and assessment parameters.

A 77-year-old mans coronary artery bypass graft has been successful and discharge planning is underway. When planning the patients subsequent care, the nurse should know that the postoperative phase of perioperative nursing ends at what time? A) When the patient is returned to his room after surgery B) When a follow-up evaluation in the clinical or home setting is done C) When the patient is fully recovered from all effects of the surgery D) When the family becomes partly responsible for the patients care

B Feedback: The postoperative phase begins with the admission of the patient to the PACU and ends with a follow-up evaluation in the clinical setting or home.

An admitting nurse is assessing a patient with COPD. The nurse auscultates diminished breath sounds, which signify changes in the airway. These changes indicate to the nurse to monitor the patient for what? A) Kyphosis and clubbing of the fingers B) Dyspnea and hypoxemia C) Sepsis and pneumothorax D) Bradypnea and pursed lip breathing

B Feedback: These changes in the airway require that the nurse monitor the patient for dyspnea and hypoxemia. Kyphosis is a musculoskeletal problem. Sepsis and pneumothorax are atypical complications. Tachypnea is much more likely than bradypnea. Pursed lip breathing can relieve dyspnea.

A nurse is caring for a patient with impaired renal function. A creatinine clearance measurement has been ordered. The nurse should facilitate collection of what samples? A) A fasting serum potassium level and a random urine sample B) A 24-hour urine specimen and a serum creatinine level midway through the urine collection process C) A BUN and serum creatinine level on three consecutive mornings D) A sterile urine specimen and an electrolyte panel, including sodium, potassium, calcium, and phosphorus values

B Feedback: To calculate creatinine clearance, a 24-hour urine specimen is collected. Midway through the collection, the serum creatinine level is measured.

The nurse is caring for a patient recently diagnosed with renal calculi. The nurse should instruct the patient to increase fluid intake to a level where the patient produces at least how much urine each day? A) 1,250 mL B) 2,000 mL C) 2,750 mL D) 3,500 mL

B Feedback: Unless contraindicated by renal failure or hydronephrosis, patients with renal stones should drink at least eight 8-ounce glasses of water daily or have IV fluids prescribed to keep the urine dilute. A urine output Test Bank - Brunner & Suddarth's Textbook of Medical-Surgical Nursing 14e (Hinkle 2017) 1039 exceeding 2 L a day is advisable

The critical care nurse is monitoring the patients urine output and drains following renal surgery. What should the nurse promptly report to the physician? A) Increased pain on movement B) Absence of drain output C) Increased urine output D) Blood-tinged serosanguineous drain output

B Feedback: Urine output and drainage from tubes inserted during surgery are monitored for amount, color, and type or characteristics. Decreased or absent drainage is promptly reported to the physician because it may indicate obstruction that could cause pain, infection, and disruption of the suture lines. Reporting increased pain on movement has nothing to do with the scenario described. Increased urine output and serosanguineous drainage are expected.

A patient underwent an open bowel resection 2 days ago and the nurses most recent assessment of the patients abdominal incision reveals that it is dehiscing. What factor should the nurse suspect may have caused the dehiscence? A) The patients surgical dressing was changed yesterday and today. Test Bank - Brunner & Suddarth's Textbook of Medical-Surgical Nursing 14e (Hinkle 2017) 393 B) The patient has vomited three times in the past 12 hours. C) The patient has begun voiding on the commode instead of a bedpan. D) The patient used PCA until this morning

B Feedback: Vomiting can produce tension on wounds, particularly of the torso. Dressing changes and light mobilization are unlikely to cause dehiscence. The use of a PCA is not associated with wound dehiscence.

The PACU nurse is caring for a patient who has arrived from the OR. During the initial assessment, the nurse observes that the patients skin has become blue and dusky. The nurse looks, listens, and feels for breathing, and determines the patient is not breathing. What is the priority intervention? A) Check the patients oxygen saturation level, continue to monitor for apnea, and perform a focused assessment. B) Treat the possible airway obstruction by tilting the head back and pushing forward on the angle of Test Bank - Brunner & Suddarth's Textbook of Medical-Surgical Nursing 14e (Hinkle 2017) 385 the lower jaw. C) Assess the arterial pulses, and place the patient in the Trendelenburg position. D) Reintubate the patient.

B Feedback: When a nurse finds a patient who is not breathing, the priority intervention is to open the airway and treat a possible hypopharyngeal obstruction. To treat the possible airway obstruction, the nurse tilts the head back and then pushes forward on the angle of the lower jaw or performs the jaw thrust method to open the airway. This is an emergency and requires the basic life support intervention of airway, breathing, and circulation assessment. Arterial pulses should be checked only after airway and breathing have been established. Reintubation and resuscitation would begin after rapidly ruling out a hypopharyngeal obstruction.

The nurse is preparing to change a patients abdominal dressing. The nurse recognizes the first step is to provide the patient with information regarding the procedure. Which of the following explanations should the nurse provide to the patient? A) The dressing change is often painful, and we will be giving you pain medication prior to the procedure so you do not have to worry. B) During the dressing change, I will provide privacy at a time of your choosing, it should not be Test Bank - Brunner & Suddarth's Textbook of Medical-Surgical Nursing 14e (Hinkle 2017) 380 painful, and you can look at the incision and help with the procedure if you want to. C) The dressing change should not be painful, but you can never be sure, and infection is always a concern. D) The best time for doing a dressing change is during lunch so we are not interrupted. I will provide privacy, and it should not be painful.

B Feedback: When having dressings changed, the patient needs to be informed that the dressing change is a simple procedure with little discomfort; privacy will be provided; and the patient is free to look at the incision or even assist in the dressing change itself. If the patient decides to look at the incision, assurance is given that the incision will shrink as it heals and that the redness will likely fade. Dressing changes should not be painful, but giving pain medication prior to the procedure is always a good preventive measure. Telling the patient that the dressing change should not be painful, but you can never be sure, and infection is always a concern does not offer the patient any real information or options and serves only to create fear. The best time for dressing changes is when it is most convenient for the patient; nutrition is important so interrupting lunch is probably a poor choice.

A nurse who provides care in a long-term care facility is aware of the high incidence and prevalence of urinary tract infections among older adults. What action has the greatest potential to prevent UTIs in this Test Bank - Brunner & Suddarth's Textbook of Medical-Surgical Nursing 14e (Hinkle 2017) 1044 population? A) Administer prophylactic antibiotics as ordered. B) Limit the use of indwelling urinary catheters. C) Encourage frequent mobility and repositioning. D) Toilet residents who are immobile on a scheduled basis.

B Feedback: When indwelling catheters are used, the risk of UTI increases dramatically. Limiting their use significantly reduces an older adults risk of developing a UTI. Regular toileting promotes continence, but has only an indirect effect on the risk of UTIs. Prophylactic antibiotics are not normally administered. Mobility does not have a direct effect on UTI risk.

The nurse is preparing a patient for surgery. The patient states that she is very nervous and really does not understand what the surgical procedure is for or how it will be performed. What is the most appropriate nursing action for the nurse to take? A) Have the patient sign the informed consent and place it in the chart. Test Bank - Brunner & Suddarth's Textbook of Medical-Surgical Nursing 14e (Hinkle 2017) 342 B) Call the physician to review the procedure with the patient. C) Explain the procedure clearly to the patient and her family. D) Provide the patient with a pamphlet explaining the procedure.

B Feedback: While the nurse may ask the patient to sign the consent form and witness the signature, it is the surgeons responsibility to provide a clear and simple explanation of what the surgery will entail prior to the patient giving consent. The surgeon must also inform the patient of the benefits, alternatives, possible risks, complications, disfigurement, disability, and removal of body parts as well as what to expect in the early and late postoperative periods. The nurse clarifies the information provided, and, if the patient requests additional information, the nurse notifies the physician. The consent formed should not be signed until the patient understands the procedure that has been explained by the surgeon. The provision of a pamphlet will benefit teaching the patient about the surgical procedure, but will not substitute for the information provided by the physician.

A patient is on call to the OR for an aortobifemoral bypass and the nurse administers the ordered preoperative medication. After administering a preoperative medication to the patient, what should the nurse do? A) Encourage light ambulation. B) Place the bed in a low position with the side rails up. C) Tell the patient that he will be asleep before he leaves for surgery. D) Take the patients vital signs every 15 minutes.

B Test Bank - Brunner & Suddarth's Textbook of Medical-Surgical Nursing 14e (Hinkle 2017) 348 Feedback: When the preoperative medication is given, the bed should be placed in low position with the side rails raised. The patient should not get up without assistance. The patient may not be asleep, but he may be drowsy. Vital signs should be taken before the preoperative medication is given; vital signs are not normally required every 15 minutes after administration.

The policies and procedures on a preoperative unit are being amended to bring them closer into alignment with the focus of the Surgical Care Improvement Project (SCIP). What intervention most directly addresses the priorities of the SCIP? A) Actions aimed at increasing participation of families in planning care B) Actions aimed at preventing surgical site infections C) Actions aimed at increasing interdisciplinary collaboration D) Actions aimed at promoting the use of complementary and alternative medicine (CAM)

B Test Bank - Brunner & Suddarth's Textbook of Medical-Surgical Nursing 14e (Hinkle 2017) 356 Feedback: SCIP identifies performance measures aimed at preventing surgical complications, including venous thromboembolism (VTE) and surgical site infections (SSI). It does not explicitly address family participation, interdisciplinary collaboration, or CAM.

A new employee asks the occupational health nurse about measures to prevent inhalation exposure of the substances. Which statement by the nurse will decrease the patients exposure risk to toxic substances? A) Position a fan blowing on the toxic substances to prevent the substance from becoming stagnant in the air. B) Wear protective attire and devices when working with a toxic substance. C) Make sure that you keep your immunizations up to date to prevent respiratory diseases resulting from toxins. D) Always wear a disposable paper face mask when you are working with inhalable toxins.

B Test Bank - Brunner & Suddarth's Textbook of Medical-Surgical Nursing 14e (Hinkle 2017) 457 Feedback: When working with toxic substances, the employee must wear or use protective devices such as face masks, hoods, or industrial respirators. Immunizations do not confer protection from toxins and a paper mask is normally insufficient protection. Never position a fan directly blowing on the toxic substance as it will disperse the fumes throughout the area.

A hospital has been the site of an increased incidence of hospital-acquired pneumonia (HAP). What is an important measure for the prevention of HAP? A) Administration of prophylactic antibiotics B) Administration of pneumococcal vaccine to vulnerable individuals C) Obtaining culture and sensitivity swabs from all newly admitted patients D) Administration of antiretroviral medications to patients over age 65

B Test Bank - Brunner & Suddarth's Textbook of Medical-Surgical Nursing 14e (Hinkle 2017) 463 Feedback: Pneumococcal vaccination reduces the incidence of pneumonia, hospitalizations for cardiac conditions, and deaths in the general older adult population. A onetime vaccination of pneumococcal polysaccharide vaccine (PPSV) is recommended for all patients 65 years of age or older and those with chronic diseases. Antibiotics are not given on a preventative basis and antiretroviral medications do not affect the most common causative microorganisms. Culture and sensitivity testing by swabbing is not performed for pneumonia since the microorganisms are found in sputum.

An asthma educator is teaching a patient newly diagnosed with asthma and her family about the use of a peak flow meter. The educator should teach the patient that a peak flow meter measures what value? A) Highest airflow during a forced inspiration B) Highest airflow during a forced expiration C) Airflow during a normal inspiration D) Airflow during a normal expiration

B Test Bank - Brunner & Suddarth's Textbook of Medical-Surgical Nursing 14e (Hinkle 2017) 489 Feedback: Peak flow meters measure the highest airflow during a forced expiration.

34. A patient in the ICU has had an endotracheal tube in place for 3 weeks. The physician has ordered that a tracheostomy tube be placed. The patients family wants to know why the endotracheal tube cannot be left in place. What would be the nurses best response? Test Bank - Brunner & Suddarth's Textbook of Medical-Surgical Nursing 14e (Hinkle 2017) 431 A) The physician may feel that mechanical ventilation will have to be used long-term. B) Long-term use of an endotracheal tube diminishes the normal breathing reflex. C) When an endotracheal tube is left in too long it can damage the lining of the windpipe. D) It is much harder to breathe through an endotracheal tube than a tracheostomy.

C Endotracheal intubation may be used for no longer than 2-3 weeks, by which time a tracheostomy must be considered to decrease irritation of and, trauma to, the tracheal lining, to reduce the incidience of vocal cord paralysis (secondary to laryngeal nerve damage), and to decrease the work of breathing. The need for long-term ventilation would not be the primary rationale for this change in treatment. Endotracheal tubes do not diminish the breathing reflex.

The nurse is caring for a patient who has undergone creation of a urinary diversion. Forty-eight hours postoperatively, the nurses assessment reveals that the stoma is a dark purplish color. What is the nurses most appropriate response? A) Document the presence of a healthy stoma. B) Assess the patient for further signs and symptoms of infection. C) Inform the primary care provider that the vascular supply may be compromised. D) Liaise with the wound-ostomy-continence (WOC) nurse because the ostomy appliance around the stoma may be too loose.

C Feedback: A healthy stoma is pink or red. A change from this normal color to a dark purplish color suggests that the vascular supply may be compromised. A loose ostomy appliance and infections do not cause a dark purplish stoma.

A patient is being treated for bacterial pharyngitis. Which of the following should the nurse recommend when promoting the patients nutrition during treatment? A) A 1.5 L/day fluid restriction B) A high-potassium, low-sodium diet C) A liquid or soft diet D) A high-protein diet

C Feedback: A liquid or soft diet is provided during the acute stage of the disease, depending on the patients appetite and the degree of discomfort that occurs with swallowing. The patient is encouraged to drink as much fluid as possible (at least 2 to 3 L/day). There is no need for increased potassium or protein intake.

The nurse is checking the informed consent for a 17-year-old who has just been married and expecting her first child. She is scheduled for a cesarean section. She is still living with her parents and is on her parents health insurance. When obtaining informed consent for the cesarean section, who is legally responsible for signing? A) Her parents B) Her husband C) The patient D) The obstetrician

C Feedback: An emancipated minor (married or independently earning his or her own living) may sign his or her own consent form. In this case, the patient is the only person who can provide consent unless she would be neurologically incapacitated or incompetent, in which case her husband would need to provide consent.

The nurse is caring for a patient after abdominal surgery in the PACU. The patients blood pressure has increased and the patient is restless. The patients oxygen saturation is 97%. What cause for this change in status should the nurse first suspect? A) The patient is hypothermic. B) The patient is in shock. C) The patient is in pain. D) The patient is hypoxic.

C Feedback: An increase in blood pressure and restlessness are symptoms of pain. The patients oxygen saturation is 97%, so hypothermia, hypoxia, and shock are not likely causes of the patients restlessness.

The nurse is providing discharge teaching for a patient who developed a pulmonary embolism after total knee surgery. The patient has been converted from heparin to sodium warfarin (Coumadin) anticoagulant therapy. What should the nurse teach the client? A) Coumadin will continue to break up the clot over a period of weeks B) Coumadin must be taken concurrent with ASA to achieve anticoagulation. C) Anticoagulant therapy usually lasts between 3 and 6 months. D) He should take a vitamin supplement containing vitamin K

C Feedback: Anticoagulant therapy prevents further clot formation, but cannot be used to dissolve a clot. The therapy continues for approximately 3 to 6 months and is not combined with ASA. Vitamin K reverses the effect of anticoagulant therapy and normally should not be taken.

A nurse is teaching a patient with asthma about Azmacort, an inhaled corticosteroid. Which adverse effects should the nurse be sure to address in patient teaching? A) Dyspnea and increased respiratory secretions B) Nausea and vomiting C) Cough and oral thrush D) Fatigue and decreased level of consciousness

C Feedback: Azmacort has possible adverse effects of cough, dysphonia, oral thrush (candidiasis), and headache. In high doses, systemic effects may occur (e.g., adrenal suppression, osteoporosis, skin thinning, and easy bruising). The other listed adverse effects are not associated with this drug.

As an intraoperative nurse, you know that the patients emotional state can influence the outcome of his or her surgical procedure. How would you best reinforce the patients ability to influence outcome? A) Teach the patient strategies for distraction. B) Pair the patient with another patient who has better coping strategies. C) Incorporate cultural and religious considerations, as appropriate. D) Give the patient antianxiety medication.

C Feedback: Because the patients emotional state remains a concern, the care initiated by preoperative nurses is continued by the intraoperative nursing staff that provides the patient with information and reassurance. The nurse supports coping strategies and reinforces the patients ability to influence outcomes by encouraging active participation in the plan of care incorporating cultural, ethnic, and religious considerations, as appropriate. Buddying a patient is normally inappropriate and distraction may or may not be effective. Nonpharmacologic measures should be prioritized.

The nurse is caring for a patient after kidney surgery. The nurse is aware that bleeding is a major complication of kidney surgery and that if it goes undetected and untreated can result in hypovolemia and hemorrhagic shock in the patient. When assessing for bleeding, what assessment parameter should the nurse evaluate? Test Bank - Brunner & Suddarth's Textbook of Medical-Surgical Nursing 14e (Hinkle 2017) 1032 A) Oral intake B) Pain intensity C) Level of consciousness D) Radiation of pain

C Feedback: Bleeding is a major complication of kidney surgery. If undetected and untreated, this can result in hypovolemia and hemorrhagic shock. The nurses role is to observe for these complications, to report their signs and symptoms, and to administer prescribed parenteral fluids and blood and blood components. Monitoring of vital signs, skin condition, the urinary drainage system, the surgical incision, and the level of consciousness is necessary to detect evidence of bleeding, decreased circulating blood, and fluid volume and cardiac output. Bleeding is not normally evidenced by changes in pain or oral intake.

A student nurse is preparing to care for a patient with bronchiectasis. The student nurse should recognize that this patient is likely to experience respiratory difficulties related to what pathophysiologic process? A) Intermittent episodes of acute bronchospasm B) Alveolar distention and impaired diffusion C) Dilation of bronchi and bronchioles D) Excessive gas exchange in the bronchioles

C Feedback: Bronchiectasis is a chronic, irreversible dilation of the bronchi and bronchioles that results from destruction of muscles and elastic connective tissue. It is not characterized by acute bronchospasm, alveolar distention, or excessive gas exchange.

A patient is admitted to the ICU after a motor vehicle accident. On the second day of the hospital admission, the patient develops acute kidney injury. The patient is hemodynamically unstable, but renal replacement therapy is needed to manage the patients hypervolemia and hyperkalemia. Which of the following therapies will the patients hemodynamic status best tolerate? A) Hemodialysis B) Peritoneal dialysis C) Continuous venovenous hemodialysis (CVVHD) Test Bank - Brunner & Suddarth's Textbook of Medical-Surgical Nursing 14e (Hinkle 2017) 1024 D) Plasmapheresis

C Feedback: CVVHD facilitates the removal of uremic toxins and fluid. The hemodynamic effects of CVVHD are usually mild in comparison to hemodialysis, so CVVHD is best tolerated by an unstable patient. Peritoneal dialysis is not the best choice, as the patient may have sustained abdominal injuries during the accident and catheter placement would be risky. Plasmapheresis does not achieve fluid removal and electrolyte balance.

The nurse is caring for a patient who had a brush biopsy 12 hours ago. The presence of what assessment finding should prompt the nurse to notify the physician? A) Scant hematuria B) Renal colic Test Bank - Brunner & Suddarth's Textbook of Medical-Surgical Nursing 14e (Hinkle 2017) 1005 C) Temperature 100.2F orally D) Infiltration of the patients intravenous catheter

C Feedback: Hematuria and renal colic are common and expected findings after the performance of a renal brush biopsy. The physician should be notified of the patients body temperature, which likely indicates the onset of an infectious process. IV infiltration does not warrant notification of the primary care physician.

You are caring for a 71-year-old patient who is 4 days postoperative for bilateral inguinal hernias. The patient has a history of congestive heart failure and peptic ulcer disease. The patient is highly reluctant to ambulate and will not drink fluids except for hot tea with her meals. The nurses aide reports to you that this patients vital signs are slightly elevated and that she has a nonproductive cough. When you assess the patient, you auscultate crackles at the base of the lungs. What would you suspect is wrong with your patient? A) Pulmonary embolism B) Hypervolemia C) Hypostatic pulmonary congestion D) Malignant hyperthermia

C Feedback: Hypostatic pulmonary congestion, caused by a weakened cardiovascular system that permits stagnation of secretions at lung bases, may develop; this condition occurs most frequently in elderly patients who are not mobilized effectively. The symptoms are often vague, with perhaps a slight elevation of temperature, pulse, and respiratory rate, as well as a cough. Physical examination reveals dullness and crackles at the base of the lungs. If the condition progresses, then the outcome may be fatal. A pulmonary embolism does not have this presentation and hypervolemia is unlikely due to the patients low fluid intake. Malignant hyperthermia occurs concurrent with the administration of anesthetic.

The ED nurse is caring for an 11-year-old brought in by ambulance after having been hit by a car. The childs parents are thought to be en route to the hospital but have not yet arrived. No other family members are present and attempts to contact the parents have been unsuccessful. The child needs emergency surgery to save her life. How should the need for informed consent be addressed? A) A social worker should temporarily sign the informed consent. B) Consent should be obtained from the hospitals ethics committee. Test Bank - Brunner & Suddarth's Textbook of Medical-Surgical Nursing 14e (Hinkle 2017) 354 C) Surgery should be done without informed consent. D) Surgery should be delayed until the parents arrive.

C Feedback: In an emergency, it may be necessary for the surgeon to operate as a lifesaving measure without the patients informed consent. However, every effort must be made to contact the patients family. In such a situation, contact can be made by electronic means. In this scenario, the surgery is considered lifesaving, and the parents are on their way to the hospital and not available. A delay would be unacceptable. Neither a social worker nor a member of the ethics committee may sign.

The nurse just received a postoperative patient from the PACU to the medicalsurgical unit. The patient is an 84-year-old woman who had surgery for a left hip replacement. Which of the following concerns should the nurse prioritize for this patient in the first few hours on the unit? A) Beginning early ambulation B) Maintaining clean dressings on the surgical site C) Close monitoring of neurologic status D) Resumption of normal oral intake

C Feedback: In the initial hours after admission to the clinical unit, adequate ventilation, hemodynamic stability, incisional pain, surgical site integrity, nausea and vomiting, neurologic status, and spontaneous voiding are primary concerns. A patient who has had total hip replacement does not ambulate during the first few hours on the unit. Dressings are assessed, but may have some drainage on them. Oral intake will take more time to resume.

A client presents to the walk-in clinic complaining of a dry, irritating cough and production of a minute amount of mucus-like sputum. The patient complains of soreness in her chest in the sternal area. The nurse should suspect that the primary care provider will assess the patient for what health problem? A) Pleural effusion B) Pulmonary embolism C) Tracheobronchitis D) Tuberculosis

C Feedback: Initially, the patient with tracheobronchitis has a dry, irritating cough and expectorates a scant amount of mucoid sputum. The patient may report sternal soreness from coughing and have fever or chills, night sweats, headache, and general malaise. Pleural effusion and pulmonary embolism do not normally cause sputum production and would likely cause acute shortness of breath. Hemoptysis is characteristic of TB.

You are caring for a male patient who has had spinal anesthesia. The patient is under a physicians order to lie flat postoperatively. When the patient asks to go to the bathroom, you encourage him to adhere to the physicians order. What rationale for complying with this order should the nurse explain to the patient? A) Preventing the risk of hypotension Test Bank - Brunner & Suddarth's Textbook of Medical-Surgical Nursing 14e (Hinkle 2017) 365 B) Preventing respiratory depression C) Preventing the onset of a headache D) Preventing pain at the lumbar injection site

C Feedback: Lying flat reduces the risk of headache after spinal anesthesia. Hypotension and respiratory depression may be adverse effects of spinal anesthesia associated with the spread of the anesthetic, but lying flat does not help reduce these effects. Pain at the lumbar injection site typically is not a problem.

The nurse is reviewing the electronic health record of a patient with an empyema. What health problem in the patients history is most likely to have caused the empyema? A) Smoking B) Asbestosis C) Pneumonia D) Lung cancer

C Feedback: Most empyemas occur as complications of bacterial pneumonia or lung abscess. Cancer, smoking, and asbestosis are not noted to be common causes.

The nurse is caring for a 78-year-old man who has had an outpatient cholecystectomy. The nurse is getting him up for his first walk postoperatively. To decrease the potential for orthostatic hypotension and consequent falls, what should the nurse have the patient do? A) Sit in a chair for 10 minutes prior to ambulating. B) Drink plenty of fluids to increase circulating blood volume. C) Stand upright for 2 to 3 minutes prior to ambulating. D) Perform range-of-motion exercises for each joint.

C Feedback: Older adults are at an increased risk for orthostatic hypotension secondary to age-related changes in vascular tone. The patient should sit up and then stand for 2 to 3 minutes before ambulating to alleviate orthostatic hypotension. The nurse should assess the patients ability to mobilize safely, but full assessment of range of motion in all joints is not normally necessary. Sitting in a chair and increasing fluid intake are insufficient to prevent orthostatic hypotension and consequent falls.

The nurse is caring for acutely ill patient. What assessment finding should prompt the nurse to inform the physician that the patient may be exhibiting signs of acute kidney injury (AKI)? A) The patient is complains of an inability to initiate voiding. B) The patients urine is cloudy with a foul odor. C) The patients average urine output has been 10 mL/hr for several hours. D) The patient complains of acute flank pain.

C Feedback: Oliguria (<500 mL/d of urine) is the most common clinical situation seen in AKI. Flank pain and inability to initiate voiding are not characteristic of AKI. Cloudy, foul-smelling urine is suggestive of a urinary tract infection.

Results of a patients 24-hour urine sample indicate osmolality of 510 mOsm/kg, which is within reference range. What conclusion can the nurse draw from this assessment finding? A) The patients kidneys are capable of maintaining acidbase balance. B) The patients kidneys reabsorb most of the potassium that the patient ingests. C) The patients kidneys can produce sufficiently concentrated urine. D) The patients kidneys are producing sufficient erythropoietin.

C Feedback: Osmolality is the most accurate measurement of the kidneys ability to dilute and concentrate urine. Osmolality is not a direct indicator of renal function as it relates to erythropoietin synthesis or maintenance of acidbase balance. It does not indicate the maintenance of healthy levels of potassium, the vast majority of which is excreted.

The nurse is caring for a 79-year-old man who has returned to the postsurgical unit following abdominal surgery. The patient is unable to ambulate and is now refusing to wear an external pneumatic compression stocking. The nurse should explain that refusing to wear external pneumatic compression stockings increases his risk of what postsurgical complication? A) Sepsis Test Bank - Brunner & Suddarth's Textbook of Medical-Surgical Nursing 14e (Hinkle 2017) 381 B) Infection C) Pulmonary embolism D) Hematoma

C Feedback: Patients who have surgery that limits mobility are at an increased risk for pulmonary embolism secondary to deep vein thrombosis. The use of an external pneumatic compression stocking significantly reduces the risk by increasing venous return to the heart and limiting blood stasis. The risk of infection or sepsis would not be affected by an external pneumatic compression stocking. A hematoma or bruise would not be affected by the external pneumatic compression stocking unless the stockings were placed directly over the hematoma.

The nurse is caring for an 82-year-old female patient in the PACU. The woman begins to awaken and responds to her name, but is confused, restless, and agitated. What principle should guide the nurses subsequent assessment? A) Postoperative confusion in older adults is an indication of impaired oxygenation or possibly a stroke during surgery. B) Confusion, restlessness, and agitation are expected postoperative findings in older adults and they will diminish in time. C) Postoperative confusion is common in the older adult patent, but it could also indicate a significant blood loss. D) Confusion, restlessness, and agitation indicate an underlying cognitive deficit such as dementia.

C Feedback: Postoperative confusion is common in the older adult patient, but it could also indicate blood loss and the potential for hypovolemic shock; it is a critical symptom for the nurse to identify. Despite being common, it is not considered to be an expected finding. Postoperative confusion is an indication of an oxygen problem or possibly a stroke during surgery, but blood loss is more likely. A new onset of confusion, restlessness, and agitation does not necessarily suggest an underlying cognitive disorder.

A patient arrives in the emergency department with an attack of acute bronchiectasis. Chest auscultation reveals the presence of copious secretions. What intervention should the nurse prioritize in this patients care? A) Oral administration of diuretics B) Intravenous fluids to reduce the viscosity of secretions C) Postural chest drainage D) Pulmonary function testing

C Feedback: Postural drainage is part of all treatment plans for bronchiectasis, because draining of the bronchiectatic areas by gravity reduces the amount of secretions and the degree of infection. Diuretics and IV fluids will not aid in the mobilization of secretions. Lung function testing may be indicated, but this assessment will not relieve the patients symptoms.

The nurse is caring for a patient on the medicalsurgical unit postoperative day 5. During each patient assessment, the nurse evaluates the patient for infection. Which of the following would be most indicative of infection? A) Presence of an indwelling urinary catheter B) Rectal temperature of 99.5F (37.5C) C) Red, warm, tender incision D) White blood cell (WBC) count of 8,000/mL

C Feedback: Redness, warmth, and tenderness in the incision area should lead the nurse to suspect a postoperative infection. The presence of any invasive device predisposes a patient to infection, but by itself does not indicate infection. An oral temperature of 99.5F may not signal infection in a postoperative patient because of the inflammatory process. A normal WBC count ranges from 4,000 to 10,000/mL.

A nurse is working with a female patient who has developed stress urinary incontinence. Pelvic floor muscle exercises have been prescribed by the primary care provider. How can the nurse best promote successful treatment? A) Clearly explain the potential benefits of pelvic floor muscle exercises. B) Ensure the patient knows that surgery will be required if the exercises are unsuccessful. C) Arrange for biofeedback when the patient is learning to perform the exercises. D) Contact the patient weekly to ensure that she is performing the exercises consistently.

C Feedback: Research shows that written or verbal instruction alone is usually inadequate to teach an individual how to identify and strengthen the pelvic floor for sufficient bladder and bowel control. Biofeedback-assisted pelvic muscle exercise (PME) uses either electromyography or manometry to help the individual identify the pelvic muscles as he or she attempts to learn which muscle group is involved when performing PME. This objective assessment is likely superior to weekly contact with the patient. Surgery is not necessarily indicated if behavioral techniques are unsuccessful.

The dressing surrounding a mastectomy patients Jackson-Pratt drain has scant drainage on it. The nurse believes that the amount of drainage on the dressing may be increasing. How can the nurse best confirm this suspicion? A) Describe the appearance of the dressing in the electronic health record. B) Photograph the patients abdomen for later comparison using a smartphone. C) Trace the outline of the drainage on the dressing for future comparison. D) Remove and weigh the dressing, reapply it, and then repeat in 8 hours.

C Feedback: Spots of drainage on a dressing are outlined with a pen, and the date and time of the outline are recorded on the dressing so that increased drainage can be easily seen. A dressing is never removed and then reapplied. Photographs normally require informed consent, so they would not be used for this purpose. Documentation is necessary, but does not confirm or rule out an increase in drainage.

The nurse is caring for a patient in the ICU admitted with ARDS after exposure to toxic fumes from a hazardous spill at work. The patient has become hypotensive. What is the cause of this complication to the ARDS treatment? Test Bank - Brunner & Suddarth's Textbook of Medical-Surgical Nursing 14e (Hinkle 2017) 466 A) Pulmonary hypotension due to decreased cardiac output B) Severe and progressive pulmonary hypertension C) Hypovolemia secondary to leakage of fluid into the interstitial spaces D) Increased cardiac output from high levels of PEEP therapy

C Feedback: Systemic hypotension may occur in ARDS as a result of hypovolemia secondary to leakage of fluid into the interstitial spaces and depressed cardiac output from high levels of PEEP therapy. Pulmonary hypertension, not pulmonary hypotension, sometimes is a complication of ARDS, but it is not the cause of the patient becoming hypotensive.

The nurse is caring for a patient suspected of having renal dysfunction. When reviewing laboratory results for this patient, the nurse interprets the presence of which substances in the urine as most suggestive of pathology? A) Potassium and sodium B) Bicarbonate and urea C) Glucose and protein D) Creatinine and chloride

C Feedback: Test Bank - Brunner & Suddarth's Textbook of Medical-Surgical Nursing 14e (Hinkle 2017) 1001 The various substances normally filtered by the glomerulus, reabsorbed by the tubules, and excreted in the urine include sodium, chloride, bicarbonate, potassium, glucose, urea, creatinine, and uric acid. Within the tubule, some of these substances are selectively reabsorbed into the blood. Glucose is completely reabsorbed in the tubule and normally does not appear in the urine. However, glucose is found in the urine if the amount of glucose in the blood and glomerular filtrate exceeds the amount that the tubules are able to reabsorb. Protein molecules are also generally not found in the urine because amino acids are also filtered at the level of the glomerulus and reabsorbed so that it is not excreted in the urine.

The nurse is caring for a patient in acute kidney injury. Which of the following complications would most clearly warrant the administration of polystyrene sulfonate (Kayexalate)? A) Hypernatremia B) Hypomagnesemia C) Hyperkalemia D) Hypercalcemia

C Feedback: Test Bank - Brunner & Suddarth's Textbook of Medical-Surgical Nursing 14e (Hinkle 2017) 1020 Hyperkalemia, a common complication of acute kidney injury, is life-threatening if immediate action is not taken to reverse it. The administration of polystyrene sulfonate reduces serum potassium levels.

A patient has had her indwelling urinary catheter removed after having it in place for 10 days during recovery from an acute illness. Two hours after removal of the catheter, the patient informs the nurse that she is experiencing urinary urgency resulting in several small-volume voids. What is the nurses best response? A) Inform the patient that urgency and occasional incontinence are expected for the first few weeks post-removal. B) Obtain an order for a loop diuretic in order to enhance urine output and bladder function. C) Inform the patient that this is not unexpected in the short term and scan the patients bladder following each void. D) Obtain an order to reinsert the patients urinary catheter and attempt removal in 24 to 48 hours.

C Feedback: Test Bank - Brunner & Suddarth's Textbook of Medical-Surgical Nursing 14e (Hinkle 2017) 1048 Immediately after the indwelling catheter is removed, the patient is placed on a timed voiding schedule, usually every 2 to 3 hours. At the given time interval, the patient is instructed to void. The bladder is then scanned using a portable ultrasonic bladder scanner; if the bladder has not emptied completely, straight catheterization may be performed. An indwelling catheter would not be reinserted to resolve the problem and diuretics would not be beneficial. Ongoing incontinence is not an expected finding after catheter removal.

The intraoperative nurse is transferring a patient from the OR to the PACU after replacement of the right knee. The patient is a 73-year-old woman. The nurse should prioritize which of the following actions? A) Keeping the patient sterile B) Keeping the patient restrained C) Keeping the patient warm D) Keeping the patient hydrated

C Feedback: Test Bank - Brunner & Suddarth's Textbook of Medical-Surgical Nursing 14e (Hinkle 2017) 388 Special attention is given to keeping the patient warm because elderly patients are more susceptible to hypothermia. It is all important for the nurse to pay attention to hydration, but hypovolemia does not occur as quickly as hypothermia. The patient is never sterile and restraints are very rarely necessary.

A patients severe asthma has necessitated the use of a long-acting beta2-agonist (LABA). Which of the patients statements suggests a need for further education? A) I know that these drugs can sometimes make my heart beat faster. B) Ive heard that this drug is particularly good at preventing asthma attacks during exercise. C) Ill make sure to use this each time I feel an asthma attack coming on. D) Ive heard that this drug sometimes gets less effective over time.

C Feedback: Test Bank - Brunner & Suddarth's Textbook of Medical-Surgical Nursing 14e (Hinkle 2017) 486 LABAs are not used for management of acute asthma symptoms. Tachycardia is a potential adverse effect and decreased protection against exercise-induced bronchospasm may occur with regular use.

The OR nurse is providing care for a 25-year-old major trauma patient who has been involved in a motorcycle accident. The nurse should know that the patient is at increased risk for what complication of surgery? A) Respiratory depression B) Hypothermia C) Anesthesia awareness D) Moderate sedation

C Feedback: The Joint Commission has issued an alert regarding the phenomenon of patients being partially awake while under general anesthesia (referred to as anesthesia awareness). Patients at greatest risk of anesthesia awareness are cardiac, obstetric, and major trauma patients. This patient does not likely face a heightened risk of respiratory depression or hypothermia. Moderate sedation is not a complication.

A patient is scheduled for a bowel resection in the morning and the patients orders include a cleansing enema tonight. The patient wants to know why this is necessary. The nurse should explain that the cleansing enema will have what therapeutic effect? A) Preventing aspiration of gastric contents B) Preventing the accumulation of abdominal gas postoperatively Test Bank - Brunner & Suddarth's Textbook of Medical-Surgical Nursing 14e (Hinkle 2017) 347 C) Preventing potential contamination of the peritoneum D) Facilitating better absorption of medications

C Feedback: The administration of a cleansing enema will allow for satisfactory visualization of the surgical site and to prevent trauma to the intestine or contamination of the peritoneum by feces. It will have no effect on aspiration of gastric contents or the absorption of medications. The patient should expect to develop gas in the postoperative period.

A patient has been admitted to the postsurgical unit following the creation of an ileal conduit. What should the nurse measure to determine the size of the appliance needed? A) The circumference of the stoma B) The narrowest part of the stoma C) The widest part of the stoma D) Half the width of the stoma

C Feedback: The correct appliance size is determined by measuring the widest part of the stoma with a ruler. The permanent appliance should be no more than 1.6 mm (1/8 inch) larger than the diameter of the stoma and the same shape as the stoma to prevent contact of the skin with drainage.

The nurse is performing a focused genitourinary and renal assessment of a patient. Where should the nurse assess for pain at the costovertebral angle? A) At the umbilicus and the right lower quadrant of the abdomen B) At the suprapubic region and the umbilicus C) At the lower border of the 12th rib and the spine D) At the 7th rib and the xyphoid process

C Feedback: The costovertebral angle is the angle formed by the lower border of the 12th rib and the spine. Renal Test Bank - Brunner & Suddarth's Textbook of Medical-Surgical Nursing 14e (Hinkle 2017) 1002 dysfunction may produce tenderness over the costovertebral angle.

A nurse is caring for a patient who is in the diuresis phase of AKI. The nurse should closely monitor the patient for what complication during this phase? A) Hypokalemia B) Hypocalcemia C) Dehydration Test Bank - Brunner & Suddarth's Textbook of Medical-Surgical Nursing 14e (Hinkle 2017) 1028 D) Acute flank pain

C Feedback: The diuresis period is marked by a gradual increase in urine output, which signals that gl

The nurse is caring for a patient who is receiving oxygen therapy for pneumonia. How should the nurse best assess whether the patient is hypoxemic? A) Assess the patients level of consciousness (LOC). B) Assess the patients extremities for signs of cyanosis. C) Assess the patients oxygen saturation level. D) Review the patients hemoglobin, hematocrit, and red blood cell levels.

C Feedback: The effectiveness of the patients oxygen therapy is assessed by the ABG analysis or pulse oximetry. ABG results may not be readily available. Presence or absence of cyanosis is not an accurate indicator of oxygen effectiveness. The patients LOC may be affected by hypoxia, but not every change in LOC is Test Bank - Brunner & Suddarth's Textbook of Medical-Surgical Nursing 14e (Hinkle 2017) 458 related to oxygenation. Hemoglobin, hematocrit, and red blood cell levels do not directly reflect current oxygenation status.

The nurse is caring for an 82-year-old patient with a diagnosis of tracheobronchitis. The patient begins complaining of right-sided chest pain that gets worse when he coughs or breathes deeply. Vital signs are Test Bank - Brunner & Suddarth's Textbook of Medical-Surgical Nursing 14e (Hinkle 2017) 465 within normal limits. What would you suspect this patient is experiencing? A) Traumatic pneumothorax B) Empyema C) Pleuritic pain D) Myocardial infarction

C Feedback: The key characteristic of pleuritic pain is its relationship to respiratory movement. Taking a deep breath, coughing, or sneezing worsens the pain. Pleuritic pain is limited in distribution rather than diffuse; it usually occurs only on one side. The pain may become minimal or absent when the breath is held. It may be localized or radiate to the shoulder or abdomen. Later, as pleural fluid develops, the pain decreases. The scenario does not indicate any trauma to the patient, so a traumatic pneumothorax is implausible. Empyema is unlikely as there is no fever indicative of infection. Myocardial infarction would affect the patients vital signs profoundly. 28. A patient with thoracic trauma is admitted to the ICU. The nurse notes the patients chest and neck are swollen and there is a crackling sensation when palpated. The nurse consequently identifies the presence of subcutaneous emphysema. If this condition becomes severe and threatens airway patency, what intervention is indicated? A) A chest tube B) A tracheostomy C) An endotracheal tube D) A feeding tube Ans: B Feedback: In severe cases in which there is widespread subcutaneous emphysema, a tracheostomy is indicated if airway patency is threatened by pressure of the trapped air on the trachea. The other listed tubes would neither resolve the subcutaneous emphysema nor the consequent airway constriction. 29. The nurse is caring for a patient in the ICU admitted with ARDS after exposure to toxic fumes from a hazardous spill

The nurse is creating an education plan for a patient who underwent a nephrectomy for the treatment of a renal tumor. What should the nurse include in the teaching plan? A) The importance of increased fluid intake B) Signs and symptoms of rejection C) Inspection and care of the incision D) Techniques for preventing metastasis

C Feedback: The nurse teaches the patient to inspect and care for the incision and perform other general postoperative care, including activity and lifting restrictions, driving, and pain management. There would be no need to teach the signs or symptoms of rejection as there has been no transplant. Increased fluid intake is not normally recommended and the patient has minimal control on the future risk for metastasis.

The nurse is caring for a trauma victim in the ED who will require emergency surgery due to injuries. Before the patient leaves the ED for the OR, the patient goes into cardiac arrest. The nurse assists in the successful resuscitation and proceeds to release the patient to the OR staff. When can the ED nurse Test Bank - Brunner & Suddarth's Textbook of Medical-Surgical Nursing 14e (Hinkle 2017) 351 perform the preoperative assessment? A) When he or she has the opportunity to review the patients electronic health record B) When the patient arrives in the OR C) When assisting with the resuscitation D) Preoperative assessment is not necessary in this case

C Feedback: The only opportunity for preoperative assessment may take place at the same time as resuscitation in the ED. Preoperative assessment is necessary, but the nurse could not normally enter the OR to perform this assessment. The health record is an inadequate data source.

In anticipation of a patients scheduled surgery, the nurse is teaching her to perform deep breathing and coughing to use postoperatively. What action should the nurse teach the patient? Test Bank - Brunner & Suddarth's Textbook of Medical-Surgical Nursing 14e (Hinkle 2017) 339 A) The patient should take three deep breaths and cough hard three times, at least every 15 minutes for the immediately postoperative period. B) The patient should take three deep breaths and exhale forcefully and then take a quick short breath and cough from deep in the lungs. C) The patient should take a deep breath in through the mouth and exhale through the mouth, take a short breath, and cough from deep in the lungs. D) The patient should rapidly inhale, hold for 30 seconds or as long as possible, and exhale slowly.

C Feedback: The patient assumes a sitting position to enhance lung expansion. The nurse then demonstrates how to take a deep, slow breath and how to exhale slowly. After practicing deep breathing several times, the patient is instructed to breathe deeply, exhale through the mouth, take a short breath, and cough from deep in the lungs.

A female patient has been experiencing recurrent urinary tract infections. What health education should the nurse provide to this patient? A) Bathe daily and keep the perineal region clean. B) Avoid voiding immediately after sexual intercourse. C) Drink liberal amounts of fluids. D) Void at least every 6 to 8 hours.

C Feedback: The patient is encouraged to drink liberal amounts of fluids (water is the best choice) to increase urine production and flow, which flushes the bacteria from the urinary tract. Frequent voiding (every 2 to 3 hours) is encouraged to empty the bladder completely because this can significantly lower urine bacterial counts, reduce urinary stasis, and prevent reinfection. The patient should be encouraged to shower rather than bathe.

The nurse is caring for a patient who underwent percutaneous lithotripsy earlier in the day. What instruction should the nurse give the patient? A) Limit oral fluid intake for 1 to 2 days. B) Report the presence of fine, sand like particles through the nephrostomy tube. C) Notify the physician about cloudy or foul-smelling urine. D) Report any pink-tinged urine within 24 hours after the procedure.

C Feedback: The patient should report the presence of foul-smelling or cloudy urine since this is suggestive of a UTI. Unless contraindicated, the patient should be instructed to drink large quantities of fluid each day to flush the kidneys. Sand like debris is normal due to residual stone products. Hematuria is common after lithotripsy.

The nurse is doing a preoperative assessment of an 87-year-old man who is slated to have a right lung lobe resection to treat lung cancer. What underlying principle should guide the nurses preoperative assessment of an elderly patient? A) Elderly patients have a smaller lung capacity than younger patients. B) Elderly patients require higher medication doses than younger patients. C) Elderly patients have less physiologic reserve than younger patients. D) Elderly patients have more sophisticated coping skills than younger patients.

C Feedback: The underlying principle that guides the preoperative assessment, surgical care, and postoperative care is that elderly patients have less physiologic reserve (the ability of an organ to return to normal after a disturbance in its equilibrium) than do younger patients. Elderly patients do not have larger lung capacities than younger patients. Elderly patients cannot necessarily cope better than younger patients and they often require lower doses of medications.

The nurse is caring for a patient who is experiencing pain and anxiety following his prostatectomy. Which intervention will likely best assist in decreasing the patients pain and anxiety? A) Administration of NSAIDs rather than opioids B) Allowing the patient to increase activity C) Use of guided imagery along with pain medication D) Use of deep breathing and coughing exercises

C Feedback: The use of guided imagery will enhance pain relief and assist in reduction of anxiety. It may be combined with analgesics. Deep breathing and the increase in activity may produce increased pain. Replacing opioids with NSAIDs may cause an increase in pain.

The circulating nurse will be participating in a 78-year-old patients total hip replacement. Which of the following considerations should the nurse prioritize during the preparation of the patient in the OR? A) The patient should be placed in Trendelenburg position. B) The patient must be firmly restrained at all times. C) Pressure points should be assessed and well padded. D) The preoperative shave should be done by the circulating nurse.

C Feedback: The vascular supply should not be obstructed by an awkward position or undue pressure on a body part. During surgical procedures, the patient is at risk for impairment of skin integrity due to a stationary position and immobility. An elderly patient is at an increased risk of injury and impaired skin integrity. A Trendelenburg position is not indicated for this patient. Once anesthetized for a total hip replacement, the patient cannot move; restraints are not necessary. A preoperative shave is not performed; excess hair is removed by means of a clipper.

The nurse in the ED is caring for a man who has returned to the ED 4 days after receiving stitches for a knife wound on his hand. The wound is now infected, so the stitches were removed, and the wound is cleaned and packed with gauze. The ED doctor plans to have the man return tomorrow to remove the packing and resuture the wound. You are aware that the wound will now heal by what means? A) Late intention B) Second intention C) Third intention D) First intention

C Feedback: Third-intention healing or secondary suture is used for deep wounds that either had not been sutured early or that had the suture break down and are resutured later, which is what has happened in this case. Secondary suture brings the two opposing granulation surfaces back together; however, this usually results in a deeper and wider scar. These wounds are also packed postoperatively with moist gauze and covered with a dry, sterile dressing. Late intention is a term that sounds plausible, but is not used in practice. Second intention is when the wound is left open and the wound is filled with granular tissue. First intention wounds are wounds made aseptically with a minimum of tissue destruction.

The nurse is caring for a patient with an indwelling urinary catheter. The nurse is aware that what nursing action helps prevent infection in a patient with an indwelling catheter? A) Vigorously clean the meatus area daily. B) Apply powder to the perineal area twice daily. C) Empty the drainage bag at least every 8 hours. D) Irrigate the catheter every 8 hours with normal saline.

C Feedback: To reduce the risk of bacterial proliferation, the nurse should empty the collection bag at least every 8 hours through the drainage spout, and more frequently if there is a large volume of urine. Vigorous cleaning of the meatus while the catheter is in place is discouraged, because the cleaning action can move the catheter, increasing the risk of infection. The spout (or drainage port) of any urinary drainage bag can become contaminated when opened to drain the bag. Irrigation of the catheter opens the closed system, increasing the likelihood of infection.

The nurse is creating a care plan for a patient who is status post-total laryngectomy. Much of the plan consists of a long-term postoperative communication plan for alaryngeal communication. What form of alaryngeal communication will likely be chosen? A) Esophageal speech B) Electric larynx C) Tracheoesophageal puncture Test Bank - Brunner & Suddarth's Textbook of Medical-Surgical Nursing 14e (Hinkle 2017) 451 D) American sign language (ASL)

C Feedback: Tracheoesophageal puncture is simple and has few complications. It is associated with high phonation success, good phonation quality, and steady long-term results. As a result, it is preferred over esophageal speech, and electric larynx or ASL.

A patient admitted to the medical unit with impaired renal function is complaining of severe, stabbing pain in the flank and lower abdomen. The patient is being assessed for renal calculi. The nurse recognizes that the stone is most likely in what anatomic location? A) Meatus B) Bladder C) Ureter D) Urethra

C Feedback: Ureteral pain is characterized as a dull continuous pain that may be intense with voiding. The pain may be described as sharp or stabbing if the bladder is full. This type of pain is inconsistent with a stone being present in the bladder. Stones are not normally situated in the urethra or meatus.

The nurse is caring for a patient with a nursing diagnosis of deficient fluid volume. The nurses assessment reveals a BP of 98/52 mm Hg. The nurse should recognize that the patients kidneys will compensate by secreting what substance? A) Antidiuretic hormone (ADH) B) Aldosterone C) Renin D) Angiotensin

C Feedback: When the vasa recta detect a decrease in BP, specialized juxtaglomerular cells near the afferent arteriole, distal tubule, and efferent arteriole secrete the hormone renin. Renin converts angiotensinogen to angiotensin I, which is then converted to angiotensin II. The vasoconstriction causes the BP to increase. The adrenal cortex secretes aldosterone in response to stimulation by the pituitary gland, which in turn is in response to poor perfusion or increasing serum osmolality. The result is an increase in BP.

The nurse is performing wound care on a 68-year-old postsurgical patient. Which of the following practices violates the principles of surgical asepsis? A) Holding sterile objects above the level of the nurses waist B) Considering a 1 inch (2.5 cm) edge around the sterile field as being contaminated C) Pouring solution onto a sterile field cloth Test Bank - Brunner & Suddarth's Textbook of Medical-Surgical Nursing 14e (Hinkle 2017) 366 D) Opening the outermost flap of a sterile package away from the body

C Feedback: Whenever a sterile barrier is breached, the area must be considered contaminated. Pouring solution onto a sterile field cloth violates surgical asepsis because moisture penetrating the cloth can carry microorganisms to the sterile field via capillary action. The other options are practices that help ensure surgical asepsis.

The home health nurse is caring for a postoperative patient who was discharged home on day 2 after surgery. The nurse is performing the initial visit on the patients postoperatative day 2. During the visit, the nurse will assess for wound infection. For most patients, what is the earliest postoperative day that a wound infection becomes evident? A) Day 9 B) Day 7 C) Day 5 D) Day 3

C Feedback: Wound infection may not be evident until at least postoperative day 5. This makes the other options incorrect.

A patient is complaining of genitourinary pain shortly after returning to the unit from a scheduled cystoscopy. What intervention should the nurse perform? A) Encourage mobilization. B) Apply topical lidocaine to the patients meatus, as ordered. C) Apply moist heat to the patients lower abdomen. D) Apply an ice pack to the patients perineum.

C Test Bank - Brunner & Suddarth's Textbook of Medical-Surgical Nursing 14e (Hinkle 2017) 1006 Feedback: Following cystoscopy, moist heat to the lower abdomen and warm sitz baths are helpful in relieving pain and relaxing the muscles. Ice, lidocaine, and mobilization are not recommended interventions.

A nurse is aware of the high incidence and prevalence of fluid volume deficit among older adults. What related health education should the nurse provide to an older adult? A) If possible, try to drink at least 4 liters of fluid daily. B) Ensure that you avoid replacing water with other beverages. C) Remember to drink frequently, even if you dont feel thirsty. D) Make sure you eat plenty of salt in order to stimulate thirst. Ans:

C Test Bank - Brunner & Suddarth's Textbook of Medical-Surgical Nursing 14e (Hinkle 2017) 1009 Feedback: The nurse emphasizes the need to drink throughout the day even if the patient does not feel thirsty, because the thirst stimulation is decreased. Four liters of daily fluid intake is excessive and fluids other than water are acceptable in most cases. Additional salt intake is not recommended as a prompt for increased fluid intake.

A school nurse is caring for a 10-year-old girl who is having an asthma attack. What is the preferred intervention to alleviate this clients airflow obstruction? A) Administer corticosteroids by metered dose inhaler B) Administer inhaled anticholinergics C) Administer an inhaled beta-adrenergic agonist D) Utilize a peak flow monitoring device

C Test Bank - Brunner & Suddarth's Textbook of Medical-Surgical Nursing 14e (Hinkle 2017) 475 Feedback: Asthma exacerbations are best managed by early treatment and education of the patient. Quick-acting beta-adrenergic medications are the first used for prompt relief of airflow obstruction. Systemic corticosteroids may be necessary to decrease airway inflammation in patients who fail to respond to inhaled beta-adrenergic medication. A peak flow device will not resolve short-term shortness of breath.

A patient has been admitted to the medical unit with a diagnosis of ureteral colic secondary to urolithiasis. When planning the patients admission assessment, the nurse should be aware of the signs and symptoms that are characteristic of this diagnosis? Select all that apply. A) Diarrhea B) High fever C) Hematuria D) Urinary frequency E) Acute pain

C, D, E Feedback: Stones lodged in the ureter (ureteral obstruction) cause acute, excruciating, colicky, wavelike pain, Test Bank - Brunner & Suddarth's Textbook of Medical-Surgical Nursing 14e (Hinkle 2017) 1049 radiating down the thigh and to the genitalia. Often, the patient has a desire to void, but little urine is passed, and it usually contains blood because of the abrasive action of the stone. This group of symptoms is called ureteral colic. Diarrhea is not associated with this presentation and a fever is usually absent due to the noninfectious nature of the health problem.

The PACU nurse is caring for a patient who has been deemed ready to go to the postsurgical floor after Test Bank - Brunner & Suddarth's Textbook of Medical-Surgical Nursing 14e (Hinkle 2017) 350 her surgery. What would the PACU nurse be responsible for reporting to the nurse on the floor? Select all that apply. A) The names of the anesthetics that were used B) The identities of the staff in the OR C) The patients preoperative level of consciousness D) The presence of family and/or significant others E) The patients full name

C, D, E Feedback: The PACU nurse is responsible for informing the floor nurse of the patients intraoperative factors (e.g., insertion of drains or catheters, administration of blood or medications during surgery, or occurrence of unexpected events), preoperative level of consciousness, presence of family and/or significant others, and identification of the patient by name. The PACU nurse does not tell which anesthetic was used, only the type and amount used. The PACU nurse does not identify the staff that was in the OR with the patient. 27. A 77-year-old mans

A nurse is providing health education to the family of a patient with bronchiectasis. What should the nurse teach the patients family members? A) The correct technique for chest palpation and auscultation B) Techniques for assessing the patients fluid balance C) The technique for providing deep nasotracheal suctioning D) The correct technique for providing postural drainage

D Feedback: A focus of the care of bronchiectasis is helping patients clear pulmonary secretions; consequently, patients and families are taught to perform postural drainage. Chest palpation and auscultation and assessment of fluid balance are not prioritized over postural drainage. Nasotracheal suctioning is not normally necessary.

nursing is planning the care of a patient with emphysema who will soon be discharged. What teaching should the nurse prioritize in the plan of care? A) Taking prophylactic antibiotics as ordered B) Adhering to the treatment regimen in order to cure the disease C) Avoiding airplanes, buses, and other crowded public places D) Setting realistic short-term and long-range goals

D Feedback: A major area of teaching involves setting and accepting realistic short-term and long-range goals. Emphysema is not considered curable and antibiotics are not used on a preventative basis. The patient does not normally need to avoid public places.

A patient is postoperative day 3 following the creation of an ileal conduit for the treatment of invasive bladder cancer. The patient is quickly learning to self-manage the urinary diversion, but expresses concern about the presence of mucus in the urine. What is the nurses most appropriate response? A) Report this finding promptly to the primary care provider. B) Obtain a sterile urine sample and send it for culture. C) Obtain a urine sample and check it for pH. D) Reassure the patient that this is an expected phenomenon.

D Feedback: Because mucous membrane is used in forming the conduit, the patient may excrete a large amount of mucus mixed with urine. This causes anxiety in many patients. To help relieve this anxiety, the nurse reassures the patient that this is a normal occurrence after an ileal conduit procedure. Urine testing for culture or pH is not required.

A female patients most recent urinalysis results are suggestive of bacteriuria. When assessing this patient, the nurses data analysis should be informed by what principle? A) Most UTIs in female patients are caused by viruses and do not cause obvious symptoms. B) A diagnosis of bacteriuria requires three consecutive positive results. C) Urine contains varying levels of healthy bacterial flora. Test Bank - Brunner & Suddarth's Textbook of Medical-Surgical Nursing 14e (Hinkle 2017) 1037 D) Urine samples are frequently contaminated by bacteria normally present in the urethral area.

D Feedback: Because urine samples (especially in women) are commonly contaminated by the bacteria normally present in the urethral area, a bacterial count exceeding 105 colonies/mL of clean-catch, midstream urine is the measure that distinguishes true bacteriuria from contamination. A diagnosis does not require three consecutive positive results and urine does not contain a normal flora in the absence of a UTI. Most UTIs have a bacterial etiology.

The nurse is caring for a patient with a history of systemic lupus erythematosus who has been recently diagnosed with end-stage kidney disease (ESKD). The patient has an elevated phosphorus level and has Test Bank - Brunner & Suddarth's Textbook of Medical-Surgical Nursing 14e (Hinkle 2017) 1016 been prescribed calcium acetate to bind the phosphorus. The nurse should teach the patient to take the prescribed phosphorus-binding medication at what time? A) Only when needed B) Daily at bedtime C) First thing in the morning D) With each meal

D Feedback: Both calcium carbonate and calcium acetate are medications that bind with the phosphate and assist in excreting the phosphate from the body, in turn lowering the phosphate levels. Phosphate-binding medications must be administered with food to be effective.

A nurse is documenting the results of assessment of a patient with bronchiectasis. What would the nurse most likely include in documentation? A) Sudden onset of pleuritic chest pain B) Wheezes on auscultation C) Increased anterior-posterior (A-P) diameter D) Clubbing of the fingers

D Feedback: Characteristic symptoms of bronchiectasis include chronic cough and production of purulent sputum in copious amounts. Clubbing of the fingers also is common because of respiratory insufficiency. Sudden pleuritic chest pain is a common manifestation of a pulmonary embolism. Wheezes on auscultation are common in patients with asthma. An increased A-P diameter is noted in patients with COPD.

The nurses aide notifies the nurse that a patient has decreased oxygen saturation levels. The nurse Test Bank - Brunner & Suddarth's Textbook of Medical-Surgical Nursing 14e (Hinkle 2017) 390 assesses the patient and finds that he is tachypnic, has crackles on auscultation, and his sputum is frothy and pink. The nurse should suspect what complication? A) Pulmonary embolism B) Atelectasis C) Laryngospasm D) Flash pulmonary edema

D Feedback: Flash pulmonary edema occurs when protein and fluid accumulate in the alveoli unrelated to elevated pulmonary artery occlusive pressure. Signs and symptoms include agitation; tachypnea; tachycardia; decreased pulse oximetry readings; frothy, pink sputum; and crackles on auscultation. Laryngospasm does not cause crackles or frothy, pink sputum. The patient with atelectasis has decreased breath sounds over the affected area; the scenario does not indicate this. A pulmonary embolism does not cause this symptomatology.

The nurse caring for a patient recently diagnosed with lung disease encourages the patient not to smoke. What is the primary rationale behind this nursing action? A) Smoking decreases the amount of mucus production. B) Smoke particles compete for binding sites on hemoglobin. Test Bank - Brunner & Suddarth's Textbook of Medical-Surgical Nursing 14e (Hinkle 2017) 455 C) Smoking causes atrophy of the alveoli. D) Smoking damages the ciliary cleansing mechanism.

D Feedback: In addition to irritating the mucous cells of the bronchi and inhibiting the function of alveolar macrophage (scavenger) cells, smoking damages the ciliary cleansing mechanism of the respiratory tract. Smoking also increases the amount of mucus production and distends the alveoli in the lungs. It reduces the oxygen-carrying capacity of hemoglobin, but not by directly competing for binding sites.

A geriatric nurse is performing an assessment of body systems on an 85-year-old patient. The nurse should be aware of what age-related change affecting the renal or urinary system? A) Increased ability to concentrate urine B) Increased bladder capacity C) Urinary incontinence Test Bank - Brunner & Suddarth's Textbook of Medical-Surgical Nursing 14e (Hinkle 2017) 999 D) Decreased glomerular filtration rate

D Feedback: Many age-related changes in the renal and urinary systems should be taken into consideration when taking a health history of the older adult. One change includes a decreased glomerular surface area resulting in a decreased glomerular filtration rate. Other changes include the decreased ability to concentrate urine and a decreased bladder capacity. It also should be understood that urinary incontinence is not a normal age-related change, but is common in older adults, especially in women because of the loss of pelvic muscle tone.

An older adult has experienced a new onset of urinary incontinence and family members identify this problem as being unprecedented. When assessing the patient for factors that may have contributed to incontinence, the nurse should prioritize what assessment? A) Reviewing the patients 24-hour food recall for changes in diet B) Assessing for recent contact with individuals who have UTIs C) Assessing for changes in the patients level of psychosocial stress Test Bank - Brunner & Suddarth's Textbook of Medical-Surgical Nursing 14e (Hinkle 2017) 1046 D) Reviewing the patients medication administration record for recent changes

D Feedback: Many medications affect urinary continence in addition to causing other unwanted or unexpected effects. Stress and dietary changes could potentially affect the patients continence, but medications are more frequently causative of incontinence. UTIs can cause incontinence, but these infections do not result from contact with infected individuals.

A nurses colleague has applied an incontinence pad to an older adult patient who has experienced occasional episodes of functional incontinence. What principle should guide the nurses management of urinary incontinence in older adults? A) Diuretics should be promptly discontinued when an older adult experiences incontinence. B) Restricting fluid intake is recommended for older adults experiencing incontinence. C) Urinary catheterization is a first-line treatment for incontinence in older adults with incontinence. D) Urinary incontinence is not considered a normal consequence of aging.

D Feedback: Nursing management is based on the premise that incontinence is not inevitable with illness or aging and that it is often reversible and treatable. Diuretics cannot always be safely discontinued. Fluid restriction and catheterization are not considered to be safe, first-line interventions for the treatment of incontinence.

A patient being treated in the hospital has been experiencing occasional urinary retention. What nursing action should the nurse take to encourage a patient who is having difficulty voiding? A) Use a slipper bedpan. B) Apply a cold compress to the perineum. C) Have the patient lie in a supine position. Test Bank - Brunner & Suddarth's Textbook of Medical-Surgical Nursing 14e (Hinkle 2017) 1041 D) Provide privacy for the patient.

D Feedback: Nursing measures to encourage normal voiding patterns include providing privacy, ensuring an environment and body position conducive to voiding, and assisting the patient with the use of the bathroom or bedside commode, rather than a bedpan, to provide a more natural setting for voiding. Most people find supine positioning not conducive to voiding.

The nurse is providing teaching about tissue repair and wound healing to a patient who has a leg ulcer. Which of the following statements by the patient indicates that teaching has been effective? A) Ill make sure to limit my intake of protein. B) Ill make sure that the bandage is wrapped tightly. C) My foot should feel cool or cold while my legs healing. D) Ill eat plenty of fruits and vegetables.

D Feedback: Optimal nutritional status is important for wound healing; the patient should eat plenty of fruits and vegetables and not reduce protein intake. To avoid impeding circulation to the area, the bandage should be secure but not tight. If the patients foot feels cold, circulation is impaired, which inhibits wound healing.

The nurse performing the health interview of a patient with a new onset of periorbital edema has completed a genogram, noting the health history of the patients siblings, parents, and grandparents. This assessment addresses the patients risk of what kidney disorder? Test Bank - Brunner & Suddarth's Textbook of Medical-Surgical Nursing 14e (Hinkle 2017) 1027 A) Nephritic syndrome B) Acute glomerulonephritis C) Nephrotic syndrome D) Polycystic kidney disease (PKD)

D Feedback: PKD is a genetic disorder characterized by the growth of numerous cysts in the kidneys. Nephritic syndrome, acute glomerulonephritis, and nephrotic syndrome are not genetic disorders.

A nurse is explaining to a patient with asthma what her new prescription for prednisone is used for. What would be the most accurate explanation that the nurse could give? A) To ensure long-term prevention of asthma exacerbations B) To cure any systemic infection underlying asthma attacks Test Bank - Brunner & Suddarth's Textbook of Medical-Surgical Nursing 14e (Hinkle 2017) 488 C) To prevent recurrent pulmonary infections D) To gain prompt control of inadequately controlled, persistent asthma

D Feedback: Prednisone is used for a short-term (310 days) burst to gain prompt control of inadequately controlled, persistent asthma. It is not used to treat infection or to prevent exacerbations in the long term.

The nurse is caring for a patient who is going to have an open renal biopsy. What would be an important nursing action in preparing this patient for the procedure? A) Discuss the patients diagnosis with the family. B) Bathe the patient before the procedure with antiseptic skin wash. C) Administer antivirals before sending the patient for the procedure. D) Keep the patient NPO prior to the procedure.

D Feedback: Preparation for an open biopsy is similar to that for any major abdominal surgery. When preparing the patient for an open biopsy you would keep the patient NPO. You may discuss the diagnosis with the family, but that is not a preparation for the procedure. A pre-procedure wash is not normally ordered and antivirals are not administered in anticipation of a biopsy.

A patient is having pulmonary-function studies performed. The patient performs a spirometry test, revealing an FEV1/FVC ratio of 60%. How should the nurse interpret this assessment finding? A) Strong exercise tolerance B) Exhalation volume is normal C) Respiratory infection D) Obstructive lung disease

D Feedback: Spirometry is used to evaluate airflow obstruction, which is determined by the ratio of forced expiration volume in 1 second to forced vital capacity. Obstructive lung disease is apparent when an FEV1/FVC ratio is less than 70%.

An 87-year-old patient has been hospitalized with pneumonia. Which nursing action would be a priority in this patients plan of care? Test Bank - Brunner & Suddarth's Textbook of Medical-Surgical Nursing 14e (Hinkle 2017) 471 A) Nasogastric intubation B) Administration of probiotic supplements C) Bedrest D) Cautious hydration

D Feedback: Supportive treatment of pneumonia in the elderly includes hydration (with caution and with frequent assessment because of the risk of fluid overload in the elderly); supplemental oxygen therapy; and assistance with deep breathing, coughing, frequent position changes, and early ambulation. Mobility is not normally discouraged and an NG tube is not necessary in most cases. Probiotics may or may not be prescribed for the patient.

A 54-year-old man has just been diagnosed with small cell lung cancer. The patient asks the nurse why the doctor is not offering surgery as a treatment for his cancer. What fact about lung cancer treatment should inform the nurses response? A) The cells in small cell cancer of the lung are not large enough to visualize in surgery. B) Small cell lung cancer is self-limiting in many patients and surgery should be delayed. C) Patients with small cell lung cancer are not normally stable enough to survive surgery. D) Small cell cancer of the lung grows rapidly and metastasizes early and extensively.

D Feedback: Surgery is primarily used for NSCLCs, because small cell cancer of the lung grows rapidly and metastasizes early and extensively. Difficult visualization and a patients medical instability are not the limiting factors. Lung cancer is not a self-limiting disease.

A 15-year-old is admitted to the renal unit with a diagnosis of postinfectious glomerular disease. The nurse should recognize that this form of kidney disease may have been precipitated by what event? A) Psychosocial stress B) Hypersensitivity to an immunization C) Menarche D) Streptococcal infection

D Feedback: Test Bank - Brunner & Suddarth's Textbook of Medical-Surgical Nursing 14e (Hinkle 2017) 1026 Postinfectious causes of postinfectious glomerular disease are group A beta-hemolytic streptococcal infection of the throat that precedes the onset of glomerulonephritis by 2 to 3 weeks. Menarche, stress, and hypersensitivity are not typical causes.

A patient in the ICU is status post embolectomy after a pulmonary embolus. What assessment parameter does the nurse monitor most closely on a patient who is postoperative following an embolectomy? A) Pupillary response B) Pressure in the vena cava C) White blood cell differential D) Pulmonary arterial pressure

D Feedback: Test Bank - Brunner & Suddarth's Textbook of Medical-Surgical Nursing 14e (Hinkle 2017) 470 If the patient has undergone surgical embolectomy, the nurse measures the patients pulmonary arterial pressure and urinary output. Pressure is not monitored in a patients vena cava. White cell levels and pupillary responses would be monitored, but not to the extent of the patients pulmonary arterial pressure.

A patient with chronic kidney disease has been hospitalized and is receiving hemodialysis on a scheduled basis. The nurse should include which of the following actions in the plan of care? A) Ensure that the patient moves the extremity with the vascular access site as little as possible. B) Change the dressing over the vascular access site at least every 12 hours. C) Utilize the vascular access site for infusion of IV fluids. D) Assess for a thrill or bruit over the vascular access site each shift.

D Feedback: The bruit, or thrill, over the venous access site must be evaluated at least every shift. Frequent dressing changes are unnecessary and the patient does not normally need to immobilize the site. The site must not be used for purposes other than dialysis.

The home care nurse is monitoring a patient discharged home after resolution of a pulmonary embolus. For what potential complication would the home care nurse be most closely monitoring this patient? A) Signs and symptoms of pulmonary infection B) Swallowing ability and signs of aspiration C) Activity level and role performance D) Residual effects of compromised oxygenation

D Feedback: The home care nurse should monitor the patient for residual effects of the PE, which involved a severe disruption in respiration and oxygenation. PE has a noninfectious etiology; pneumonia is not impossible, but it is a less likely sequela. Swallowing ability is unlikely to be affected; activity level is important, but secondary to the effects of deoxygenation.

A patient has presented with signs and symptoms that are characteristic of acute kidney injury, but preliminary assessment reveals no obvious risk factors for this health problem. The nurse should recognize the need to interview the patient about what topic? A) Typical diet B) Allergy status C) Psychosocial stressors D) Current medication use

D Feedback: The kidneys are susceptible to the adverse effects of medications because they are repeatedly exposed to substances in the blood. Nephrotoxic medications are a more likely cause of AKI than diet, allergies, or stress.

A 90-year-old female patient is scheduled to undergo a partial mastectomy for the treatment of breast cancer. What nursing diagnosis should the nurse prioritize when planning this patients postoperative care? A) Risk for Delayed Growth and Development related to prolonged hospitalization B) Risk for Decisional Conflict related to discharge planning C) Risk for Impaired Memory related to old age D) Risk for Infection related to reduced immune function

D Feedback: The lessened physiological reserve of older adults results in an increased risk for infection postoperatively. This physiological consideration is a priority over psychosocial considerations, which may or may not be applicable. Impaired memory is always attributed to a pathophysiological etiology, not advanced age.

A 59-year-old male patient is scheduled for a hemorrhoidectomy. The OR nurse should anticipate assisting the other team members with positioning the patient in what manner? A) Dorsal recumbent position B) Trendelenburg position C) Sims position Test Bank - Brunner & Suddarth's Textbook of Medical-Surgical Nursing 14e (Hinkle 2017) 376 D) Lithotomy position

D Feedback: The lithotomy position is used for nearly all perineal, rectal, and vaginal surgical procedures. The Sims or lateral position is used for renal surgery and the Trendelenburg position usually is used for surgery on the lower abdomen and pelvis. The usual position for surgery, called the dorsal recumbent position, is flat on the back, but this would be impracticable for rectal surgery.

A patient admitted with nephrotic syndrome is being cared for on the medical unit. When writing this patients care plan, based on the major clinical manifestation of nephrotic syndrome, what nursing diagnosis should the nurse include? A) Constipation related to immobility B) Risk for injury related to altered thought processes C) Hyperthermia related to the inflammatory process D) Excess fluid volume related to generalized edema

D Feedback: The major clinical manifestation of nephrotic syndrome is edema, so the appropriate nursing diagnosis is Excess fluid volume related to generalized edema. Edema is usually soft, pitting, and commonly occurs around the eyes, in dependent areas, and in the abdomen.

The nurse is caring for a 78-year-old female patient who is scheduled for surgery to remove her brain tumor. The patient is very apprehensive and keeps asking when she will get her preoperative medicine. The medicine is ordered to be given on call to OR. When would be the best time to give this medication? A) As soon as possible, in order to alleviate the patients anxiety B) As the patient is transferred to the OR bed C) When the porter arrives on the floor to take the patient to surgery D) After being notified by the OR and before other preoperative preparations

D Feedback: The nurse can have the medication ready to administer as soon as a call is received from the OR staff. It usually takes 15 to 20 minutes to prepare the patient for the OR. If the nurse gives the medication before attending to the other details of preoperative preparation, the patient will have at least partial benefit from the preoperative medication and will have a smoother anesthetic and operative course.

What nursing action should the nurse perform when caring for a patient undergoing diagnostic testing of the renal-urologic system? A) Withhold medications until 12 hours post-testing. B) Ensure that the patient knows the importance of temporary fluid restriction after testing. C) Inform the patient of his or her medical diagnosis after reviewing the results. D) Assess the patients understanding of the test results after their completion.

D Feedback: The nurse should ensure that the patient understands the results that are presented by the physician. Informing the patient of a diagnosis is normally the primary care providers responsibility. Withholding fluids or medications is not normally required after testing.

The nurse is collaborating with the wound-ostomy-continence (WOC) nurse to teach a patient how to manage her new ileal conduit in the home setting. To prevent leakage or skin breakdown, the nurse should encourage which of the following practices? A) Empty the collection bag when it is between one-half and two-thirds full. B) Limit fluid intake to prevent production of large volumes of dilute urine. C) Reinforce the appliance with tape if small leaks are detected. D) Avoid using moisturizing soaps and body washes when cleaning the peristomal area.

D Feedback: The patient is instructed to avoid moisturizing soaps and body washes when cleaning the area because they interfere with the adhesion of the pouch. To maintain skin integrity, a skin barrier or leaking pouch is never patched with tape to prevent accumulation of urine under the skin barrier or faceplate. Fluids should be encouraged, not limited, and the collection bag should not be allowed to become more than one-third full.

The nurse admitting a patient who is insulin dependent to the same-day surgical suite for carpal tunnel surgery. How should this patients diagnosis of type 1 diabetes affect the care that the nurse plans? A) The nurse should administer a bolus of dextrose IV solution preoperatively. B) The nurse should keep the patient NPO for at least 8 hours preoperatively. C) The nurse should initiate a subcutaneous infusion of long-acting insulin. D) The nurse should assess the patients blood glucose levels vigilantly.

D Feedback: The patient with diabetes who is undergoing surgery is at risk for hypoglycemia and hyperglycemia. Close glycemic monitoring is necessary. Dextrose infusion and prolonged NPO status are contraindicated. There is no specific need for an insulin infusion preoperatively.

The nurse is doing preoperative patient education with a 61-year-old male patient who has a 40-pack per year history of cigarette smoking. The patient will undergo an elective bunionectomy at a time that fits his work schedule in a few months. What would be the best instruction to give to this patient? Test Bank - Brunner & Suddarth's Textbook of Medical-Surgical Nursing 14e (Hinkle 2017) 343 A) Reduce smoking by 50% to prevent the development of pneumonia. B) Stop smoking at least 6 weeks before the scheduled surgery to enhance pulmonary function and decrease infection. C) Aim to quit smoking in the postoperative period to reduce the chance of surgical complications D) Stop smoking 4 to 8 weeks before the scheduled surgery to enhance pulmonary function and decrease infection.

D Feedback: The reduction of smoking will enhance pulmonary function; in the preoperative period, patients who smoke should be urged to stop 4 to 8 weeks before surgery.

The nursing instructor is talking with a group of medicalsurgical students about deep vein thrombosis (DVT). A student asks what factors contribute to the formation of a DVT. What would be the instructors best response? A) There is a genetic link in the formation of deep vein thrombi. B) Hypervolemia is often present in patients who go on to develop deep vein thrombi. C) No known factors contribute to the formation of deep vein thrombi; they just occur. D) Dehydration is a contributory factor to the formation of deep vein thrombi.

D Feedback: The stress response that is initiated by surgery inhibits the fibrinolytic system, resulting in blood hypercoagulability. Dehydration, low cardiac output, blood pooling in the extremities, and bedrest add to the risk of thrombosis formation. Hypervolemia is not a risk factor and there are no known genetic factors.

The nurse is admitting a patient to the medicalsurgical unit from the PACU. What should the nurse do to help the patient clear secretions and help prevent pneumonia? Test Bank - Brunner & Suddarth's Textbook of Medical-Surgical Nursing 14e (Hinkle 2017) 392 A) Encourage the patient to eat a balanced diet that is high in protein. B) Encourage the patient to limit his activity for the first 72 hours. C) Encourage the patient to take his medications as ordered. D) Encourage the patient to use the incentive spirometer every 2 hours.

D Feedback: To clear secretions and prevent pneumonia, the nurse encourages the patient to turn frequently, take deep breaths, cough, and use the incentive spirometer at least every 2 hours. These pulmonary exercises should begin as soon as the patient arrives on the clinical unit and continue until the patient is discharged. A balanced, high protein diet; visiting family in the waiting room; or taking medications as ordered would not help to clear secretions or prevent pneumonia

A patient has experienced excessive losses of bicarbonate and has subsequently developed an acidbase imbalance. How will this lost bicarbonate be replaced? A) The kidneys will excrete increased quantities of acid. B) Bicarbonate will be released from the adrenal medulla. C) Alveoli in the lungs will synthesize new bicarbonate. D) Renal tubular cells will generate new bicarbonate.

D Feedback: To replace any lost bicarbonate, the renal tubular cells generate new bicarbonate through a variety of chemical reactions. This newly generated bicarbonate is then reabsorbed by the tubules and returned to the body. The lungs and adrenal glands do not synthesize bicarbonate. Excretion of acid compensates for a lack of bicarbonate, but it does not actively replace it.

An adult patient is in the recovery room following a nephrectomy performed for the treatment of renal cell carcinoma. The patients vital signs and level of consciousness stabilized, but the patient then complains of severe nausea and begins to retch. What should the nurse do next? A) Administer a dose of IV analgesic. B) Apply a cool cloth to the patients forehead. C) Offer the patient a small amount of ice chips. D) Turn the patient completely to one side.

D Feedback: Turning the patient completely to one side allows collected fluid to escape from the side of the mouth if the patient vomits. After turning the patient to the side, the nurse can offer a cool cloth to the patients forehead. Ice chips can increase feelings of nausea. An analgesic is not administered for nausea and vomiting.

The nurse is caring for a patient who has a fluid volume deficit. When evaluating this patients urinalysis results, what should the nurse anticipate? A) A fluctuating urine specific gravity B) A fixed urine specific gravity C) A decreased urine specific gravity D) An increased urine specific gravity

D Feedback: Urine specific gravity depends largely on hydration status. A decrease in fluid intake will lead to an increase in the urine specific gravity. With high fluid intake, specific gravity decreases. In patients with kidney disease, urine specific gravity does not vary with fluid intake, and the patients urine is said to have a fixed specific gravity.

A 21-year-old patient is positioned on the OR bed prior to knee surgery to correct a sports-related injury. The anesthesiologist administers the appropriate anesthetic. The OR nurse should anticipate which of the following events as the teams next step in the care of this patient? A) Grounding B) Making the first incision C) Giving blood D) Intubating

D Feedback: When the patient arrives in the OR, the anesthesiologist or anesthetist reassesses the patients physical condition immediately prior to initiating anesthesia. The anesthetic is administered, and the patients airway is maintained through an intranasal intubation, oral intubation, or a laryngeal mask airway. Grounding or blood administration does not normally follow anesthetic administration immediately. An incision would not be made prior to intubation.

The care team is considering the use of dialysis in a patient whose renal function is progressively declining. Renal replacement therapy is indicated in which of the following situations? A) When the patients creatinine level drops below 1.2 mg/dL (110 mmol/L) B) When the patients blood urea nitrogen (BUN) is above 15 mg/dL C) When approximately 40% of nephrons are not functioning D) When about 80% of the nephrons are no longer functioning

D Feedback: When the total number of functioning nephrons is less than 20%, renal replacement therapy needs to be considered. Dialysis is an example of a renal replacement therapy. Prior to the loss of about 80% of the nephron functioning ability, the patient may have mild symptoms of compromised renal function, but symptom management is often obtained through dietary modifications and drug therapy. The listed creatinine and BUN levels are within reference ranges.

A patient has a flaccid bladder secondary to a spinal cord injury. The nurse recognizes this patients high risk for urinary retention and should implement what intervention in the patients plan of care? A) Relaxation techniques B) Sodium restriction C) Lower abdominal massage D) Double voiding

D Test Bank - Brunner & Suddarth's Textbook of Medical-Surgical Nursing 14e (Hinkle 2017) 1047 Feedback: To enhance emptying of a flaccid bladder, the patient may be taught to double void. After each voiding, the patient is instructed to remain on the toilet, relax for 1 to 2 minutes, and then attempt to void again in an effort to further empty the bladder. Relaxation does not affect the neurologic etiology of a flaccid bladder. Sodium restriction and massage are similarly ineffective.

A nurse is caring for a patient with COPD. The patients medication regimen has been recently changed and the nurse is assessing for therapeutic effect of a new bronchodilator. What assessment parameters suggest a consequent improvement in respiratory status? Select all that apply. A) Negative sputum culture B) Increased viscosity of lung secretions C) Increased respiratory rate D) Increased expiratory flow rate E) Relief of dyspnea

D, E Feedback: The relief of bronchospasm is confirmed by measuring improvement in expiratory flow rates and volumes (the force of expiration, how long it takes to exhale, and the amount of air exhaled) as well as by assessing the dyspnea and making sure that it has lessened. Increased respiratory rate and viscosity of secretions would suggest a worsening of the patients respiratory status. Bronchodilators would not have a direct result on the patients infectious process.


Ensembles d'études connexes

Physics Test 2 Misconception Q's (Ch 4 & 5), Physics Test 3 Misconceptual Questions, Misconception Q's Test 4

View Set

American Government Chapter 14 Quizzes

View Set

Chapter 4 Quiz:Managing Income Taxes

View Set

Chapter 11 Adaptive Reading Assignment

View Set